MI Vol 4
MI Vol 4
!
" # # " $ "
" " %" ##" !
! " #" " " % & '
(" ) * ("
+" , - + $ .
" " " % " " " %
" " & ! " " / " $ !
0 1 2
3 1 4" " -" % (% 5
#" "% " %
6 7) " " ."
# 6% #"
$ " ! &
% % "
"$ #" %"# #"8 "
% $
Vasile Cîrtoaje
▀▀▀▀▀▀▀▀▀▀▀▀▀▀▀▀▀▀▀▀▀▀▀▀▀
MATHEMATICAL
INEQUALITIES
▀▀▀▀▀▀▀▀▀▀▀▀▀▀▀▀▀▀▀▀▀▀▀▀▀
Volume 4
A Glosar 539
B Bibliography 549
i
ii Vasile Cîrtoaje
Chapter 1
The following statement is known as the Right Half Convex Function Theorem
(RHCF-Theorem).
Right Half Convex Function Theorem (Vasile Cîrtoaje, 2004). Let f be a real
function defined on an interval I and convex on I≥s , where s ∈ int(I). If
f (x) + (n − 1) f ( y) ≥ n f (s)
k ∈ {1, 2, . . . , n − 1}
so that
a1 ≤ · · · ≤ ak < s ≤ ak+1 ≤ · · · ≤ an .
1
2 Vasile Cîrtoaje
where
ak+1 + · · · + an
z= , z ∈ I.
n−k
Thus, it suffices to show that
Let b1 , . . . , bk be defined by
ai + (n − 1)bi = ns, i = 1, . . . , k.
We claim that
z ≥ b1 ≥ · · · ≥ bk > s,
which involves
b1 , . . . , bk ∈ I≥s .
Indeed, we have
b1 ≥ · · · ≥ b k ,
s − ak
bk − s = > 0,
n−1
and
z ≥ b1
because
···
f (ak ) + (n − 1) f (bk ) ≥ n f (s),
hence
f (a1 ) + · · · + f (ak ) + (n − 1)[ f (b1 ) + · · · + f (bk )] ≥ kn f (s),
f (a1 ) + · · · + f (ak ) ≥ kn f (s) − (n − 1)[ f (b1 ) + · · · + f (bk )].
According to this result, the inequality (2) is true if
which is equivalent to
n−k
p f (z) + (k − p) f (s) ≥ f (b1 ) + · · · + f (bk ), p= ≤ 1.
n−1
By Jensen’s inequality, we have
Since the decreasingly ordered vector A~k = (w, s, . . . , s) majorizes the decreasingly
ordered vector B~k = (b1 , b2 , . . . , bk ), this inequality follows from Karamata’s in-
equality for convex functions.
According to this result, the inequality (1) holds for all a1 , a2 , . . . , an ∈ I satisfying
a1 + a2 + · · · + an = ns1 if f (x 1 ) + (n − 1) f ( y1 ) ≥ n f (s1 ) for all x 1 , y1 ∈ I so that
x 1 ≤ s1 ≤ y1 and x 1 + (n − 1) y1 = ns1 . Thus, we need to show that if
f (x) + (n − 1) f ( y) ≥ n f (s)
f (x 1 ) + (n − 1) f ( y1 ) ≥ n f (s1 ) (3)
f (x 1 ) + (n − 1) f ( y2 ) ≥ n f (s),
x 1 + (n − 1) y2 = ns, y2 > s.
n f (s) − (n − 1) f ( y2 ) + (n − 1) f ( y1 ) ≥ n f (s1 ),
that is
(n − 1) f ( y1 ) + n f (s) ≥ (n − 1) f ( y2 ) + n f (s1 ).
Since
(n − 1) y1 + ns = (n − 1) y2 + ns1
and the decreasingly ordered vector C2n−1 ~ = ( y1 , . . . , y1 , s, . . . , s) majorizes the vec-
~
tor D2n−1 = ( y2 , . . . , y2 , s1 , . . . , s1 ), this inequality follows from Karamata’s inequality
for convex functions.
Similarly, we can prove the Left Half Convex Function Theorem (LHCF-Theorem).
4 Vasile Cîrtoaje
Left Half Convex Function Theorem. Let f be a real function defined on an interval
I and convex on I≤s , where s ∈ int(I). If
f (x) + (n − 1) f ( y) ≥ n f (s)
From the RHCF-Theorem and the LHCF-Theorem, we find the HCF-Theorem (Half
Convex Function Theorem).
a1 + a2 + · · · + an = ns
if and only if
f (x) + (n − 1) f ( y) ≥ n f (s)
for all x, y ∈ I so that x + (n − 1) y = ns.
If a1 , a2 , . . . , an ∈ I so that
a1 + a2 + · · · + an = S = const ant,
then
(a) E is minimum for a1 = a2 = · · · = an−1 ≤ an ;
(b) E is maximum for either a1 = a or a < a1 ≤ a2 = · · · = an .
Half Convex Function Method 5
Λ = {(a1 , a2 , . . . , an ) : a1 + a2 + · · · + an = S, a1 , a2 , . . . , an ∈ I},
b1 + bn−1
f (b1 ) + f (bn−1 ) > 2 f ,
2
while for bn−1 > c, by Karamata’s inequality for strictly concave functions we have
b2 + b n
f (b2 ) + f (bn ) < 2 f .
2
Clearly, all these results contradict the assumption that E is maximum at (b1 , b2 , . . . , bn ).
f (x) + (n − 1) f ( y) ≥ n f (s)
f (x) + (n − 1) f ( y) ≥ n f (s)
To prove this, we will show that the new condition H(x, y) ≥ 0 implies
f (x) + (n − 1) f ( y) ≥ n f (s)
f1 (x) ≥ n f (s),
where ns − x
f1 (x) = f (x) + (n − 1) f ( y) = f (x) + (n − 1) f .
n−1
From
ns − x
f10 (x) = f 0 (x) − f 0
n−1
= f (x) − f ( y)
0 0
n
= (x − s)H(x, y),
n−1
Half Convex Function Method 7
Note 3. From the proof of the RHCF-Theorem, it follows that the RHCF-Theorem,
the LHCF-Theorem and the HCF-Theorem are also valid in the case when f is de-
fined on I \ {u0 }, where u0 ∈ I<s for the RHCF-Theorem, and u0 ∈ I>s for the LHCF-
Theorem.
Note 4. The desired inequalities in the RHCF-Theorem, the LHCF-Theorem and the
HCF-Theorem become equalities for
a1 = a2 = · · · = an = s.
In addition, if there exist x, y ∈ I so that
x + (n − 1) y = ns, f (x) + (n − 1) f ( y) = n f (s), x 6= y,
then the equality holds also for
a1 = x, a2 = · · · = an = y
(or any cyclic permutation). Notice that these equality conditions are equivalent to
x + (n − 1) y = ns, h(x, y) = 0
(x < y for the RHCF-Theorem, and x > y for the LHCF-Theorem).
Note 5. The part (a) in LCRCF-Theorem is also true in the case where I = (a, ∞)
and f (a+ ) = ∞.
Note 6. Similarly, we can extend the weighted Jensen’s inequality to right and left
half convex functions establishing the WRHCF-Theorem, the WLHCF-Theorem and
the WHCF-Theorem (Vasile Cîrtoaje, 2008).
WHCF-Theorem. Let p1 , p2 , . . . , pn be positive real numbers so that
p1 + p2 + · · · + pn = 1, p = min{p1 , p2 , . . . , pn },
and let f be a real function defined on an interval I and convex on I≥s or I≤s , where
s ∈ int(I). The inequality
p1 f (a1 ) + p2 f (a2 ) + · · · + pn f (an ) ≥ f (p1 a1 + p2 a2 + · · · + pn an )
holds for all a1 , a2 , . . . , an ∈ I so that
p1 a1 + p2 a2 + · · · + pn an = s,
if and only if
p f (x) + (1 − p) f ( y) ≥ f (s)
for all x, y ∈ I satisfying
px + (1 − p) y = s.
8 Vasile Cîrtoaje
Half Convex Function Method 9
1.2 Applications
3(a4 + b4 + c 4 ) + a2 + b2 + c 2 + 6 ≥ 6(a3 + b3 + c 3 ).
1 − 2n
1.2. If a1 , a2 , . . . , an ≥ so that a1 + a2 + · · · + an = n, then
n−2
−n
1.3. If a1 , a2 , . . . , an ≥ so that a1 + a2 + · · · + an = n, then
n−2
1 1 1
n 2
+ + ··· + − n ≥ 4(n − 1)(a12 + a22 + · · · + an2 − n).
a1 a2 an
1 1 1
2
+ 2 + · · · + 2 ≥ a12 + a22 + · · · + a82 .
a1 a2 a8
1 1 1
1.11. If a1 , a2 , . . . , an are positive real numbers so that + +···+ = n, then
a1 a2 an
p
n−1
a12 + a22 + ··· + an2 −n≥2 1+ (a1 + a2 + · · · + an − n).
n
1.13. If a, b, c are nonnegative real numbers, no two of which are zero, then
1 1 1 2 1 1 1
+ + ≤ + + .
3a + b + c 3b + c + a 3c + a + b 5 b + c c + a a + b
p
1.14. If a, b, c, d ≥ 3 − 7 so that a + b + c + d = 4, then
1 1 1 1 4
+ + + ≥ .
2 + a2 2 + b2 2 + c 2 2 + d 2 3
Half Convex Function Method 11
p
1.15. If a1 , a2 , . . . , an ∈ [− n, n − 2] so that a1 + a2 + · · · + an = n, then
1 1 1 n
+ + ··· + ≤ .
n + a1 n + a2
2 2
n + an
2 n+1
3−a 3− b 3−c 3
+ + ≥ .
9+a 2 9+ b 2 9+c 2 5
n2
k≥ ,
4(n − 1)
then
a1 (a1 − 1) a2 (a2 − 1) an (an − 1)
+ + · · · + ≥ 0.
a12 + k a22 + k an2 + k
p
1.26. If a1 , a2 , . . . , an ≥ n − 1 − n2 − n + 1 so that a1 + a2 + · · · + an = n, then
(a2 + 1)(b2 + 1)(c 2 + 1)(d 2 + 1)(e2 + 1) ≥ (a + 1)(b + 1)(c + 1)(d + 1)(e + 1).
then
1 1 1
ka1 + ka2 + · · · kan + ≥ (k + 1)n .
a1 a2 an
1.40. If a, b, c are nonnegative real numbers, no two of which are zero, then
v v v
t 48a t 48b t 48c
1+ + 1+ + 1+ ≥ 15.
b+c c+a a+b
Half Convex Function Method 15
ln 3
k ≥ k0 , k0 = − 1 ≈ 0.585,
ln 2
then k k k
2a 2b 2c
+ + ≥ 3.
b+c c+a a+b
p
1.44. If a, b, c ∈ [1, 7 + 4 3], then
v v v
t 2a t 2b t 2c
+ + ≥ 3.
b+c c+a a+b
ln 2
0 < k ≤ k0 , k0 = ≈ 1.71,
ln 3 − ln 2
then
a k (b + c) + b k (c + a) + c k (a + b) ≤ 6.
a+b k b+c k c + a k
a + b +c +3≥2
k k k
+2 +2 .
2 2 2
where p p p
k = ( n − 1)( n + n − 1).
7
1.53. If a1 , a2 , a3 , a4 , a5 ≤so that a1 + a2 + a3 + a4 + a5 = 5, then
2
a1 a2 a3 a4 a5
+ + + + ≤ 1.
a12 − a1 + 5 a22 − a2 + 5 a32 − a3 + 5 a42 − a4 + 5 a52 − a5 + 5
then
a12 a22 an2 n
+ + ··· + ≥ .
ka12 + a2 + · · · + an a1 + ka22 + · · · + an a1 + a2 + · · · + kan2 k+n−1
1
1.56. Let a1 , a2 , . . . , an ∈ [0, n] so that a1 + a2 + · · · + an ≥ n. If 0 < k ≤ , then
n
a1 − 1 a2 − 1 an − 1
+ + ··· + ≥ 0.
ka12 + a2 + · · · + an a1 + ka2 + · · · + an
2
a1 + a2 + · · · + kan2
1
1.58. If a, b, c, d ≥ p so that abcd = 1, then
1+ 6
1 1 1 1 4
+ + + ≤ .
a+2 b+2 c+2 d +2 3
a2 + b2 + c 2 − 3 ≥ 2(ab + bc + ca − a − b − c).
18 Vasile Cîrtoaje
a2 + b2 + c 2 − 3 ≥ 18(a + b + c − ab − bc − ca).
1 1 1 n
+ + ··· + ≥ .
1 + pa1 + qa1 1 + pa2 + qa2
2 2
1 + pan + qan2 1+p+q
1.64. Let a, b, c, d be positive real numbers so that abcd = 1. If p and q are non-
negative real numbers so that p + q = 3, then
1 1 1 1
+ + + ≥ 1.
1 + pa + qa3 1 + pb + qb3 1 + pc + qc 3 1 + pd + qd 3
1 1 1
+ + ··· + ≥ 1.
1 + a1 + · · · + a1
n−1
1 + a2 + · · · + a2
n−1
1 + an + · · · + ann−1
k ≥ n2 − 1,
then
1 1 1 n
+p + ··· + p ≥p .
1 + ka1 1 + ka2 1 + kan 1+k
p
Half Convex Function Method 19
1 1 1
a1 + a2 + · · · + an + n(n − 2) ≥ (n − 1)
n−1 n−1 n−1
+ + ··· + .
a1 a2 an
1 a1 1 a2 1 an
1− + 1− + ··· + 1 − ≤ n − 1.
n n n
a6 b6 c6
+ + ≥ 1.
1 + 2a5 1 + 2b5 1 + 2c 5
a + b + c + d = 4,
then
a b c d 4
+ 3 + 3 + 3 ≤ .
3a3 + 2 3b + 2 3c + 2 3d + 2 5
1.3 Solutions
3(a4 + b4 + c 4 ) + a2 + b2 + c 2 + 6 ≥ 6(a3 + b3 + c 3 ).
a+b+c
f (a) + f (b) + f (c) ≥ 3 f (s), s= = 1,
3
where
f (u) = 3u4 − 6u3 + u2 , u ∈ R.
From
f 00 (u) = 2(18u2 − 18u + 1),
it follows that f 00 (u) > 0 for u ≥ 1, hence f is convex on [s, ∞). By the RHCF-
Theorem, it suffices to show that f (x) + 2 f ( y) ≥ 3 f (1) for all real x, y so that
x + 2 y = 3. Let
E = f (x) + 2 f ( y) − 3 f (1).
We have
n−1
(a12 − a1 )2 + (a22 − a2 )2 + · · · + (an2 − an )2 ≥ (a12 + a22 + · · · + an2 − n),
n2 − 3n + 3
24 Vasile Cîrtoaje
1 − 2n
P 1.2. If a1 , a2 , . . . , an ≥ so that a1 + a2 + · · · + an = n, then
n−2
a13 + a23 + · · · + an3 ≥ n.
−n
P 1.3. If a1 , a2 , . . . , an ≥ so that a1 + a2 + · · · + an = n, then
n−2
a13 + a23 + · · · + an3 ≥ a12 + a22 + · · · + an2 .
Half Convex Function Method 25
and
We have
and
a1 = n, a2 = a3 = · · · = an = 0
then
a4 + b4 + c 4 − 3 ≥ k(a3 + b3 + c 3 − 3).
Solution. Denote
n k−1 n k−2 n k−3
m = (n − 1) −1 = + + · · · + 1,
n−1 n−1 n−1
and write the inequality as
a1 + a2 + · · · + an
f (a1 ) + f (a2 ) + · · · + f (an ) ≥ n f (s), s= = 1,
n
where
f (u) = uk − mu2 , u ∈ [0, n].
We will show that f is convex on [1, n]. Since
k(k − 1) 1 1
≥ (k − 1) 1 + + · · · + .
2 2 k−1
This is true if
k 1 1
≥ 1 + + ··· + ,
2 2 k−1
which can be easily proved by induction. According to the RHCF-Theorem and Note
1, we only need to show that h(x, y) ≥ 0 for x, y ≥ 0 so that x + (n − 1) y = n,
where
g(x) − g( y) f (u) − f (1)
h(x, y) = , g(u) = .
x−y u−1
We have
(uk − 1) − m(u2 − 1)
g(u) = = (uk−1 + uk−2 + · · · + 1) − m(u + 1),
u−1
30 Vasile Cîrtoaje
x k−1 − y k−1 x k−2 − y k−1
h(x, y) = + + ··· + 1 − m
x−y x−y
k−2 j+1
X x − y j+1 n j
= − .
j=1
x−y n−1
h n i
It suffices to show that f j ( y) ≥ 0 for y ∈ 0, and j = 1, 2, . . . , k − 2, where
n−1
n j
f j ( y) = x j + x j−1 y + · · · + x y j−1 + y j − , x = n − (n − 1) y.
n−1
For j = 1, we have
n (n − 2)x
f1 ( y) = x + y − = ≥ 0.
n−1 n−1
For j ≥ 2, from x 0 = −(n − 1) and n − 1 ≥ k − 2 ≥ j, we get
which is equivalent to
3 X 3n − 1 X
ai a j ak + n2 ≥ ai a j ,
n − 2 1≤i< j<k≤n n − 1 1≤i< j≤n
Half Convex Function Method 31
a1 + a2 + · · · + an = n,
then
Solution. Denote
nk−1 − 1
m= = nk−2 + nk−3 + · · · + 1,
n−1
and write the inequality as
a1 + a2 + · · · + an
f (a1 ) + f (a2 ) + · · · + f (an ) ≥ n f (s), s= = 1,
n
where
f (u) = mu2 − uk , u ∈ [0, n].
We will show that f is convex on [0, 1]. Since
This is true if
k(k − 1)
2k−2 + 2k−3 + · · · + 1 ≥ ,
2
which is equivalent to
k(k − 1)
2k−1 − 1 ≥ ,
2
2k ≥ k2 − k + 2.
Since
k k k
2 = (1 + 1) ≥ 1 +
k k
+ +
1 2 3
k(k − 1) k(k − 1)(k − 2)
=1+k+ + ,
2 6
it suffices to show that
k(k − 1) k(k − 1)(k − 2)
1+k+ + ≥ k2 − k + 2,
2 6
which reduces to
(k − 1)(k − 2)(k − 3) ≥ 0.
According to the LHCF-Theorem and Note 1, we only need to show that h(x, y) ≥ 0
for x, y ≥ 0 so that x + (n − 1) y = n, where
We have
m(u2 − 1) − (uk − 1)
g(u) = = m(u + 1) − (uk−1 + uk−2 + · · · + 1)
u−1
and
x k−1 − y k−1 x k−2 − y k−1
h(x, y) = m − − − ··· − 1
x−y x−y
k−1 k−1
k−2 k−2
2 2
x − y x − y x − y
= nk−2 − + nk−3 − + ··· + n − .
x−y x−y x−y
It suffices to show that
x j+1 − y j+1
nj ≥ , j = 1, 2, . . . , k − 2.
x−y
n − (x + y) = x + (n − 1) y − (x + y) = (n − 2) y ≥ 0.
and
x j+1 − y j+1
= x j + x j−1 y + · · · + x y j−1 + y j .
x−y
The equality holds for a1 = n and a2 = a3 = · · · = an = 0 (or any cyclic permuta-
tion).
Remark. For k = 3 and k = 4, we get the following statements (Vasile C. , 2002):
• If a1 , a2 , . . . , an are nonnegative real numbers so that a1 + a2 + · · · + an = n, then
a1 = n, a2 = a3 = · · · = an = 0
a1 = n, a2 = a3 = · · · = an = 0
1 1 1
n 2
+ + ··· + − n ≥ 4(n − 1)(a12 + a22 + · · · + an2 − n).
a1 a2 an
2n2
f 00 (u) = − 8(n − 1) ≥ 2n2 − 8(n − 1) = 2(n − 2)2 ≥ 0.
u3
Thus, f is convex on I≤s . By the LHCF-Theorem and Note 1, it suffices to show that
h(x, y) ≥ 0 for x, y > 0 so that x + (n − 1) y = n, where
6
f 00 (u) = − 2 ≥ 6 − 2 > 0.
u4
Thus, f is convex on (0, s]. By the LHCF-Theorem and Note 1, it suffices to show
that h(x, y) ≥ 0 for x, y > 0 so that x + 7 y = 8, where
We have
1 1
g(u) = −u − 1 − − 2
u u
and
1 x+y
h(x, y) = −1 + + 2 2.
xy x y
From 8 = x + 7 y ≥ 2 7x y, we get x y ≤ 16/7. Therefore,
p
1 1 1
P 1.11. If a1 , a2 , . . . , an are positive real numbers so that + +···+ = n, then
a1 a2 an
p
n−1
a12 + a22 + ··· + an2 −n≥2 1+ (a1 + a2 + · · · + an − n).
n
where p
1 2k n−1
f (u) = 2 − , k =1+ , u ∈ (0, n).
u u n
For u ∈ (0, 1], we have
p
6 − 4ku 6 − 4k 2( n − 1 − 1)2
f (u) =
00
≥ = ≥ 0.
u4 u4 nu4
Thus, f is convex on (0, s]. By the LHCF-Theorem and Note 1, it suffices to show
that h(x, y) ≥ 0 for x, y > 0 so that x + (n − 1) y = n, where
g(x) − g( y) f (u) − f (1)
h(x, y) = , g(u) = .
x−y u−1
We have
−1 2k − 1
g(u) = +
u2 u
and
1 1 1
h(x, y) = + + 1 − 2k .
xy x y
We only need to show that
1 1
+ ≥ 2k − 1.
x y
Indeed, using the Cauchy-Schwarz inequality, we get
p p
1 1 (1 + n − 1)2 (1 + n − 1)2
+ ≥ = = 2k − 1,
x y x + (n − 1) y n
p
with equality for x = n − 1 y. From x + (n − 1) y = n and h(x, y) = 0, we get
n n
x= p , y= p .
1+ n−1 n−1+ n−1
In accordance with Note 4, the original equality holds for a1 = a2 = · · · = an = 1,
and also for
p p
1+ n−1 n−1+ n−1
a1 = , a2 = a3 = · · · = an =
n n
(or any cyclic permutation).
p p p p p
Solution. Replacing a, b, c, d, e by a, b, c, d, e, respectively, we need to
prove that
a+b+c+d+e
f (a) + f (b) + f (c) + f (d) + f (e) ≥ 5 f (s), s= = 1,
5
where p
1 p 4(1 + 5)
f (u) = p + k u, k= ≈ 2.59, u ∈ (0, 5).
u 5
For u ∈ (0, 1], we have
3 − ku
f 00 (u) = p > 0;
4u2 u
therefore, f is convex on (0, s]. By the LHCF-Theorem and Note 1, it suffices to
show that h(x, y) ≥ 0 for x, y > 0 so that x + 4 y = 5. We have
p
f (u) − f (1) k u − 1
g(u) = = p
u−1 u+ u
and p p p
g(x) − g( y) x + y +1−k xy
h(x, y) = =p p p p p .
x−y x y( x + y)( x + 1)( y + 1)
Thus, we only need to show that
p p p
x+ y + 1 − k x y ≥ 0,
which is true if
p p
2 4 x y + 1 − k x y ≥ 0.
Let
p
t= 4
x y.
From
p
5 = x + 4 y ≥ 4 x y = 4t 2 ,
we get p
5
t≤ .
2
Thus,
p p
2 4 x y + 1 − k x y = 2t + 1 − kt 2
2 1
= 1 − p t 1 + 2 1 + p t ≥ 0.
5 5
The equality holds for a = b = c = d = e = 1.
38 Vasile Cîrtoaje
P 1.13. If a, b, c are nonnegative real numbers, no two of which are zero, then
1 1 1 2 1 1 1
+ + ≤ + + .
3a + b + c 3b + c + a 3c + a + b 5 b + c c + a a + b
p
P 1.14. If a, b, c, d ≥ 3 − 7 so that a + b + c + d = 4, then
1 1 1 1 4
+ + + ≥ .
2+a 2 2+ b 2 2+c 2 2+d 2 3
a+b+c+d
f (a) + f (b) + f (c) + f (d) ≥ 4 f (s), s= = 1,
4
where
1 p
f (u) = , u ≥ 3 − 7.
2 + u2
For u ≥ s = 1, f (u) is convex because
3(3u2 − 2)
f (u) =
00
> 0.
(2 + u2 )3
f (u) − f (1) −1 − u
g(u) = =
u−1 3(2 + u2 )
and
g(x) − g( y) xy + x + y −2
h(x, y) = = ,
x−y 3(2 + x 2 )(2 + y 2 )
where
p p
−x 2 + 6x − 2 (3 + 7 − x)(x − 3 + 7)
xy + x + y −2= =
3p p 3
(−1 + 7 + 3 y)(x − 3 + 7)
= ≥ 0.
3
In accordance with Note 4, the equality holds for a = b = c = d = 1, and also for
p
p 1+ 7
a = 3 − 7, b=c=d=
3
(or any cyclic permutation).
Remark. Similarly, we can prove the following generalization:
p
• If a1 , a2 , . . . , an ≥ n − 1 − n2 − 3n + 3 so that a1 + a2 + · · · + an = n, then
1 1 1 n
+ + ··· + ≥ ,
2 + a1 2 + a2
2 2
2 + an
2 3
p
P 1.15. If a1 , a2 , . . . , an ∈ [− n, n − 2] so that a1 + a2 + · · · + an = n, then
1 1 1 n
+ + ··· + ≤ .
n + a1 n + a2
2 2
n + an
2 n+1
2(n − u2 )
f 00 (u) = ≥ 0,
(n + u2 )3
p
hence f is convex on [− n, s]. By the LHCF-Theorem and Note 1, it suffices to
p
show that h(x, y) ≥ 0 for x, y ∈ [− n, n − 2] so that x + (n − 1) y = n. We have
and
g(x) − g( y) n− x − y − xy
h(x, y) = =
x−y (n + 1)(n + x 2 )(n + y 2 )
(n − x)(n − 2 − x)
= 2 ≥ 0.
(n − 1)(n + x 2 )(n + y 2 )
2
a1 = n − 2, a2 = a3 = · · · = an =
n−1
(or any cyclic permutation).
3−a 3− b 3−c 3
+ + ≥ .
9+a 2 9+ b 2 9+c 2 5
3
The equality holds for a = b = c = 1, and also for a = 0 and b = c = (or any
2
cyclic permutation).
1 00 12u2 − 6u − 1
f (u) = > 0.
2 (1 − u + 2u2 )3
Thus, f is convex on [s, 3]. By the RHCF-Theorem and Note 1, it suffices to show
that h(x, y) ≥ 0 for x, y ≥ 0 so that x + 2 y = 3, where
3
The equality holds for a = b = c = 1, and also for a = 0 and b = c = (or any
2
cyclic permutation).
2k(3ku2 − 1) 2k(3k − 1)
f 00 (u) = ≥ > 0.
(1 + ku2 )3 (1 + ku2 )3
Thus, f is convex on [s, n]. By the RHCF-Theorem and Note 1, it suffices to show
that h(x, y) ≥ 0 for x, y ≥ 0 so that x + (n − 1) y = n. We have
and
g(x) − g( y) k2 (x + y + x y) − k
h(x, y) = = .
x−y (1 + k)(1 + kx 2 )(1 + k y 2 )
We need to show that
k(x + y + x y) − 1 ≥ 0.
Indeed, we have
1 x(2n − 2 − x)
k(x + y + x y) − 1 ≥ 1 − (x + y + x y) − 1 = ≥ 0.
n n
1
The equality holds for a1 = a2 = · · · = an = 1. If k = 1 − , then the equality
n
holds also for
n
a1 = 0, a2 = a3 = · · · = an =
n−1
(or any cyclic permutation).
46 Vasile Cîrtoaje
n−1
0<k≤ ,
n2 − n + 1
then
1 1 1 n
+ + ··· + ≤ .
1 + ka1 1 + ka2
2 2
1 + kan
2 1+k
Solution. Replacing all negative numbers ai by −ai , we need to show the same
inequality for
a1 , a2 , . . . , an ≥ 0, a1 + a2 + · · · + an ≥ n.
Since the left side of the desired inequality is decreasing with respect to each ai , is
sufficient to consider that a1 + a2 + · · · + an = n. Write this inequality as
a1 + a2 + · · · + an
f (a1 ) + f (a2 ) + · · · + f (an ) ≥ n f (s), s= = 1,
n
where
−1
f (u) = , u ∈ [0, n].
1 + ku2
For u ∈ [0, 1], we have
2k(1 − 3ku2 )
f 00 (u) = ≥ 0,
(1 + ku2 )3
since
3(n − 1) (n − 2)2
1 − 3ku2 ≥ 1 − 3k ≥ 1 − = ≥ 0.
n2 − n + 1 n2 − n + 1
Thus, f is convex on [0, s]. By the LHCF-Theorem and Note 1, it suffices to show
that h(x, y) ≥ 0 for x, y ≥ 0 so that x + (n − 1) y = n. We have
and
g(x) − g( y) k − k2 (x + y + x y)
h(x, y) = = .
x−y (1 + k)(1 + kx 2 )(1 + k y 2 )
It suffices to show that
1 − k(x + y + x y) ≥ 0.
Indeed, we have
n−1 (x − n + 1)2
1 − k(x + y + x y) ≥ 1 − (x + y + x y) = ≥ 0.
n2 − n + 1 n2 − n + 1
Half Convex Function Method 47
n−1
The equality holds for a1 = a2 = · · · = an = 1. If k = , then the equality
n2 −n+1
holds also for
1
a1 = n − 1, a2 = a3 = · · · = an =
n−1
(or any cyclic permutation).
f (u) − f (1) 1
g(u) = =
u−1 (n − 2u)2
and
g(x) − g( y) 4(n − x − y) 4(n − 2) y
h(x, y) = = = ≥ 0.
x−y (n − 2x)2 (n − 2 y)2 (n − 2x)2 (n − 2 y)2
p
P 1.26. If a1 , a2 , . . . , an ≥ n − 1 − n2 − n + 1 so that a1 + a2 + · · · + an = n, then
Since
f (u) − f (1) −u − 1
g(u) = = ,
u−1 (u + 2)2
g(x) − g( y) x+ y+xy −x 2 + 2(n − 1)x + n
h(x, y) = = = ,
x−y (x + 2)2 ( y + 2)2 (n − 1)(x + 2)2 ( y + 2)2
we need to show that
p p
n−1− n2 − n + 1 ≤ x ≤ n − 1 + n2 − n + 1.
f (x) + (n − 1) f ( y) ≥ n f (1)
n2
The equality holds for a1 = a2 = · · · = an = 1. If k = , then the equality
n−1
holds also for
n(n − 1)2 n
a1 = , a2 = a3 = · · · = an =
n2 − 2n + 2 (n − 1)(n2 − 2n + 2)
(or any cyclic permutation).
f (x) + (n − 1) f ( y) ≥ n f (1)
y0 1 −1 2k(1 + nx − 2x)
g10 (x) = − + 2k(x − y y 0 ) = n + .
y x x(n − x) (n − 1)2
For 0 < x ≤ 1, g10 (x) has the same sign as
−(n − 1)2
h(x) = + x(n − x)(1 + nx − 2x).
2k
Since
−(n − 1)2 1
h(0) = < 0, h(1) = (n − 1) 1 −2
> 0,
2k 2k
it follows that there is x 1 ∈ (0, 1) so that h(x 1 ) = 0, h(x) < 0 for x ∈ [0, x 1 ) and
h(x) > 0 for x ∈ (x 1 , 1]. Therefore, g1 is strictly decreasing on (0, x 1 ] and strictly
increasing on [x 1 , 1]. Since g1 (0+ ) = ∞ and g1 (1) = 0, there is x 2 ∈ (0, x 1 ) so that
g1 (x 2 ) = 0, g1 (x) > 0 for x ∈ (0, x 2 ) and g1 (x) < 0 for x ∈ (x 2 , 1). Consequently, g
is strictly increasing on [0, x 2 ] and strictly decreasing on [x 2 , 1]. Because g(0) > 0
and g(1) = 0, it follows that g(x) ≥ 0 for x ∈ [0, 1]. The proof is completed.
The equality holds for a1 = a2 = · · · = an = 1.
a+b+c+d
f (a) + f (b) + f (c) + f (d) ≥ 4 f (s), s= = 1,
4
where
f (u) = − ln(3u2 + 1), u ∈ [0, 4].
Half Convex Function Method 57
f (x) + 3 f ( y) ≥ 4 f (1)
for 0 ≤ x ≤ 1 ≤ y so that x + 3 y = 4; that is, to show that g(x) ≥ 0 for x ∈ [0, 1],
where
4− x
g(x) = f (x) + 3 f ( y) − 4 f (1), y= .
3
Since y 0 (x) = −1/3, we have
−6x 6y
g 0 (x) = f 0 (x) + 3 y 0 f 0 ( y) = +
3x + 1 3 y 2 + 1
2
(a2 + 1)(b2 + 1)(c 2 + 1)(d 2 + 1)(e2 + 1) ≥ (a + 1)(b + 1)(c + 1)(d + 1)(e + 1).
Solution. Consider the nontrivial case a, b, c, d, e > −1, and write the inequality
as
a+b+c+d+e
f (a) + f (b) + f (c) + f (d) + f (e) ≥ n f (s), s= = 1,
5
where
f (u) = ln(u2 + 1) − ln(u + 1), u > −1.
For u ∈ (−1, 1], we have
2(1 − u2 ) 1
f 00 (u) = + > 0.
(u2 + 1)2 (u + 1)2
58 Vasile Cîrtoaje
f 0 (x) − f 0 ( y) 2(1 − x y) 1
H(x, y) = = 2 + ;
x−y (x + 1)( y + 1) (x + 1)( y + 1)
2
(x 2 + 1)( y 2 + 1)
2(1 − x y) + ≥ 0.
(x + 1)( y + 1)
Since
x2 + 1 x +1 y2 + 1 y +1
≥ , ≥ ,
x +1 2 y +1 2
it suffices to prove that
(x + 1)( y + 1)
2(1 − x y) + ≥ 0,
4
which is equivalent to
x + y + 9 − 7x y ≥ 0,
28x 2 − 38x + 14 ≥ 0,
(28x − 19)2 + 31 ≥ 0.
The equality holds for a = b = c = d = e = 1.
From
3 − ku
f 00 (u) =
,
4u5/2
it follows that f is convex on (0, 1]. Thus, according to the LHCF-Theorem and Note
1, it suffices to show that h(x, y) ≥ 0 for x ≥ 1 ≥ y > 0 such that x + (n − 1) y = n,
where
g(x) − g( y) f (u) − f (1)
h(x, y) = , g(u) = .
x−y u−1
We have
k 1
g(u) = p − p
u+1 u+ u
and p p
p p p p x + y +1
( x + y)( x + 1)( y + 1)h(x, y) = −k + p .
xy
So, we need to show that p p
x + y +1
p ≥ k.
xy
Since p p p
x+ y ≥ 2 4 x y,
it suffices to show that p
2 4 xy +1
p ≥ k,
xy
which is equivalent to
1 2
p +p ≥ k.
xy 4
xy
From Æ
n = x + (n − 1) y ≥ 2 (n − 1)x y,
we get p
1 2 n−1
p ≥ ,
xy n
hence
p v p
1 2 2 n−1 2 n−1
t
p +p ≥ +2 ≥ k.
xy 4
xy n n
The proof is completed. The equality holds for a1 = a2 = · · · = an = 1.
Remark. Since
p v p
2 n−1 2 n−1
t
1< +2
n n
for n ≤ 134, the following inequality holds for a1 , a2 , . . . , a134 > 0 such that a1 +
a2 + · · · + a134 = 134:
p 1 1 1
a1 + a2 + · · · + a134 + p + p + · · · + p
p p
≥ 268.
a1 a2 a134
60 Vasile Cîrtoaje
Since
p v p
2 n−1 2 n−1
t
2< +2
n n
for n ≤ 12, the following inequality holds for a1 , a2 , . . . , a12 > 0 such that a1 + a2 +
· · · + a12 = 12:
p 1 1 1
2( a1 + a2 + · · · + a12 ) + p + p + · · · + p
p p
≥ 36.
a1 a2 a12
1 n
1 1 1 p
p p p
p − a1 p − a2 · · · p − an ≥ n− p .
a1 a2 an n
a1 + a2 + · · · + an 1
f (a1 ) + f (a2 ) + · · · + f (an ) ≥ n f (s), s= = ,
n n
where
1 p 1
f (u) = ln p − u = ln(1 − u) − ln u, u ∈ (0, 1).
u 2
From
−1 1 1 − 2u − u2
f 0 (u) = − , f 00 (u) = 2 ,
1 − u 2u 2u (1 − u)2
p
it follows that f 00 (u) ≥ 0 for u ∈ (0, 2 − 1]. Since
1 1 p
s= ≤ < 2 − 1,
n 3
f is convex on (0, s]. Thus, we can apply the LHCF-Theorem.
First Solution. By the LHCF-Theorem, it suffices to show that
1
f (x) + (n − 1) f ( y) ≥ n f
n
f 0 (x) − f 0 ( y)
H(x, y) = .
x−y
We have
1− x − y − xy n( y + 1) − y − 3
H(x, y) = =
2x y(1 − x)(1 − y) 2x(1 − x)(1 − y)
3( y + 1) − y − 3 y
≥ = > 0.
2x(1 − x)(1 − y) x(1 − x)(1 − y)
On the other hand, by the AM-GM inequality and the Cauchy-Schwarz inequality,
we have
n v !n
n n n
1 Xp t1 1 n
Y u X
(1 + ai ) ≤ 1 + ai ≤ 1 + ai = 1 + p
p
.
i=1
n i=1
n i=1
n
1+ x
On the other hand, since the function f (x) = ln is convex on (0, 1), by
1− x
Jensen’s inequality we have
a1 + a2 + · · · + an n
n 1+
1 + ai n+1 n
n
Y
≥ = .
1 − ai a1 + a2 + · · · + an n−1
i=1 1−
n
Multiplying these inequalities yields the following result (Kee-Wai Lau, 2000):
• If a1 , a2 , . . . , an (n ≥ 3) are positive real numbers so that a1 + a2 + · · · + an = 1,
then
1 1 1 1 n
− a1 − a2 · · · − an ≥ n − ,
a1 a2 an n
with equality for a1 = a2 = · · · = an = 1/n.
then
1 1 1
ka1 + ka2 + · · · kan + ≥ (k + 1)n .
a1 a2 an
(Vasile C., 2006)
Solution. Write the inequality as
a1 + a2 + · · · + an
f (a1 ) + f (a2 ) + · · · + f (an ) ≥ n f (s), s= = 1,
n
where
1
f (u) = ln ku + , u ∈ (0, n).
u
We have
ku2 − 1 1 + 4ku2 − k2 u4
f 0 (u) = , f 00 (u) = .
u(ku2 + 1) u2 (ku2 + 1)2
For u ∈ (0, 1], we get f 00 (u) > 0 since
f 0 (x) − f 0 ( y)
H(x, y) = .
x−y
Since
1 + k(x + y)2 − k2 x 2 y 2 k[(x + y)2 − kx 2 y 2 ]
H(x, y) = > ,
x y(kx 2 + 1)(k y 2 + 1) x y(kx 2 + 1)(k y 2 + 1)
it suffices to show that p
x+y≥ k x y.
Indeed, by the Cauchy-Schwarz inequality, we have
p
(x + y)[(n − 1) y + x] ≥ ( n − 1 + 1)2 x y,
hence p
1 p
2 n − 1 p
x + y ≥ ( n − 1 + 1)2 x y = 1 + x y ≥ k x y.
n n
The equality holds for a1 = a2 = · · · = an = 1.
−1
a, b, c, d ≥ , a + b + c + d = 4,
2
then
1 1 1 1 1 1 1 1
3 2 + 2 + 2 + 2 + + + + ≥ 16.
a b c d a b c d
Solution. Write the inequality as
a+b+c+d
f (a) + f (b) + f (c) + f (d) ≥ 4 f (s), s= = 1,
4
where
3 1 −1 11
f (u) = 2 + , u ∈ I = , \ {0},
u u 2 2
is convex on I≥s (because 3/u2 and 1/u are convex). By the RHCF-Theorem, Note
1 and Note 3, it suffices to prove that h(x, y) ≥ 0 for x, y ∈ I so that
x + 3 y = 4,
where
g(x) − g( y) f (u) − f (1)
h(x, y) = , g(u) = .
x−y u−1
64 Vasile Cîrtoaje
Indeed, we have
4 3
g(u) = − − 2 ,
u u
4x y + 3x + 3 y 2(1 + 2x)(6 − x)
h(x, y) = = ≥ 0.
x2 y2 3x 2 y 2
In accordance with Note 4, the equality holds for a = b = c = d = 1, and also
for
−1 3
a= , b=c=d=
2 2
(or any cyclic permutation).
where
a1 + a2 + · · · + an
s= =1
n
and
p p
s
n
f (u) = u u + k u, k= , u ∈ [0, n].
n−1
For u ≥ 1, we have
3u − k 3−k
f 00 (u) = p ≥ p > 0.
4u u 4u u
Therefore, f is convex on [s, n]. According to the RHCF-Theorem and Note 1, it
suffices to show that h(x, y) ≥ 0 for x, y ≥ 0 so that x + (n − 1) y = n. Since
and p p p
g(x) − g( y) x + y+ xy−k
h(x, y) = = p p p p ,
x−y ( x + y)( x + 1)( y + 1)
we need to show that
p p p
x+ y+ x y ≥ k.
Half Convex Function Method 65
Since p p
p p p
x+ y+ x+ x + y,
p
xy ≥ y≥
it suffices to show that
x + y ≥ k2 .
Indeed, we have
x n
x+y≥ +y= = k2 .
n−1 n−1
In accordance with Note 4, the equality holds for a1 = a2 = · · · = an = 1, and
also for s
n
a1 = 0, a2 = · · · = an =
n−1
(or any cyclic permutation).
where
a+b+c+d+e
s= =1
5
and
1
f (u) = p , u ∈ [0, 5].
2 u−7
For u ∈ [0, 1], we have
p
7−6 u
f (u) = p
00
p > 0.
2u u(7 − 2 u)3
Therefore, f is convex on [0, s]. According to the LHCF-Theorem and Note 1, it
suffices to show that h(x, y) ≥ 0 for x, y ≥ 0 so that x + 4 y = 5. Since
f (u) − f (1) −2
g(u) = = p p
u−1 5(7 − 2 u)(1 + u)
and
p p
g(x) − g( y) 2(5 − 2 x − 2 y)
h(x, y) = = p p p p p p ,
x−y ( x + y)(1 + x)(1 + y)(7 − 2 x)(7 − 2 y)
66 Vasile Cîrtoaje
where
a1 + a2 + · · · + an
s= =1
n
Half Convex Function Method 67
and
1
f (u) = p , u ∈ [0, k2 ).
k− u
From p
3 u−k
f (u) = p
00
p ,
4u u(k − u)3
it follows that f is convex on [s, k2 ). According to the RHCF-Theorem and Note 1,
it suffices to show that h(x, y) ≥ 0 for all x, y ∈ [0, k2 ) so that x + (n − 1) y = n.
Since
f (u) − f (1) 1
g(u) = = p p
u−1 (k − 1)(k − u)(1 + u)
and
p p
g(x) − g( y) x + y +1−k
h(x, y) = = p p p p p p ,
x−y (k − 1)( x + y)(1 + x)(1 + y)(k − x)(k − y)
P 1.40. If a, b, c are nonnegative real numbers, no two of which are zero, then
v v v
t 48a t 48b t 48c
1+ + 1+ + 1+ ≥ 15.
b+c c+a a+b
and v
t 1 + 47u
f (u) = , u ∈ [0, 1).
1−u
From
48(47u − 11)
f 00 (u) = p ,
(1 − u)5 (1 + 47u)3
it follows that f is convex on [s, 1). By the RHCF-Theorem, it suffices to show that
1
f (x) + 2 f ( y) ≥ 3 f
3
Setting
v
t 49 − 47x
t= , 1 < t ≤ 7,
1+ x
the inequality turns into
v
t 1175 − 23t 2
≥ 15 − 2t.
t2 − 1
(5 − t)2 (2 + t)(7 − t) ≥ 0.
it follows that f is convex on [s, 3]. By the RHCF-Theorem, it suffices to prove the
original homogeneous inequality for b = c = 0 and b = c = 1. For the nontrivial
case b = c = 1, we need to show that
v v
t 3a2 t 3
+ 2 ≤ 1.
7a + 20
2 5a + 10a + 12
2
where
a+b+c
s= =1
3
and v
t 3u2 u
f (u) = =p , u ∈ [0, 3].
u + 2(3 − u)
2 2
u2 − 4u + 6
From
2(2u2 − 11u + 12) 2(−11u + 12)
f 00 (u) = ≥ 2 ,
(u − 4u + 6)
2 5/2 (u − 4u + 6)5/2
it follows that f is convex on [0, s]. By the LHCF-Theorem, it suffices to prove the
original homogeneous inequality for b = c = 0 and b = c = 1. For the nontrivial
case b = c = 1, the inequality has the form
a 2
p +p ≥ 1.
a2 + 8 2a2 + 4a + 3
For the nontrivial case 3a2 + 8a − 2 > 0, by squaring both sides we get
ln 3
k ≥ k0 , k0 = − 1 ≈ 0.585,
ln 2
then k k k
2a 2b 2c
+ + ≥ 3.
b+c c+a a+b
(Vasile C., 2005)
Solution. For k = 1, the inequality is just the well known Nesbitt’s inequality
2a 2b 2c
+ + ≥ 3.
b+c c+a a+b
Half Convex Function Method 71
For k ≥ 1, the inequality follows from Nesbitt’s inequality and Jensens’s inequality
applied to the convex function f (u) = uk :
2a 2b 2c
k
+ +
k k k
2a 2b 2c
b+c c+a a+b
+ + ≥3 ≥ 3.
b+c c+a a+b 3
2
g(a) = (k − 1) ln a − (k + 1) ln .
a+1
From
2ka + k − 1
g 0 (a) = ,
a(a + 1)
it follows that g 0 (a0 ) = 0 for a0 = (1 − k)/(2k) < 1, g 0 (a) < 0 for a ∈ (0, a0 ) and
g 0 (a) > 0 for a ∈ (a0 , 1]. Consequently, g is strictly decreasing on (0, a0 ] and strictly
increasing on (a0 , 1]. Since g(0+ ) = ∞ and g(1) = 0, there exists a1 ∈ (0, a0 ) so
72 Vasile Cîrtoaje
that g(a1 ) = 0, g(a) > 0 for a ∈ (0, a1 ) and g(a) < 0 for a ∈ (a1 , 1); therefore, h(a)
is strictly increasing on [0, a1 ] and strictly decreasing on [a1 , 1]. As a result,
Since h(0) = 2k+1 ≥ 3 and h(1) = 3, we get h(a) ≥ 3. The proof is completed. The
equality holds for a = b = c. If k = k0 , then the equality holds also for a = 0 and
b = c (or any cyclic permutation).
Remark. For k = 2/3, we can give the following solution (based on the AM-GM
inequality):
X 2a 2/3 X 2a
=
b+c
p
2a · (b + c) · (b + c)
3
X 6a
≥ = 3.
2a + (b + c) + (b + c)
p
P 1.44. If a, b, c ∈ [1, 7 + 4 3], then
v v v
t 2a t 2b t 2c
+ + ≥ 3.
b+c c+a a+b
Solution. Denoting
a+b+c p
s= , 1 ≤ s ≤ 7 + 4 3,
3
we need to show that
f (a) + f (b) + f (c) ≥ 3 f (s),
where v
t 2u
f (u) = , 1 ≤ u < 3s.
3s − u
For u ≥ s, we have
3/2
3s − u 4u − 3s
f (u) = 3s
00
> 0.
2u (3s − u)4
Therefore, f (u) is convex for u ≥ s. By the RHCF-Theorem, it suffices to prove the
original inequality for b = c; that is,
s v
a t 2b
+2 ≥ 3.
b a+b
Half Convex Function Method 73
v
tb p
Putting t = , the condition a, b ∈ [1, 7 + 4 3] involves
a
p p
2− 3 ≤ t ≤ 2 + 3.
ln 2
0 < k ≤ k0 , k0 = ≈ 1.71,
ln 3 − ln 2
then
a k (b + c) + b k (c + a) + c k (a + b) ≤ 6.
Solution. For 0 < k ≤ 1, the inequality follows from Jensens’s inequality applied
to the convex function f (u) = −uk :
k
(b + c)a + (c + a)b + (a + b)c
(b + c)a + (c + a)b + (a + b)c ≤ 2(a + b + c)
k k k
2(a + b + c)
ab + bc + ca a + b + c 2k
k
=6 ≤6 = 6.
3 3
Consider now that
1 < k ≤ k0 ,
and write the inequality as
where
a+b+c
s= =1
3
74 Vasile Cîrtoaje
and
f (u) = uk (u − 3), u ∈ [0, 3].
For u ≥ 1, we have
a+b+c
k+1
a k (b + c) + b k (c + a) + c k (a + b) ≤ 6
3
for b = c = 1 and a ∈ [0, 1]; that is, to show that g(a) ≥ 0 for a ≥ 0, where
a+2
k+1
g(a) = 3 − a k − a − 1.
3
We have
a+2 k + 1 a + 2 k−1 k − 1
k
1 00
g (a) = (k + 1)
0
− ka k−1
− 1, g (a) = − 2−k .
3 k 3 3 a
Since g 00 is strictly increasing, g 00 (0+ ) = −∞ and g 00 (1) = 2k(2 − k)/3 > 0, there
exists a1 ∈ (0, 1) so that g 00 (a1 ) = 0, g 00 (a) < 0 for a ∈ (0, a1 ), g 00 (a) > 0 for
a ∈ (a1 , 1]. Therefore, g 0 is strictly decreasing on [0, a1 ] and strictly increasing on
[a1 , 1]. Since
k+1 k−1
g 0 (0) = (k + 1)(2/3)k − 1 ≥ (k + 1)(2/3)k0 − 1 = −1= > 0,
2 2
g 0 (1) = 0,
there exists a2 ∈ (0, a1 ) so that g 0 (a2 ) = 0, g 0 (a) > 0 for a ∈ [0, a2 ), g 0 (a) < 0
for a ∈ (a2 , 1]. Thus, g is strictly increasing on [0, a2 ] and strictly decreasing on
[a2 , 1]; consequently,
g(a) ≥ min{g(0), g(1)}.
From
we get g(a) ≥ 0. This completes the proof. The equality holds for a = b = c = 1.
If k = k0 , then the equality holds also for a = 0 and b = c = 3/2 (or any cyclic
permutation).
Remark 1. Using the Cauchy-Schwarz inequality and the inequality in P 1.45, we
get
X a (a + b + c)2 9 3
≥ =P ≥ .
b +c a(b + c ) a (b + c) 2
P
k k k k k
Half Convex Function Method 75
ln 2
0 < k ≤ k0 , k0 = ≈ 1.71,
ln 3 − ln 2
then
a b c 3
+ + ≥ ,
bk + c k c k + ak ak + bk 2
with equality for a = b = c = 1. If k = k0 , then the equality holds also for a = 0 and
b = c = 3/2 (or any cyclic permutation).
Remark 2. Also, the following statement holds:
• Let a, b, c be nonnegative real numbers so that a + b + c = 3. If
ln 9 − ln 8
k ≥ k1 , k1 = ≈ 0.2905,
ln 3 − ln 2
then
ak bk ck 3
+ + ≥ ,
b+c c+a a+b 2
with equality for a = b = c = 1. If k = k1 , then the equality holds also for a = 0 and
b = c = 3/2 (or any cyclic permutation).
For k ≥ 2, the inequality can be deduced from the Cauchy-Schwarz inequality and
Bernoulli’s inequality, as follows:
P k/2 2 P k/2 2
X ak a a
≥ P = ,
b+c (b + c) 6
X X k
a k/2
≥ 1 + (a − 1) = 3.
2
76 Vasile Cîrtoaje
a+b+c
f (a) + f (b) + f (c) ≥ 3 f (s), s= = 1,
3
where v
p t3 − u
f (u) = u − 13 , u ∈ [0, 3].
2
For u ∈ [1, 3), we have
13 3 − u −3/2 13
4 f (u) = −u
00 −3/2
+ ≥ −1 + > 0.
4 2 4
x2 + 1
h(x) = m · − 1.
x2 + 2
Half Convex Function Method 77
and g(1) = 0, it follows that g(x) ≥ 0 for x ∈ [0, 1]. This completes the proof. The
equality holds for a = b = c = 1.
Remark. Similarly, we can prove the following generalizations:
• Let a, b, c be nonnegative real numbers so that a + b + c = 3. If k ≥ k0 , where
p
6−2 p p
k0 = p p = (2 + 2)(2 + 3) ≈ 12.74 ,
6− 2−1
then v v
p p p ta + b tb+c s
c+a
a+ b+ c−3≥ k + + −3 ,
2 2 2
with equality for a = b = c = 1. If k = k0 , then the equality holds also for a = 0 and
b = c = 3/2 (or any cyclic permutation).
a+b k b+c k c + a k
a + b +c +3≥2
k k k
+2 +2 .
2 2 2
78 Vasile Cîrtoaje
f 00 (u) 1 3 − u k−2 1
=u −
k−2
≥ 1 − > 0.
k(k − 1) 2 2 2
Therefore, f is convex on [s, 3]. By the RHCF-Theorem, it suffices to consider only
the case a ≤ b = c. Write the original inequality in the homogeneous form
a+2 a+1
k k
a +3
k
≥4
3 2
for a ∈ [0, 1]. Substituting
a k = t, t ∈ [0, 1],
We have
k−1 k−1
t 1/k + 2 t 1/k + 1
g 0 (t) = 1 + t 1/k−1 − 2t 1/k−1 ,
3 2
k−2 k−2
t 1/k + 1 t 1/k + 2
kt 2−1/k 00 2
g (t) = − .
k−1 2 3 3
Setting
1
2
k−2
m= , 0 < m < 1,
3
we see that g 00 (t) has the same sign as h(t), where
t +1 t 1/k + 2
1/k
h(t) = 6 −m = (3 − 2m)t 1/k + 3 − 4m
2 3
Half Convex Function Method 79
is strictly increasing. There are two cases to consider: 0 < m ≤ 3/4 and 3/4 < m <
1.
Case 1: 0 < m ≤ 3/4. Since h(0) = 3 − 4m ≥ 0, we have h(t) > 0 for t ∈ (0, 1],
hence g 0 is strictly increasing on (0, 1]. From g 0 (1) = 0, it follows that g 0 (t) < 0 for
t ∈ (0, 1), hence g is strictly decreasing on [0, 1]. Since g(1) = 0, we get g(t) > 0
for t ∈ [0, 1).
Case 2: 3/4 < m < 1. From m > 3/4, we get
22k−3 > 3k−1 .
Since h(0) = 3 − 4m < 0 and h(1) = 3(1 − m) > 0, there is t 1 ∈ (0, 1) so that
h(t 1 ) = 0, h(t) < 0 for t ∈ [0, t 1 ) and h(t) > 0 for t ∈ (t 1 , 1]. Thus, g 0 (t) is strictly
decreasing on (0, t1] and strictly increasing on [t 1 , 1]. Since g 0 (0+ ) = +∞ and
g 0 (1) = 0, there exists t 2 ∈ (0, t 1 ) so that g 0 (t 2 ) = 0, g 0 (t) > 0 for t ∈ (0, t 2 ) and
g 0 (t) < 0 for t ∈ (t 2 , 1). Therefore, g(t) is strictly increasing on [0, t 2 ] and strictly
decreasing on [t 2 , 1]. Since
22k−2 − 3k−1
g(0) = >0
2k 3k−1
and g(1) = 0, we have g(t) ≥ 0 for t ∈ [0, 1].
The equality holds for a = b = c = 1.
Remark 1. The inequality in P 1.47 is Popoviciu’s inequality
a+b+c a+b b+c c + a
f (a) + f (b) + f (c) + 3 f ≥ 2f + 2f + 2f
3 2 2 2
applied to the convex function f (x) = x k defined on [0, ∞).
Remark 2. In the same manner, we can prove the following refinements (Vasile C.,
2008):
• Let a, b, c be nonnegative real numbers so that a + b + c = 3. If k > 2 and
m ≤ m0 , where
2k (3k−1 − 2k−1 )
m0 = k−1 > 2,
6 + 3k−1 − 22k−1
then
a+b k b + c k c + a k
a + b +c −3≥ m
k k k
+ + −3 ,
2 2 2
with equality for a = b = c = 1. If m = m0 , then the equality holds also for a = 0
and b = c = 3/2 (or any cyclic permutation).
then
n − a1 k n − a2 k n − a k
n
a1k + a2k + ··· + ank −n≥m + + ··· + −n ,
n−1 n−1 n−1
with equality for a1 = a2 = · · · = an = 1. If m = m1 , then the equality holds also for
n
a1 = 0 and a2 = a3 = · · · = an = (or any cyclic permutation).
n−1
where p p p
k = ( n − 1)( n + n − 1).
k k
4 f 00 (u) = u−3/2 − p (n − u)−3/2 ≥ 1 − p (n − 1)−3/2
n−1 n−1
k k
=1− ≥1− > 0.
(n − 1)2 2(n − 1)
Half Convex Function Method 81
We have v v
t nx 2 t (n − 1)x 2
g (x) = 1 + (k − 1)
0
−k ,
x2 + n − 1 x2 + n − 2
p 3/2
k(n − 2) n − 1 x 2
+ n − 2
g 00 (x) = m· 2 −1 ,
(x 2 + n − 2)3/2 x +n−1
where v
3 (k − 1) n(n − 1)
t 2
m= .
k2 (n − 2)2
Clearly, g 00 (x) has the same sign as h(x), where
m(x 2 + n − 2) 1
h(x) = −1= m 1− 2 − 1.
x2 + n − 1 x +n−1
We have
m(n − 1)
h(1) = − 1, lim h(x) = m − 1.
n x→∞
We will show that h(1) < 0 and lim x→∞ h(x) > 0; that is, to show that
n
1<m< .
n−1
The inequality m > 1 is equivalent to
1 n−2
1− >p ,
k n(n − 1)
n
The inequality m < is equivalent to
n−1
1 n(n − 2)
1− < ,
k (n − 1)2
which is also true because
1 1 2n − 3 n(n − 2)
1− <1− = ≤ .
k 2(n − 1) 2(n − 1) (n − 1)2
Since h is strictly increasing on [1, ∞), h(1) < 0 and lim x→∞ h(x) > 0, there
is x 1 ∈ (1, ∞) so that h(x 1 ) = 0, h(x) < 0 for x ∈ [1, x 1 ) and h(x) > 0 for
x ∈ (x 1 , ∞). Therefore, g 0 is strictly decreasing on [1, x 1 ] and strictly increasing
on [x 1 , ∞). Since g 0 (1) = 0 and lim x→∞ g 0 (x) = 0, it follows that g 0 (x) < 0 for
x ∈ (1, ∞). Thus, g(x) is strictly decreasing on [1, ∞), hence g(x) ≤ g(1) = 0.
The equality holds for a1 = a2 = · · · = an = 1, and also for
a1 = n, a2 = a3 = · · · = an = 0
Since
1 2
≥ , u ∈ [1, 3/2),
3 − 2u 3 − u
it follows that f is convex on [s,3/2). By the RHCF-Theorem and Note 1, it suffices
to show that h(x, y) ≥ 0 for x, y ∈ [0, 3/2) so that x + 2 y = 3. We have
f (u) − f (1) 2 2k
g(u) = = −
u−1 3 − 2u 3 − u
and
g(x) − g( y) 2 k
h(x, y) = = −
x−y (3 − 2x)(3 − 2 y) (3 − x)(3 − y)
2 k
= −
(2 y − x)x 2 y(x + y)
kx 2 − 2(k − 2)x y + 4 y 2
=
2x y(x + y)(2 y − x)
p
[( 3 + 1)x − 2 y]2
= ≥ 0.
2x y(x + y)(2 y − x)
Solution. Since each term of the left hand side of the inequality decreases by in-
creasing any number ai , it suffices to consider the case
a1 + a2 + a3 + a4 + a5 = 5,
where
a1 + a2 + a3 + a4 + a5
s= =1
5
and
1
f (u) = , u ∈ [0, 5].
ku2 − u + 5
For u ≥ 1, we have
2[3ku(ku − 1) − 5k + 1]
f 00 (u) =
(ku2 − u + 5)3
2[3k(k − 1) − 5k + 1]
≥
(ku2 − u + 5)3
2[k(3k − 8) + 1]
= > 0;
(ku2 − u + 5)3
therefore, f is convex on [s, 5]. By the RHCF-Theorem, it suffices to show that
1 4 5
+ ≥
kx 2 − x +5 ky − y +5 k +4
2
for
0 ≤ x ≤ 1 ≤ y, x + 4 y = 5.
Write this inequality as follows:
1 1 1 1
− +4 − ≥ 0,
kx 2 − x + 5 k + 4 k y2 − y + 5 k + 4
(x − 1)(1 − k − kx) 4( y − 1)(1 − k − k y)
+ ≥ 0.
kx 2 − x + 5 k y2 − y + 5
Since
4( y − 1) = 1 − x,
the inequality is equivalent to
1 − k − kx 1− k − ky
(x − 1) − ≥ 0,
kx 2 − x + 5 k y 2 − y + 5
a1 + a2 + a3 + a4 + a5 = 5,
where
a1 + a2 + a3 + a4 + a5
s= = 1,
5
and
1
f (u) = , u ∈ [0, 5].
ku2 −u+5
For u ∈ [0, 1], we have
hence
2[3ku(ku − 1) − 5k + 1]
f 00 (u) =
(ku2 − u + 5)3
2[3k(k − 1) − 5k + 1]
≥
(ku2 − u + 5)3
2[(1 − 8k) + 3k2 ]
= > 0;
(ku2 − u + 5)3
therefore, f is convex on [0, s]. By the LHCF-Theorem, it suffices to show that
1 4 5
+ ≥
kx 2 − x +5 ky − y +5 k +4
2
for
x ≥ 1 ≥ y ≥ 0, x + 4 y = 5.
Write this inequality as follows:
1 1 1 1
− +4 − ≥ 0,
kx 2 − x + 5 k + 4 k y2 − y + 5 k + 4
Since
5k02 − 11k0 + 1 = 0,
we get
a1 = n, a2 = · · · = an = 0
Since s ≤ 1, it suffices to prove the inequality for s = 1; that is, to show that
a1 a2 an n
+ + · · · + ≥
ka12 − a1 + n ka22 − a2 + n kan2 − an + n k + n − 1
for
a1 + a2 + · · · + an = n.
Write the desired inequality as
where
a1 + a2 + · · · + an
s= =1
n
and
u
f (u) = , u ∈ [0, n].
u2 − u + n
We have
n − ku2 f1 (u)
f 0 (u) = , f 00 (u) = ,
(ku2 − u + n)2 (u2 − u + n)3
where
f1 (u) = k2 u3 − 3knu + n.
Half Convex Function Method 89
x (n − 1) y n
+ ≥
kx 2 − x + n ky − y + n k + n−1
2
n − kx n− ky
(x − 1) − ≥ 0,
kx 2 − x + n k y 2 − y + n
(x − 1)2 h(x, y)
≥ 0,
(kx 2 − x + n)(k y 2 − y + n)
where
h(x, y) = k2 x y − kn(x + y) + n − nk.
We need to show that h(x, y) ≥ 0. Indeed,
1
The equality holds for a1 = a2 = · · · = an = 1. If k = , then the equality holds
n+1
also for
a1 = n, a2 = a3 = · · · = an = 0
(or any cyclic permutation).
7
P 1.53. If a1 , a2 , a3 , a4 , a5 ≤ so that a1 + a2 + a3 + a4 + a5 = 5, then
2
a1 a2 a3 a4 a5
+ + + + ≤ 1.
a12 − a1 + 5 a22 − a2 + 5 a32 − a3 + 5 a42 − a4 + 5 a52 − a5 + 5
where
a1 + a2 + a3 + a4 + a5
s= =1
5
and
−u 7
f (u) = , u≤ .
u2 −u+5 2
7
For u ∈ 1, , we have
2
−u3 + 15u − 5
f 00 (u) =
(u2 − u + 5)3
(2u + 9)(u − 1)(7 − 2u) + 43 − 7u
= > 0.
4(u2 − u + 5)3
7
Thus, f is convex on s, . By the RHCF-Theorem, it suffices to show that
2
x 4y
+ ≤1
x2 − x + 5 y2 − y + 5
7
for all nonnegative x, y ≤ which satisfy x + 4 y = 5. Write this inequality as
2
follows:
1 1
x y
− +4 − ] ≤ 0,
x2 − x + 5 5 y2 − y + 5 5
5− x 5− y
(x − 1) − ≤ 0,
x2 − x + 5 y2 − y + 5
(x − 1)2 [5(x + y) − x y]
≥ 0,
(x 2 − x + 5)( y 2 − y + 5)
(x − 1)2 [(x + 4 y)(x + y) − x y]
≥ 0,
(x 2 − x + 5)( y 2 − y + 5)
(x − 1)2 (x + 2 y)2
≥ 0.
(x 2 − x + 5)( y 2 − y + 5)
The equality holds for a1 = a2 = a3 = a4 = a5 = 1, and also for
5
a1 = −5, a2 = a3 = a4 = a5 =
2
(or any cyclic permutation).
Remark. In the same manner, we can prove the following generalization:
Half Convex Function Method 91
p
• Let a1 , a2 , . . . , an ≤ 3 so that a1 + a2 + · · · + an ≤ n. If
p
n2 + 2n − 2 − 2 (n − 1)(2n2 − 1)
k= ,
n
then
a1 a an n
+ 2 2 + ··· + 2 ≤ ,
ka12 − a1 + n ka2 − a2 + n kan − an + n k − 1 + n
with equality for a1 = a2 = · · · = an = 1, and also for
n(k − n + 2) n(k + n − 2)
a1 = , a2 = · · · = a n =
2k 2k(n − 1)
(or any cyclic permutation).
then
a12 a22 an2 n
+ + ··· + ≥ .
ka12 + a2 + · · · + an a1 + ka22 + · · · + an a1 + a2 + · · · + kan2 k+n−1
x 12 x n2 n
+ ··· + ≥ .
kx 12 + (x 2 + · · · + x n )/s (x 1 + · · · + x n−1 )/s + kx n2 k+n−1
Since s ≥ 1, it suffices to prove the inequality for s = 1; that is, to show that
for
a1 + a2 + · · · + an = n.
92 Vasile Cîrtoaje
where
a1 + a2 + · · · + an
s= =1
n
and
u2
f (u) = , u ∈ [0, n].
u2 − u + n
We have
u(2n − u) 2 f1 (u)
f 0 (u) = , f 00 (u) = ,
(ku2 − u + n)2 (u2 − u + n)3
where
f1 (u) = ku3 − 3knu2 + n2 .
For u ∈ [0, 1] and n ≥ 3, we have
x2 (n − 1) y 2 n
+ ≥
kx − x + n k y − y + n k + n − 1
2 2
(x − 1)(nx − x + n) 4( y − 1)(n y − y + n)
+ ≥ 0,
kx 2 − x + 5 k y2 − y + 5
nx − x + n ny − y + n
(x − 1) − ≥ 0,
kx 2 − x + n k y 2 − y + n
(x − 1)2 h(x, y)
≥ 0,
(kx 2 − x + n)(k y 2 − y + n)
where
h(x, y) = n2 − kn(x + y) − k(n − 1)x y.
Half Convex Function Method 93
Since
1
0 < k ≤ k0 , k0 = 1
,
1 + 4(n−1)2
we have
x 12 x n2 n
+ ··· + ≤ .
kx 12 + (x 2 + · · · + x n )/s (x 1 + · · · + x n−1 )/s + kx n2 k+n−1
Since s ≤ 1, it suffices to prove the inequality for s = 1; that is, to show that
for
a1 + a2 + · · · + an = n.
94 Vasile Cîrtoaje
a1 + a2 + · · · + an
f (a1 ) + f (a2 ) + · · · + f (an ) ≥ n f (s), s= = 1,
n
where
−u2
f (u) = 2 , u ∈ [0, n].
u −u+n
We have
u(u − 2n) 2 f1 (u)
f 0 (u) = , f 00 (u) = ,
(ku2 − u + n)2 (u2 − u + n)3
where
f1 (u) = −ku3 + 3knu2 − n2 .
For u ∈ [1, n], we have
Since f 00 (u) ≥ 0 for u ∈ [1, n], it follows that f is convex on [s, n]. By the RHCF-
Theorem, it suffices to show that
x2 (n − 1) y 2 n
+ ≤
kx 2 − x + n k y 2 − y + n k + n − 1
Since k ≥ n − 1, we have
1
P 1.56. Let a1 , a2 , . . . , an ∈ [0, n] so that a1 + a2 + · · · + an ≥ n. If 0 < k ≤ , then
n
a1 − 1 a2 − 1 an − 1
+ + · · · + ≥ 0.
ka12 + a2 + · · · + an a1 + ka22 + · · · + an a1 + a2 + · · · + kan2
Solution. Let
a1 + a2 + · · · + an
s= , s ≥ 1.
n
Case 1: s > 1 Without loss of generality, assume that
a1 ≥ b1 ≥ 1, . . . , a j ≥ b j ≥ 1, b j+1 = a j+1 , . . . , bn = an .
where
u−1
f (u) = , u ∈ [0, n],
ku2 − u + ns
f1 (u)
f 0 (u) , f1 (u) = k(−u2 + 2u) + ns − 1.
(ku2 − u + ns)2
For u ∈ [1, n), we have
Since
f (b1 ) + f (b2 ) + · · · + f (bn ) ≤ f (a1 ) + f (a2 ) + · · · + f (an ),
it suffices to show that f (b1 ) + f (b2 ) + · · · + f (bn ) ≥ 0 for b1 + b2 + · · · + bn = n.
This inequality is proved at Case 2.
Case 2: s = 1. Write the inequality as
a1 + a2 + · · · + an
f (a1 ) + f (a2 ) + · · · + f (an ) ≥ n f (s), s= = 1,
n
96 Vasile Cîrtoaje
where
u−1
f (u) = , u ∈ [0, n],
ku2 − u + n
2g(u)
f 00 (u) = , g(u) = k2 u3 − 3k2 u2 − 3k(n − 1)u + kn + n − 1.
(ku2− u + n)3
We will show that f 00 (u) ≥ 0 for u ∈ [0, 1]. From
x −1 (n − 1)( y − 1)
+ ≥0
kx 2 − x + n k y2 − y + n
1
The proof is completed. The equality holds for a1 = a2 = · · · = an = 1. If k = ,
n
then the equality holds also for
a1 = n, a2 = a3 = · · · = an = 0.
Half Convex Function Method 97
a = ex , b = ey, c = ez ,
where x = eu ≥ 1. Indeed,
and
b = c := t, a = 1/t 2 , t > 0;
that is, p
t4 − t2 + 1 p 1
2
+ 2 t 2 − t + 1 ≥ 2 + 2t,
t t
2
t −1 2(1 − t)
p +p ≥ 0.
t4 − t2 + 1 + 1 t2 − t + 1 + t
Since
t2 − 1 t2 − 1
p ≥ ,
t4 − t2 + 1 t2 + 1
it suffices to show that
t2 − 1 2(1 − t)
+ p ≥ 0,
t2 + 1 t2 − t + 1 + t
98 Vasile Cîrtoaje
which is equivalent to
t +1 2
(t − 1) 2 −p ≥ 0,
t +1 t2 − t + 1 + t
p
(t − 1) (t + 1) t 2 − t + 1 − t 2 + t − 2 ≥ 0,
(t − 1)2 (3t 2 − 2t + 3)
p ≥ 0.
(t + 1) t 2 − t + 1 + t 2 − t + 2
The equality holds for a = b = c = 1.
1
P 1.58. If a, b, c, d ≥ p so that abcd = 1, then
1+ 6
1 1 1 1 4
+ + + ≤ .
a+2 b+2 c+2 d +2 3
a = ex , b = ey, c = ez , d = ew,
(t − 1)2 (5t 2 + 2t − 1) ≥ 0.
Half Convex Function Method 99
a2 + b2 + c 2 − 3 ≥ 2(ab + bc + ca − a − b − c).
a = ex , b = ey, c = ez ,
4t 5 − 3t 4 − 4t 3 + 2t 2 + 1 ≥ 0,
which is equivalent to
(t − 1)2 (4t 3 + 5t 2 + 2t + 1) ≥ 0.
a2 + b2 + c 2 − 3 ≥ 18(a + b + c − ab − bc − ca).
100 Vasile Cîrtoaje
a = ex , b = ey, c = ez ,
1 (t 2 − 1)2 (2t 2 + 1)
a2 + b2 + c 2 − 3 = + 2t 2
− 3 =
t4 t4
and
For u ≤ 0, we have p
f 00 (u) = 4e2u + 6 3(e−u − eu ) > 0,
hence f is convex on I≤s . By the LHCF-Theorem and Note 2, it suffices to show that
H(x, y) ≥ 0 for x, y ∈ R so that x + (n − 1) y = 0, where
f 0 (x) − f 0 ( y)
H(x, y) = .
x−y
From p
f 0 (u) = 2e2u − 6 3 (eu + e−u ),
we get
2(e x − e y ) x p p
H(x, y) = e + e y − 3 3 + 3 3 e−x− y .
x−y
Since (e x − e y )/(x − y) > 0, we need to prove that
p p
e x + e y + 3 3 e−x− y ≥ 3 3.
x1 + x2 + · · · + x n
f (x 1 ) + f (x 2 ) + · · · + f (x n ) ≥ n f (s), s= = 0,
n
where
f (u) = (n − 1)e2u − (2n + 2)eu , u ∈ I = R.
For u ≥ 0, we have
Æ pn
= n (n − 1)e x+(n−1) y = n n − 1.
n
t 2n−2 n−1 n
+ ≥ .
t 2n−2 + pt n−1 + q 1 + pt + qt 2 1+p+q
(t n−1 + n − 1)2 n
≥ ,
(t 2n−2 + pt n−1 + q) + (n − 1)(1 + pt + qt ) 1 + p + q
2
which is equivalent to
pB + qC ≥ A,
where
A = (n − 1)(t n−1 − 1)2 ≥ 0,
A
B = (t n−1 − 1)2 + nE = + nE, E = t n−1 + n − 2 − (n − 1)t,
n−1
A
C = (t n−1 − 1)2 + nF = + nF, F = 2t n−1 + n − 3 − (n − 1)t 2 .
n−1
By the AM-GM inequality applied to n − 1 positive numbers, we have E ≥ 0 and
F ≥ 0 for n ≥ 3. Since A ≥ 0 and p + q ≥ n − 1, we have
(p + q)A
pB + qC − A ≥ pB + qC − = n(pE + qF ) ≥ 0.
n−1
104 Vasile Cîrtoaje
1 1 1 n
+ + ··· + ≥ ,
(1 + ka1 )2 (1 + ka2 )2 (1 + kan )2 (1 + k)2
In addition, for n = 4 and k = 1, we get the known inequality (Vasile C., 1999):
1 1 1 1
+ + + ≥ 1,
(1 + a)2 (1 + b)2 (1 + c)2 (1 + d)2
1 1 1
+ + ··· + ≥ 1,
1 + pa1 + qa1 1 + pa2 + qa2
2 2
1 + pan + qan2
1 1 1
+ + ··· + ≥ 1,
1 + (n − 1)a1 1 + (n − 1)a2 1 + (n − 1)an
1 1 1
p + p + ··· + p ≥ 1,
[1 + ( n − 1)a1 ] 2 [1 + ( n − 1)a1 ] 2 [1 + ( n − 1)a1 ]2
1 1 1 1
+ + ··· + ≥ .
2 + (n − 1)(a1 + a1 ) 2 + (n − 1)(a2 + a2 )
2 2
2 + (n − 1)(an + an ) 2
2
1 1 1 1
+ + + ≥ 1.
1 + pa + qa3 1 + pb + qb3 1 + pc + qc 3 1 + pd + qd 3
a = ex , b = ey, c = ez , d = ew,
th(t)
f 00 (u) = ,
(1 + pt + qt 3 )3
where
h(t) = 9q2 t 5 + 2pqt 3 − 9qt 2 + p2 t − p, t = eu ,
we need to show that h(t) ≥ 0 for t ≥ 1. Indeed, we have
where
b = c = d = t, a = 1/t 3 , t > 0;
that is,
t9 3
+ ≥ 1,
t 9 + pt 6 + q 1 + pt + qt 3
3 pt 6 + q
≥ ,
1 + pt + qt 3 t 9 + pt 6 + q
(3 − pq)t 9 − p2 t 7 + 2pt 6 − q2 t 3 − pqt + 2q ≥ 0,
106 Vasile Cîrtoaje
B = t 9 − 3t 3 + 2 = (t 3 − 1)2 (t 3 + 2) ≥ 0,
C = t 9 − 2t 6 + 3t − 2.
Since A ≥ 0 and B ≥ 0, it suffices to consider the case C ≥ 0. Since
p
Ap2 + Bq2 ≥ 2 ABpq,
t 3 − 3t + 2 = (t − 1)2 (t + 2) ≥ 0,
we get 3t − 2 ≤ t 3 . Therefore
C ≤ t 9 − 2t 6 + t 3 = t 3 (t 3 − 1)2 ,
hence
1 1 1 1
+ + + ≥ 1,
1 + a + 2a 3 1 + b + 2b 3 1 + c + 2c 3 1 + d + 2d 3
1 1 1 1
+ + + ≥ 1.
1 + 2a + a 3 1 + 2b + b 3 1 + 2c + c 3 1 + 2d + d 3
1 1 1 1 4
+ + + ≥ .
1 + pa + qa 3 1 + pb + qb 3 1 + pc + qc 3 1 + pd + qd 3 1+p+q
Half Convex Function Method 107
Since
ai + ai3
ai2 ≤
,
2
the best inequality with respect to q if for q = 0:
4
X 1
≥ 1, p + r = 3.
i=1
1 + pai + r ai3
where
A = t + 2t 2 + · · · + (n − 1)t n−1 ,
B = t + 4t 2 + · · · + (n − 1)2 t n−1 ,
C = t + t 2 + · · · + t n−1 .
108 Vasile Cîrtoaje
For n = 2, the inequality becomes t(t − 1) ≥ 0. Assume now that the inequality is
true for n and prove it for n + 1, n ≥ 2. So, we need to show that 2A2 ≥ B(1 + C)
involves
2(A + nt n )2 ≥ (B + n2 t n )(1 + C + t n ),
which is equivalent to
where
n−1
X
D = 4nA − B − n C = 2
bi t i , bi = 3n2 − (2n − i)2 .
i=1
Since 2A2 − B(1 + C) ≥ 0 (by the induction hypothesis), it suffices to show that
D ≥ 0. Since
b1 < b2 < · · · < bn−1 , t ≤ t 2 ≤ · · · ≤ t n−1 ,
we may apply Chebyshev’s inequality to get
1
D≥ (b1 + b2 + · · · + bn−1 )(t + t 2 + · · · + t n−1 ).
n
Thus, it suffices to show that b1 + b2 + · · · + bn−1 ≥ 0. Indeed,
n−1
X n(n − 1)(4n + 1)
b1 + b2 + · · · + bn−1 = [3n2 − (2n − i)2 ] = > 0.
i=1
6
a1 = 1/t n−1 , a2 = · · · = an = t, t ≥ 1,
Since 1 < t < · · · < t k−1 and 1 < t k < · · · < t (k−1)k , the last inequality follows from
Chebyshev’s inequality.
The equality holds for a1 = a2 = · · · = an = 1.
In addition, since
(m − k)ai + (k − 1)aim
aik ≤ , k = 2, 3, . . . , m − 1
m−1
(by the AM-GM inequality applied to m − 1 positive numbers), the best inequality
with respect to k2 , . . . , km−1 is for k2 = 0, . . . , km−1 = 0; that is,
n
X 1 n
≥ , k1 + km ≥ n − 1, 1 ≤ m ≤ n − 1.
i=1
1 + k1 a i + k m a i
m
1 + k1 + km
If k1 + km = n − 1, then
n
X 1
≥ 1, 1 ≤ m ≤ n − 1,
i=1
1 + k1 ai + km aim
therefore
n
X 1
≥ 1, k1 + kn−1 = n − 1.
i=1
1 + k1 ai + kn−1 ain−1
For k1 = 1 and k1 = n − 2, we get the following strong inequalities:
n
X 1
≥ 1,
i=1
1 + ai + (n − 2)ain−1
n
X 1
≥ 1.
i=1
1 + (n − 2)ai + ain−1
110 Vasile Cîrtoaje
k ≥ n2 − 1,
then
1 1 1 n
+p + ··· + p ≥p .
1 + ka1 1 + ka2 1 + kan 1+k
p
where
h2 (t) = k − t m−1 − t 2m−1 .
For t > 1, we have
h02 (t) = t m−2 [−m + 1 − (2m − 1)t m ] < t m−2 [−m + 1 − (2m − 1)]
= −(3m − 2)t m−2 ≤ 0,
Half Convex Function Method 111
there exists t 1 > 1 so that h2 (t 1 ) = 0, h2 (t) > 0 for t ∈ [1, t 1 ), h2 (t) < 0 for
t ∈ (t 1 , ∞). Since h2 (t), h1 (t) and h0 (t) has the same sign for t > 1, h(t) is strictly
increasing for t ∈ [1, t 1 ] and strictly decreasing for t ∈ [t 1 , ∞); this yields
m ≤ n − 1, k ≥ n1/m − 1,
then
1 1 1 n
+ + ··· + ≥ ,
(1 + ka1 ) m (1 + ka2 ) m (1 + kan ) m (1 + k)m
with equality for a1 = a2 = · · · = an = 1.
For u ≤ 0, we have
t 2n−2 n−1 n
+ ≤ ,
t 2n−2 + pt n−1 + q 1 + pt + qt 2 1+p+q
p2 A + q2 B + pqC ≤ pD + qE,
where
A = t n−1 (t n − nt + n − 1), B = t 2n − nt 2 + n − 1,
C = t 2n−1 + t 2n − nt n+1 + (n − 1)t n−1 − nt + n − 1,
D = t n−1 [(n − 1)t n + 1 − nt n−1 ], E = (n − 1)t 2n + 1 − nt 2n−2 .
Applying the AM-GM inequality to n positive numbers yields D ≥ 0 and E ≥ 0.
Since (n − 1)(p + q) ≤ 1 involves pD + qE ≥ (n − 1)(p + q)(pD + qE), it suffices to
show that
p2 A + q2 B + pqC ≤ (n − 1)(p + q)(pD + qE).
Write this inequality as
p2 A1 + q2 B1 + pqC1 ≥ 0,
where
A1 = (n − 1)D − A = nt n [(n − 2)t n−1 + 1 − (n − 1)t n−2 ],
B1 = (n − 1)E − B = nt 2 [(n − 2)t 2n−2 + 1 − (n − 1)t 2n−4 ],
C1 = (n − 1)(D + E) − C = nt[(n − 2)(t 2n−1 + t 2n−2 ) − 2(n − 1)t 2n−3 + t n + 1].
Applying the AM-GM inequality to n − 1 nonnegative numbers yields A1 ≥ 0 and
B1 ≥ 0. So, it suffices to show that C1 ≥ 0. Indeed, we have
where
A2 = (n − 2)t 2n−1 + t − (n − 1)t 2n−3 ≥ 0,
Half Convex Function Method 113
1 1 1
+ + ··· + ≤ 1,
n − 1 + a1 n − 1 + a2 n − 1 + an
1 1 1 1
+ + ··· + ≤ .
2n − 2 + a1 + a1 2n − 2 + a2 + a2
2 2
2n − 2 + an + an2 2
Remark 2. For
4n − 3 1
p= , q= ,
2(n − 1)(2n − 1) 2(n − 1)(2n − 1)
1 1 1
+ ··· + ≤ ,
(a1 + 2n − 2)(a1 + 2n − 1) (an + 2n − 2)(an + 2n − 1) 4n − 2
which is equivalent to
1 1 1 1 1
+ ··· + ≤ + + ··· + .
a1 + 2n − 2 an + 2n − 2 4n − 2 a1 + 2n − 1 an + 2n − 1
then
1 1 1 n
+p + ··· + p ≤p .
1 + ka1 1 + ka2 1 + kan 1+k
p
where
h2 (t) = t m−1 + t 2m−1 − k
is strictly increasing for t ∈ [0, 1]. There are two possible cases: h2 (0) ≥ 0 and
h2 (0) < 0.
Half Convex Function Method 115
Case 1: h2 (0) ≥ 0. This case is possible only for m = 1 and k ≤ 1, when h2 (t) =
t + 1 − k > 0 for t ∈ (0, 1]. Also, we have h1 (t) > 0 and h0 (t) > 0 for t ∈ (0, 1).
Therefore, h is strictly increasing on [0, 1], hence h(t) ≤ h(1) = 0.
Case 2: h2 (0) < 0. This case is possible for either m = 1 (n = 3) and 1 < k ≤ 5/4,
or m > 1 (n ≥ 4). Since h2 (1) = 2−k > 0, there exists t 1 ∈ (0, 1) so that h2 (t 1 ) = 0,
h2 (t) < 0 for t ∈ (0, t 1 ), and h2 (t) > 0 for t ∈ (t 1 , 1). Since h0 has the same sign as
h2 on (0, 1), it follows that h is strictly decreasing on [0, t 1 ] and strictly increasing
n
on [t 1 , 1]. Therefore, h(t) ≤ max{h(0), h(1)}. Since h(0) = n − 1 − p ≤ 0 and
1+k
h(1) = 0, we have h(t) ≤ 0 for all t ∈ (0, 1].
The equality holds for a1 = a2 = · · · = an = 1.
Remark. The following generalization holds (Vasile C., 2005):
• Let a1 , a2 , . . . , an (n ≥ 3) be positive real numbers so that a1 a2 · · · an = 1. If k
and m are positive numbers so that
1 n 1/m
m≥ , k≤ − 1,
n−1 n−1
then
1 1 1 n
+ + ··· + ≤ ,
(1 + ka1 ) m (1 + ka2 ) m (1 + kan ) m (1 + k)m
with equality for a1 = a2 = · · · = an = 1.
1 n 1/m
For n ≥ 3, m ≥ and k = − 1, we get the beautiful inequality
n−1 n−1
1 1 1
+ + ··· + ≤ n − 1.
(1 + ka1 ) m (1 + ka2 ) m (1 + kan )m
Since
h0 (b)
= (n − 1)b k − nb n−1 + 1 ≥ (n − 1)b n − nb n−1 + 1
k+1
= nb n−1 (b − 1) − (b n − 1)
= (b − 1) (b n−1 − b n−2 ) + (b n−1 − b n−3 ) + · · · + (b n−1 − 1) ≥ 0,
h is increasing on [1, ∞), hence h(b) ≥ h(1) = 0. The proof is completed. The
equality holds for a1 = a2 = · · · = an = 1.
118 Vasile Cîrtoaje
1 a1 1 a2 1 an
1− + 1− + ··· + 1 − ≤ n − 1.
n n n
Solution. Let
n
k= , k > 1,
n−1
and
m = ln k, 0 < m ≤ ln 2 < 1.
Using the substitutions ai = e x i for i = 1, 2, . . . , n, we need to show that
x1 + x2 + · · · + x n
f (x 1 ) + f (x 2 ) + · · · + f (x n ) ≥ n f (s), s= = 0,
n
where
u
f (u) = −k−e , u ∈ I = R.
From
u
f 00 (u) = meu k−e (1 − meu ),
it follows that f 00 (u) > 0 for u ≤ 0, since
1 − meu ≥ 1 − m ≥ 1 − ln 2 > 0.
a2 = · · · = an := t, a1 = t −n+1 , 0 < t ≤ 1.
it follows that there is t 1 ∈ (0, 1) so that h2 (t 1 ) = 0, h2 (t) > 0 for t ∈ [0, t 1 ) and
h2 (t) < 0 for t ∈ (t 1 , 1]. Therefore, h1 is strictly increasing on (0, t 1 ] and strictly
decreasing on [t 1 , 1]. Since h1 (0+ ) = −∞ and h1 (1) = 0, there is t 2 ∈ (0, t 1 ) so
that h1 (t 2 ) = 0, h1 (t) < 0 for t ∈ (0, t 2 ), h1 (t) > 0 for t ∈ (t 2 , 1). Thus, h is strictly
decreasing on (0, t 2 ] and strictly increasing on [t 2 , 1]. Since h(0+ ) = n − 1 and
h(1) = n − 1, we have h(t) ≤ n − 1 for all t ∈ (0, 1]. This completes the proof. The
equality holds for a1 = a2 = · · · = an = 1.
1 1 1
p + p + p ≤ 1.
1 + 1 + 3a 1 + 1 + 3b 1 + 1 + 3c
Since
Æ
2 (b2 + 3b)(3b2 + 1) ≤ (b2 + 3b) + (3b2 + 1) = 4b2 + 3b + 1,
4n
0<k≤ ,
(n − 1)2
then
1 1 1 n
+ + ··· + ≤ p .
1 + 1 + ka1 1 + 1 + ka2 1 + 1 + kan 1+ 1+k
p p p
1 1 1 1
+ + ··· + ≥ .
1+ 1 + 4n(n − 1)a1 1+ 1 + 4n(n − 1)a2 1+ 1 + 4n(n − 1)an
p p p
2
Solution. Denote
k = 4n(n − 1), k ≥ 8,
and write the inequality as follows:
4t 2 + 6kt + k2
f (u) = t
00
p −1 >0
4(t + k) t(t + k)
because
k b n−1 [1 + (n − 1)b n/2 ]2 + [1 + (n − 1)b n ]2 ≥ [(n − 1)b n + 2n(n − 1)b n−1 + 1]2 ,
which is equivalent to
3n−2
n n−2
4n(n − 1)2 b 2 2 + (n − 2)b 2 − nb 2 ≥ 0.
a6 b6 c6
+ + ≥ 1.
1 + 2a5 1 + 2b5 1 + 2c 5
a = ex , b = ey, c = ez ,
1 2b6
+ ≥ 1.
b2 (b10 + 2) 1 + 2b5
Since
1 + 2b5 ≤ 1 + b4 + b6 ,
Half Convex Function Method 123
x 3 (x 6 − x 5 − x 3 + 2x − 1) + (x − 1)2 ≥ 0,
x 3 (x − 1)2 (x 4 + x 3 + x 2 − 1) + (x − 1)2 ≥ 0,
(x − 1)2 [x 7 + x 5 + (x 6 − x 3 + 1)] ≥ 0.
The equality holds for a = b = c = 1.
a = ex , b = ey, c = ez ,
5w(25w2 + 288)
f (u) = 5w 1 −
00
, w = eu ∈ (0, 1],
(25w2 + 144)3/2
25
Setting 25w2 = 144z, we have z ∈ 0, and
144
(25w2 + 144)3 − 25w2 (25w2 + 288)2 = 1443 (z + 1)3 − 1443 z(z + 2)2
= 1443 (1 − z − z 2 ) > 0.
124 Vasile Cîrtoaje
a = t 2, b = c = 1/t, t > 0;
that is, p p
5t 3 + 24t + 10 ≥ 25t 6 + 144t 2 + 2 25 + 144t 2 .
Squaring and dividing by 4t give
Æ
60t 3 + 25t 2 − 36t + 120 ≥ (25t 4 + 144)(144t 2 + 25).
a = ex , b = ey, c = ez ,
4w(16w2 + 18)
f (u) = 4w
00
−1 , w = eu ≥ 1,
(16w2 + 9)3/2
16
Setting 16w2 = 9z, we have z ≥ and
9
a = t 2, b = c = 1/t, t > 0;
that is, p p
16t 6 + 9t 2 + 2 16 + 9t 2 ≥ 4t 3 + 3t + 8.
Squaring and dividing by 4t give
Æ
(16t 4 + 9)(9t 2 + 16) ≥ 6t 3 + 16t 2 − 9t + 12.
with equality for an equilateral triangle, for a degenerate triangle with A = 0 and
B = C = π/2 (or any cyclic permutation), and for a degenerate triangle with A = π
and B = C = 0 (or any cyclic permutation).
If ABC is an acute or right triangle, then this inequality follows by replacing A,
B and C with π − 2A, π − 2B and π − 2C in the inequality from P 1.78. Consider
now that
π
A > > B ≥ C ≥ 0.
2
The inequality is true for B ≤ π/3, because
Therefore it suffices to
which is equivalent to
Since
2π π π
C < A− B < − = ,
3 3 3
we have cos C > cos(A − B). Therefore, it suffices to show that
2 cos C − 1 ≥ 1 − 2 cos B,
which is equivalent to
cos B + cos C ≥ 1.
From B + C < π/2, we get cos B > cos(π/2 − C) = sin C, hence
p
cos B + cos C > sin C + cos C = 1 + sin 2C ≥ 1.
we get
9 5u 25 5u
f 00 (u) ≥ −4 sin 2u + cos − cos
4 2 4 2
π − 5u π−u 5π − 9u
= −4 sin 2u + sin = 8 sin cos .
2 4 4
For π/3 ≤ u ≤ π/2, we have
π 5π − 9u π
≤ ≤ ,
8 4 2
hence f 00 (u) ≥ 0. By the RHCF-Theorem, it suffices to prove the original inequality
for B = C, 0 ≤ B ≤ π/2, when it becomes
B
− sin 4B(1 − 2 cos B) + 2 sin 2B 1 − 2 sin ≥ 0,
2
B
2 sin 2B cos 2B(2 cos B − 1) + 1 − sin ≥ 0.
2
We need to show that
B
cos 2B(2 cos B − 1) + 1 − sin ≥ 0,
2
which is equivalent to g(t) ≥ 0, where
B 1
g(t) = (1 − 8t 2 + 8t 4 )(1 − 4t 2 ) + 1 − 2t, t = sin , 0≤t ≤ p .
2 2
Indeed, we have
because
1 + 3t + 2t 2 − 4t 3 − 4t 4 ≥ 1 + 3t + 2t 2 − 2t − 2t 2 = 1 + t > 0.
The equality occurs for an equilateral triangle, for a degenerate triangle with
A = 0 and and B = C = π/2 (or any cyclic permutation), and for a degenerate
triangle with A = π and B = C = 0 (or any cyclic permutation).
Remark 1. Actually, the inequality holds also for an obtuse triangle ABC. To prove
this, consider that
π
A > > B ≥ C ≥ 0.
2
The inequality is true for B ≤ π/3, because
A B C
sin 2A 1 − 2 sin ≥ 0, sin 2B 1 − 2 sin ≥ 0, sin 2C 1 − 2 sin ≥ 0.
2 2 2
Half Convex Function Method 129
with equality for an equilateral triangle, for a triangle with A = π/2 and B = C =
π/4 (or any cyclic permutation), and for a degenerate triangle with A = 0 and and
B = C = π/2 (or any cyclic permutation).
f (u), u ≤ 2
f0 (u) = .
f (2), u > 2
Since f0 (u) ≤ f (u) for u ∈ R and f0 (1) = f (1), it suffices to show that
f0 (u) =
0
0, u>2
Half Convex Function Method 131
g(x) − g( y) 4(x + y) − x y
h(x, y) = =
x−y 4(x − x + 4)( y 2 − y + 4)
2
3( y − 2)2 + 4
= > 0.
4(x 2 − x + 4)( y 2 − y + 4)
ln 2
k0 ≤ k ≤ 3, k0 = ≈ 1.71,
ln 3 − ln 2
then
a k (b + c) + b k (c + a) + c k (a + b) ≤ 2.
where
f (u) = uk (2 − u), u ∈ [0, ∞).
From
f 00 (u) = kuk−2 [2k − 2 − (k + 1)u],
2k − 2 2k − 2
it follows that f is convex on 0, and concave on , 2 . According
k+1 k+1
to LCRCF-Theorem, the sum f (a) + f (b) + f (c) is maximum when either a = 0 or
0 < a ≤ b = c.
132 Vasile Cîrtoaje
k−1 2 k−1 2
b k−1 + c k−1 = (b2 )(k−1)/2 + (c 2 )(k−1)/2 ≤ 1 + (b − 1) + 1 + (c − 1)
2 2
k−1 2
=3−k+ (b + c 2 ).
2
Thus, it suffices to show that
k−1
(3 − k)bc + bc(b2 + c 2 ) ≤ 2.
2
Since 2
b+c
bc ≤ = 1,
2
we only need to show that
k−1
3−k+ bc(b2 + c 2 ) ≤ 2,
2
which is equivalent to
bc(b2 + c 2 ) ≤ 2.
Indeed, we have
a+b+c
k+1
a (b + c) + b (c + a) + c (a + b) ≤ 2
k k k
2
for b = c = 1 and 0 < a ≤ 1; that is,
a k+1
1+ − a k − a − 1 ≥ 0.
2
a k+1
Since 1 + is increasing and a k is decreasing with respect to k, it suffices
2
consider the case k = k0 ; that is, to prove that g(a) ≥ 0, where
a k0 +1
g(a) = 1 + − a k0 − a − 1, 0 < a ≤ 1.
2
We have
k0 + 1 a k0
g (a) =
0
1+ − k0 a k0 −1 − 1,
2 2
Half Convex Function Method 133
1 00 k0 + 1 a k0 −1 k0 − 1
g (a) = 1+ − 2−k .
k0 4 2 a 0
Since g 00 is increasing on (0, 1], g 00 (0+ ) = −∞ and
1 00 k0 + 1 3 k0 −1 k0 + 1 2(2 − k0 )
g (1) = − k0 + 1 = − k0 + 1 = > 0,
k0 4 2 3 3
there exists a1 ∈ (0, 1) so that g 00 (a1 ) = 0, g 00 (a) < 0 for a ∈ (0, a1 ), g 00 (a) > 0 for
a ∈ (a1 , 1]. Therefore, g 0 is strictly decreasing on [0, a1 ] and strictly increasing on
[a1 , 1]. Since
k0 − 1 k0 + 1
g 0 (0) = > 0, g 0 (1) = (3/2)k0 − 2 = 0,
2 2
there exists a2 ∈ (0, a1 ) so that g 0 (a2 ) = 0, g 0 (a) > 0 for a ∈ [0, a2 ), g 0 (a) < 0
for a ∈ (a2 , 1). Thus, g is strictly increasing on [0, a2 ] and strictly decreasing on
[a2 , 1]. Consequently,
g(a) ≥ min{g(0), g(1)},
and from
g(0) = 0, g(1) = (3/2)k0 +1 − 3 = 0,
we get g(a) ≥ 0.
The equality holds for a = 0 and b = c (or any cyclic permutation). If k = k0 ,
then the equality holds also for a = b = c.
where
(n + 1)2
f (u) = − 4(n + 2)u2 , u ∈ (0, ∞).
u
From
2(n + 1)2
f 00 (u) = − 8(n + 2),
u3
it follows that f is strictly convex on (0, c] and strictly concave on [c, ∞), where
v
3 (n + 1)
t 2
c= .
4(n + 2)
134 Vasile Cîrtoaje
2 n−1 1
(n + 1) + ≥ 4(n + 2)[(n − 1)x 2 + an2 ) + n(n2 − 3n − 6),
x an
(n + 1)2 2n − 1 2 3(n − 2)
(n − 1)x − (2n − 1)x +
2
= (n − 1) x − + ≥ 0.
n+2 2n − 2 4(n − 1)(n + 2)
The equality holds for
1 n+1
a1 = a2 = · · · = an−1 = , an =
2 2
(or any cyclic permutation).
Since
27
g(u) = − − 4(u2 + u + 1),
u
A(x, y)
h(x, y) = , A(x, y) = 27 − 4x y(x + y + 1),
xy
we need show that A(x, y) ≥ 0. Indeed,
1
A(x, y) = 9 − 4 y(4 y − 5)( y − 2) = 9 − 40 y + 52 y 2 − 16 y 3
3
= (1 − 2 y)2 (9 − 4 y) ≥ 0.
The equality holds for a = b = c = d = e = 1, and for a = 3 and b = c = d = e =
1/2 (or any cyclic permutation).
Generalization. If a1 , a2 , ..., an are positive real numbers such that
a1 + a2 + · · · + an = n,
then
1 1 1
(n + 1)2 (2n − 1)( + + ··· + − n) ≥ 27(n − 1)2 (a13 + a23 + · · · + an3 − n),
a1 a2 an
2n − 1 n+1
a1 = , a2 = · · · = an =
3 3(n − 1)
where
f (u) = ln(u2 + 10), u ∈ [0, 12].
From
2(10 − u2 )
f 00 (u) = ,
(u2 + 10)2
136 Vasile Cîrtoaje
p p
it follows that f is convex on [0, 10] and concave on [ 10, 12]. According to
LCRCF-Theorem, the sum f (a) + f (b) + f (c) is minimum when a = b ≤ c. There-
fore, it suffices to prove that g(a) ≥ 0, where
Therefore, g 0 (a) < 0 for a ∈ [0, 1) and g 0 (a) > 0 for a ∈ (1, 4), hence g is strictly
decreasing on [0, 1] and strictly increasing on [1, 4]. Thus, we have
g(a) ≥ g(1) = 0.
(n2 − 2n + 2)n
f (a1 ) + f (a2 ) + · · · + f (an ) ≥ ln k, k= ,
(n − 1)2n−2
where
f (u) = ln(u2 + 1), u ∈ [0, n].
From
2(1 − u2 )
f 00 (u) = ,
(u2 + 1)2
Half Convex Function Method 137
it follows that f is strictly convex on [0, 1] and strictly concave on [1, n]. According
to LCRCF-Theorem, it suffices to consider the case a1 = a2 = · · · = an−1 ≤ an ; that
is, to show that g(x) ≥ 0, where
where
f (u) = ln(u2 + 2), u ∈ [0, 3].
From
2(2 − u2 )
f 00 (u) = ,
(u2 + 2)2
p p
it follows that f is strictly convex on [0, 2] and strictly concave on [ 2, 3]. Ac-
cording to LCRCF-Theorem, the sum f (a)+ f (b)+ f (c) is maximum for either a = 0
or 0 < a ≤ b = c.
Case 1: a = 0. We need to show that b + c = 3 involves
which is equivalent to
bc(bc − 4) ≤ 0.
This is true because
b+c 2 9
bc ≤ = < 4.
2 4
Case 2: 0 < a ≤ b = c. We need to show that a + 2b = 3 (0 < a ≤ 1) involves
16 · 22 − 172 = 63 > 0.
with equality for a = b = 0 and c = 3 (or any cyclic permutation). If k = 9/8, then
the equality holds also for a = 0 and b = c = 3/2 (or any cyclic permutation).
where
f (u) = ln(u2 + 1), u ∈ [0, 3].
From
2(1 − u2 )
f (u) = 2
00
,
(u + 1)2
it follows that f is strictly convex on [0, 1] and strictly concave on [1, 3]. According
to LCRCF-Theorem, it suffices to consider the cases a = 0 and 0 < a ≤ b = c.
Case 1: a = 0. We need to show that b + c = 3 involves
169
(b2 + 1)(c 2 + 1) ≤ ,
16
which is equivalent to
(4bc + 1)(4bc − 9) ≤ 0.
This is true because
4bc ≤ (b + c)2 = 9.
Case 2: 0 < a ≤ b = c. We need to show that a + 2b = 3 (0 < a ≤ 1) involves
169
(a2 + 1)(b2 + 1)2 ≤ ,
16
which is equivalent to g(a) ≤ 0, where
3−a
g(a) = ln(a2 + 1) + 2 ln(b2 + 1) − ln 169 + ln 16, b= , a ∈ (0, 1].
2
Since b0 (a) = −1/2, we have
The equality holds for a = 0 and b = c = 3/2 (or any cyclic permutation).
140 Vasile Cîrtoaje
where
f (u) = ln(2u2 + 1), u ∈ [0, 3].
From
4(1 − 2u2 )
f 00 (u) = ,
(2u2 + 1)2
p p
it follows that f is strictly convex on [0, 1/ 2] and strictly concave on [1/ 2, 3].
By LCRCF-Theorem, it suffices to consider the cases a = 0 and 0 < a ≤ b = c.
Case 1: a = 0. We need to show that b + c = 3 involves
121
(2b2 + 1)(2c 2 + 1) ≤ ,
4
which is equivalent to
(4bc + 5)(4bc − 9) ≤ 0.
This is true because
4bc ≤ (b + c)2 = 9.
Case 2: 0 < a ≤ b = c. We need to show that a + 2b = 3 (0 < a ≤ 1) involves
121
(2a2 + 1)(2b2 + 1)2 ≤ ,
4
which is equivalent to g(a) ≤ 0, where
3−a
g(a) = ln(2a2 + 1) + 2 ln(2b2 + 1) − ln 121 + ln 4, b= , a ∈ (0, 1].
2
Since b0 (a) = −1/2, we have
4a 4b 4(a − b)(1 − 2ab)
g 0 (a) = − 2 =
2a + 1 2b + 1 (2a2 + 1)(2b2 + 1)
2
6(a − 1)(a2 − 3a + 1)
=
(2a2 + 1)(2b2 + 1)
p p
3(1 − a)(3 + 5 − 2a)(2a − 3 + 5)
= ,
2(2a2 + 1)(2b2 + 1)
Half Convex Function Method 141
p p p
3 − 5 3 − 5 3 − 5
hence g 0 = 0, g 0 (a) < 0 for a ∈ 0, , g 0 (a) > 0 for a ∈ ,1 .
2 2 2 p
p
3− 5 3− 5
Therefore, g is strictly decreasing on 0, and strictly increasing on ,1 .
2 2
Since g(0) = 0, it suffices to show that g(1) ≤ 0, which reduces to 27 · 4 ≤ 121.
The equality holds for a = 0 and b = c = 3/2 (or any cyclic permutation).
3
Æ p
k0 = 66 + 10 105 ≈ 4.867,
8
then 2
a+b+c
Æ
3
(a2 + 1)(b2 + 1)(c 2 + 1) ≤ + 1.
3
(Vasile C., 2018)
a+b+c k0
f (a) + f (b) + f (c) ≥ 3 f (s), s= = ,
3 3
where
f (u) = − ln(u2 + 1), u ∈ [0, k0 ].
For u ∈ [s, k0 ], f (u) is convex because
6(3u2 − 1)
f 00 (u) = > 0.
(3u2 + 1)2
f (x) + 2 f ( y) ≥ 3 f (s)
for 0 ≤ x ≤ s ≤ y so that x + 2 y = 3s; that is, to show that g(x) ≥ 0 for x ∈ [0, s],
where
k0 − x
g(x) = f (x) + 2 f ( y) − 3 f (s), y= .
2
Since y 0 (x) = −1/3, we have
−2x 2y
g 0 (x) = f 0 (x) + 2 y 0 f 0 ( y) = +
x2 + 1 y2 + 1
2(x − y)(x y − 1) 3(s − x)(x 2 − k0 x + 2)
= = .
(x 2 + 1)( y 2 + 1) 2(x 2 + 1)( y 2 + 1)
142 Vasile Cîrtoaje
Æ
k0 − k02 − 8
Since g is increasing on [0, s1 ] and decreasing on [s1 , s], where s1 = ,
2
it suffices to show that g(0) ≥ 0 and g(s) ≥ 0. These inequalities are true because
g(0) = 0 and g(s) = 0. The equality g(0) = 0 is equivalent to
2
2y
Æ
( y2 + 1)2 = + 1,
3
3
k0
where y = .
2
According to RHCF-Theorem, if the inequality
a+b+c
f (a) + f (b) + f (c) ≥ 3 f
3
where
f (u) = ln(u2 + 3), u ∈ [0, 4].
From
2(3 − u2 )
f 00 (u) = ,
(u2 + 3)2
p p
it follows that f is strictly convex on [0, 3] and strictly concave on [ 3, 4]. By
LCRCF-Theorem, it suffices to consider the cases a = 0 and 0 < a ≤ b = c.
Case 1: a = 0. We need to show that b + c + d = 4 involves
where k = 27/16. According to Remark from the proof of P 1.86, this inequality
holds for all k ≥ 9/8.
Case 2: 0 < a ≤ b = c = d. We need to show that a + 3b = 4 (0 < a ≤ 1) involves
4−a
g(a) = ln(a2 + 3) + 3 ln(b2 + 3) − ln 513, b= , a ∈ (0, 1].
3
Since b0 (a) = −1/3, we have
Because
a2 − 4a + 9 = (a − 2)2 + 5 > 0,
we have g 0 (a) > 0 for a ∈ [0, 1), g is strictly decreasing on [0, 1], hence it suffices
to show that g(0) ≤ 0. This reduces to show that the original inequality holds for
a = 0 and b = c = d = 4/3, which follows immediately from the case 1.
The equality holds for a = b = c = 0 and d = 4 (or any cyclic permutation).
where
f (u) = ln(u2 + 2), u ∈ [0, 4].
From
2(2 − u2 )
f 00 (u) = ,
(u2 + 2)2
p p
it follows that f is strictly convex on [0, 2] and strictly concave on [ 2, 4]. By
LCRCF-Theorem, it suffices to consider the cases a = 0 and 0 < a ≤ b = c.
144 Vasile Cîrtoaje
4−a
g(a) = ln(a2 + 2) + 3 ln(b2 + 2) − ln 144, b= , a ∈ (0, 1].
3
Because
a2 − 4a + 6 = (a − 2)2 + 2 > 0,
we have g 0 (a) > 0 for a ∈ [0, 1), g is strictly decreasing on [0, 1], hence it suffices
to show that g(0) ≤ 0. This reduces to show that the original inequality holds for
a = 0 and b = c = d = 4/3, which follows immediately from the case 1.
The equality holds for a = b = c = 0 and d = 4 (or any cyclic permutation), and
also for a = b = 0 and c = d = 2 (or any permutation).
a + b + c + d = 4,
then
a b c d 4
+ 3 + 3 + 3 ≤ .
3a3 + 2 3b + 2 3c + 2 3d + 2 5
where
From p
4 = x + 3 y ≥ 2 3x y,
we get 3x y ≤ 4. Thus, we have
a1 a2 = p, p ≤ 1/4,
we have
a13 + a23 = (a1 + a2 )3 − 3a1 a2 (a1 + a2 ) = 1 − 3p,
a14 + a24 = (a12 + a22 )2 − 2a12 a22 = 2p2 − 4p + 1,
and the inequality is equivalent to
(4p − 1)2 ≥ 0.
146 Vasile Cîrtoaje
1
f (a1 ) + f (a2 ) + · · · + f (an ) ≤ ,
8
where
f (u) = u3 − u4 , u ∈ [0, 1].
From
f 00 (u) = 6u(1 − 2u),
it follows that f is strictly convex on [0, 1/2] and strictly concave on [1/2, 1]. By
LCRCF-Theorem, it suffices to consider the cases a1 = 0 and 0 < a1 ≤ a2 = · · · = an .
Case 1: a1 = 0. The inequality follows by the induction hypothesis.
Case 2: 0 < a1 ≤ a2 = · · · = an . We only need to prove the homogeneous inequality
a1 + a2 + · · · + an = ns
and
a1 ≤ a2 ≤ · · · ≤ am ≤ s, m ∈ {1, 2, . . . , n − 1},
if and only if
f (x) + (n − m) f ( y) ≥ (1 + n − m) f (s)
for all x, y ∈ I so that
x ≤ s ≤ y, x + (n − m) y = (1 + n − m)s.
Proof. For
a1 = x, a2 = · · · = am = s, am+1 = · · · = an = y,
the inequality
f (a1 ) + f (a2 ) + · · · + f (an ) ≥ n f (s)
becomes
f (x) + (n − m) f ( y) ≥ (1 + n − m) f (s);
147
148 Vasile Cîrtoaje
a1 ≤ a2 ≤ · · · ≤ an .
k ∈ {m, m + 1, . . . , n − 1}
so that
a1 ≤ · · · ≤ ak ≤ s ≤ ak+1 ≤ · · · ≤ an .
Since f is convex on I≥s , we may apply Jensen’s inequality to get
where
ak+1 + · · · + an
z= , z ∈ I.
n−k
Therefore, to prove the desired inequality
Let b1 , . . . , bk be defined by
ai + (n − m)bi = (1 + n − m)s, i = 1, . . . , k.
We claim that
z ≥ b1 ≥ · · · ≥ bk ≥ s, b1 , . . . , bk ∈ I.
Indeed, we have
b1 ≥ · · · ≥ b k ,
s − ak
bk − s = ≥ 0,
n−m
and
z ≥ b1
because
(n − m)b1 = (1 + n − m)s − a1
= −(m − 1)s + (a2 + · · · + ak ) + (ak+1 + · · · + an )
≤ −(m − 1)s + (k − 1)s + (ak+1 + · · · + an ) =
= (k − m)s + (n − k)z ≤ (n − m)z.
HCF Method for Ordered Variables 149
···
f (ak ) + (n − m) f (bk ) ≥ (1 + n − m) f (s),
hence
which is equivalent to
n−k
p f (z) + (k − p) f (s) ≥ f (b1 ) + · · · + f (bk ), p= ≤ 1.
n−m
By Jensen’s inequality, we have
Since the decreasingly ordered vector A~k = (w, s, . . . , s) majorizes the decreasingly
ordered vector B~k = (b1 , b2 , . . . , bk ), this inequality follows from Karamata’s in-
equality for convex functions.
Similarly, we can prove the Left Half Convex Function Theorem for Ordered Vari-
ables (LHCF-OV Theorem).
a1 + a2 + · · · + an = ns
and
a1 ≥ a2 ≥ · · · ≥ am ≥ s, m ∈ {1, 2, . . . , n − 1},
150 Vasile Cîrtoaje
if and only if
f (x) + (n − m) f ( y) ≥ (1 + n − m) f (s)
for all x, y ∈ I so tht
x ≥ s ≥ y, x + (n − m) y = (1 + n − m)s.
From the RHCF-OV Theorem and the LHCF-OV Theorem, we find the HCF-OV
Theorem (Half Convex Function Theorem for Ordered Variables).
HCF-OV Theorem. Let f be a real function defined on an interval I and convex on
I≥s (or I≤s ), where s ∈ int(I). The inequality
a + a + ··· + a
1 2 n
f (a1 ) + f (a2 ) + · · · + f (an ) ≥ n f
n
holds for all a1 , a2 , . . . , an ∈ I so that
a1 + a2 + · · · + an = ns
f (x) + (n − m) f ( y) ≥ (1 + n − m) f (s)
in the RHCF-OV Theorem and the LHCF-OV Theorem by the equivalent condition
h(x, y) ≥ 0 for all x, y ∈ I so that x + (n − m) y = (1 + n − m)s.
f 0 (x) − f 0 ( y)
H(x, y) = .
x−y
The desired inequality of Jensen’s type in the RHCF-OV Theorem and the LHCF-OV
Theorem holds true by replacing the hypothesis
f (x) + (n − m) f ( y) ≥ (1 + n − m) f (s)
f (x) + (n − m) f ( y) ≥ (1 + n − m) f (s)
f1 (x) ≥ (1 + n − m) f (s),
where
(1 + n − m)s − x
f1 (x) = f (x) + (n − m) f .
n−m
From
(1 + n − m)s − x
f10 (x) = f (x) − f
0 0
n−m
= f (x) − f ( y)
0 0
1+n−m
= (x − s)H(x, y),
n−m
it follows that f1 is decreasing on I≤s and increasing on I≥s ; therefore,
Note 3. The RHCF-OV Theorem and the LHCF-OV Theorem are also valid in the
case when f is defined on I \ {u0 }, where u0 ∈ I<s for the RHCF-OV Theorem, and
u0 ∈ I>s for the LHCF-OV Theorem.
Note 4. The desired inequalities in the RHCF-OV Theorem and the LHCF-OV The-
orem become equalities for
a1 = a2 = · · · = an = s.
a1 = x, a2 = · · · = am = s, am+1 = · · · = an = y
x + (n − m) y = (1 + n − m)s, h(x, y) = 0
(x < y for the RHCF-OV Theorem, and x > y for the LHCF-OV Theorem).
Note 5. The WRHCF-OV Theorem and the WLHCF-OV Theorem are extensions of
the weighted Jensen’s inequality to right and left half convex functions with ordered
variables (Vasile Cirtoaje, 2008).
WRHCF-OV Theorem. Let p1 , p2 , . . . , pn be positive real numbers so that
p1 + p2 + · · · + pn = 1,
and let f be a real function defined on an interval I and convex on I≥s , where s ∈ int(I).
The inequality
p1 f (x 1 ) + p2 f (x 2 ) + · · · + pn f (x n ) ≥ f (p1 x 1 + p2 x 2 + · · · + pn x n )
x1 ≤ x2 ≤ · · · ≤ x n, x m ≤ s, m ∈ {1, 2, . . . , n − 1},
if and only if
f (x) + k f ( y) ≥ (1 + k) f (s)
for all x, y ∈ I satisfying
x ≤ s ≤ y, x + k y = (1 + k)s,
where
pm+1 + pm+2 + · · · + pn
k= .
p1
p1 + p2 + · · · + pn = 1,
and let f be a real function defined on an interval I and convex on I≤s , where s ∈ int(I).
The inequality
p1 f (x 1 ) + p2 f (x 2 ) + · · · + pn f (x n ) ≥ f (p1 x 1 + p2 x 2 + · · · + pn x n )
x1 ≥ x2 ≥ · · · ≥ x n, x m ≥ s, m ∈ {1, 2, . . . , n − 1},
HCF Method for Ordered Variables 153
if and only if
f (x) + k f ( y) ≥ (1 + k) f (s)
for all x, y ∈ I satisfying
x ≥ s ≥ y, x + k y = (1 + k)s,
where
pm+1 + pm+2 + · · · + pn
k= .
p1
154 Vasile Cîrtoaje
HCF Method for Ordered Variables 155
2.2 Applications
a ≤ b ≤ 1 ≤ c ≤ d, a + b + c + d = 4,
then
(3a2 − 2)(a − 1)2 + (3b2 − 2)(b − 1)2 + (3c 2 − 2)(c − 1)2 + (3d 2 − 2)(d − 1)2 ≥ 0.
a ≥ b ≥ 1 ≥ c ≥ d, a + b + c + d = 4,
then
1 1 1 1 4
+ 3 + 3 + 3 ≤ .
2a3 + 5 2b + 5 2c + 5 2d + 5 7
2.3. If
−2n − 1
≤ a1 ≤ · · · ≤ an ≤ 1 ≤ an+1 ≤ · · · ≤ a2n , a1 + a2 + · · · + a2n = 2n,
n−1
then
a13 + a23 + · · · + a2n
3
≥ 2n.
(b) if a1 ≥ a2 ≥ · · · ≥ am ≥ 1, then
2(n2 − 3n + 3) 2
a14 + a24 + · · · + an4 − n ≥ (a1 + a22 + · · · + an2 − n).
n2 − 5n + 7
21(a2 + b2 + c 2 + d 2 + e2 ) ≥ a4 + b4 + c 4 + d 4 + e4 + 100;
(b) if a ≥ b ≥ c ≥ 1 ≥ d ≥ e, then
13(a2 + b2 + c 2 + d 2 + e2 ) ≥ a4 + b4 + c 4 + d 4 + e4 + 60.
then
1 1 1
(n − m + 1) 2
+ + ··· + − n ≥ 4(n − m)(a12 + a22 + · · · + an2 − n).
a1 a2 an
HCF Method for Ordered Variables 157
1 1 1
2.10. If a1 , a2 , . . . , an are positive real numbers so that + + ··· + = n and
a1 a2 an
then
p
n−m
a12 + a22 + ··· + an2 −n≥2 1+ (a1 + a2 + · · · + an − n).
n−m+1
1 1 1 n
+ 2 + ··· + 2 ≥ ;
a12 + 2 a2 + 2 an + 2 3
1 1 1 n
+ 2 + ··· + 2 ≥ .
2a12 + 3 2a2 + 3 2an + 3 5
then
1 1 1 2n
+ 2 + ··· + 2 ≤ .
na12 + n + n + 1 na2 + n + n + 1
2 2 na2n + n + n + 1 (n + 1)2
2
a ≥ b ≥ c ≥ 1 ≥ d ≥ e ≥ f, a + b + c + d + e + f = 6,
then
3a + 4 3b + 4 3c + 4 3d + 4 3e + 4 3f + 4
+ 2 + 2 + 2 + 2 + 2 ≤ 6.
3a + 4 3b + 4 3c + 4 3d + 4 3e + 4 3 f + 4
2
158 Vasile Cîrtoaje
a ≥ b ≥ 1 ≥ c ≥ d ≥ e ≥ f, a + b + c + d + e + f = 6,
then
a2 − 1 b2 − 1 c2 − 1 d2 − 1 e2 − 1 f2−1
+ + + + + ≥ 0.
(2a + 7)2 (2b + 7)2 (2c + 7)2 (2d + 7)2 (2e + 7)2 (2 f + 7)2
a ≤ b ≤ 1 ≤ c ≤ d ≤ e ≤ f, a + b + c + d + e + f = 6,
then
a2 − 1 b2 − 1 c2 − 1 d2 − 1 e2 − 1 f2−1
+ + + + + ≤ 0.
(2a + 5)2 (2b + 5)2 (2c + 5)2 (2d + 5)2 (2e + 5)2 (2 f + 5)2
a ≤ b ≤ 1 ≤ c, a + b + c = 3,
then v v v
t 2a t 2b t 2c
+ + ≥ 3.
b+c c+a a+b
a1 ≥ a2 ≥ a3 ≥ a4 ≥ 1 ≥ a5 ≥ a6 ≥ a7 ≥ a8 , a1 + a2 + · · · + a8 = 8,
then
(a12 + 1)(a22 + 1) · · · (a82 + 1) ≥ (a1 + 1)(a2 + 1) · · · (a8 + 1).
a ≥ b ≥ 1 ≥ c ≥ d, abcd = 1,
then
1 1 1 1
a + b + c + d − 4 ≥ 18 a + b + c + d − − − −
2 2 2 2
.
a b c d
a ≤ b ≤ 1 ≤ c ≤ d, abcd = 1,
then
p p p p
a2 − a + 1 + b2 − b + 1 + c2 − c + 1 + d 2 − d + 1 ≥ a + b + c + d.
a ≤ b ≤ c ≤ 1 ≤ d, abcd = 1,
then
1 1 1 1 2
+ + + ≥ .
a3 + 3a + 2 b3 + 3b + 2 c 3 + 3c + 2 d 3 + 3d + 2 3
a1 ≥ · · · ≥ an−1 ≥ 1 ≥ an , a1 a2 · · · an = 1,
then
1 1 1
+ + ··· + ≥ a1 + a2 + · · · + an .
a1 a2 an
160 Vasile Cîrtoaje
a1 ≤ · · · ≤ an−1 ≤ 1 ≤ an , a1 a2 · · · an = 1.
If k ≥ 1, then
1 1 1 n
+ + ··· + ≥ .
1 + ka1 1 + ka2 1 + kan 1+k
a1 ≤ · · · ≤ a8 ≤ 1 ≤ a9 , a1 a2 · · · a9 = 1,
then
1 1 1
+ + · · · + ≥ 1.
(a1 + 2)2 (a2 + 2)2 (a9 + 2)2
a1 ≤ · · · ≤ an−1 ≤ 1 ≤ an , a1 a2 · · · an = 1.
If p, q ≥ 0 so that
2pq
p+q ≥1+ ,
p + 4q
then
1 1 1 n
+ + ··· + ≥ .
1 + pa1 + qa1 1 + pa2 + qa2
2 2
1 + pan + qan
2 1+p+q
a1 ≤ · · · ≤ an−1 ≤ 1 ≤ an , a1 a2 · · · an = 1.
a1 ≤ · · · ≤ an−1 ≤ 1 ≤ an , a1 a2 · · · an = 1,
then
1 1 1 n
+p + ··· + p ≥ .
1 + 3a1 1 + 3a2 1 + 3an
p
2
HCF Method for Ordered Variables 161
a1 ≤ · · · ≤ an−1 ≤ 1 ≤ an , a1 a2 · · · an = 1.
1
If 0 < m < 1 and 0 < k ≤ , then
21/m −1
1 1 1 n
+ + · · · + ≥ .
(a1 + k)m (a2 + k)m (an + k)m (1 + k)m
a1 ≥ a2 ≥ a3 ≥ 1 ≥ a4 ≥ · · · ≥ an , a1 a2 · · · an = 1,
then
1 1 1 n
+ + ··· + ≥ .
3a1 + 1 3a2 + 1 3an + 1 4
a1 ≥ a2 ≥ a3 ≥ 1 ≥ a4 ≥ · · · ≥ an , a1 a2 · · · an = 1,
then
1 1 1 n
+ + ··· + ≥ .
(a1 + 1)2 (a2 + 1)2 (an + 1)2 4
a1 ≥ · · · ≥ an−1 ≥ 1 ≥ an , a1 a2 · · · an = 1,
then
1 1 1 n
+ + · · · + ≤ .
(a1 + 3)2 (a2 + 3)2 (an + 3)2 16
a1 ≥ · · · ≥ an−1 ≥ 1 ≥ an , a1 a2 · · · an = 1.
If p, q ≥ 0 so that p + q ≤ 1, then
1 1 1 n
+ + ··· + ≤ .
1 + pa1 + qa1 1 + pa2 + qa2
2 2
1 + pan + qan
2 1+p+q
162 Vasile Cîrtoaje
2.3 Solutions
a ≤ b ≤ 1 ≤ c ≤ d, a + b + c + d = 4,
then
(3a2 − 2)(a − 1)2 + (3b2 − 2)(b − 1)2 + (3c 2 − 2)(c − 1)2 + (3d 2 − 2)(d − 1)2 ≥ 0.
a+b+c+d
f (a) + f (b) + f (c) + f (d) ≥ 4 f (s), s= = 1,
4
where
f (u) = (3u2 − 2)(u − 1)2 , u ∈ I = R.
From
f 00 (u) = 2(18u2 − 18u + 1),
it follows that f 00 (u) > 0 for u ≥ 1, hence f is convex on I≥s . Therefore, we may
apply the RHCF-OV Theorem for n = 4 and m = 2. Thus, it suffices to show that
f (x) + 2 f ( y) ≥ 3 f (1) for all real x, y so that x + 2 y = 3. Using Note 1, we only
need to show that h(x, y) ≥ 0, where
We have
1 4
a= , b = 1, c=d= .
3 3
n
If k = , then
n2 − n + 1
(a12 − k)(a1 − 1)2 + (a22 − k)(a2 − 1)2 + · · · + (a2n
2
− k)(a2n − 1)2 ≥ 0,
1 n2
a1 = 2 , a2 = · · · = an = 1, an+1 = · · · = an = 2 .
n −n+1 n −n+1
a ≥ b ≥ 1 ≥ c ≥ d, a + b + c + d = 4,
then
1 1 1 1 4
+ 3 + 3 + 3 ≤ .
2a3 + 5 2b + 5 2c + 5 2d + 5 7
(Vasile C., 2009)
a+b+c+d
f (a) + f (b) + f (c) + f (d) ≥ 4 f (s), s= = 1,
4
where
−1
f (u) = , u ≥ 0.
2u3 + 5
From
12u(5 − 4u3 )
f 00 (u) = ,
(2u3 + 5)3
it follows that f 00 (u) ≥ 0 for u ∈ [0, 1], hence f is convex on [0, s]. Therefore, we
may apply the LHCF-OV Theorem for n = 4 and m = 2. Using Note 1, we only need
to show that h(x, y) ≥ 0 for x, y ≥ 0 so that x + 2 y = 3. We have
g(x) − g( y) 2E
h(x, y) = = ,
x−y 7(2x 3 + 5)(2 y 3 + 5)
where
Since
E = (1 − 2 y)2 (2 + 3 y − 2 y 2 ) = (1 − 2 y)2 (2 + x y) ≥ 0,
HCF Method for Ordered Variables 165
then
1 1 1 2n2
+ + ··· + ≥ 2 ,
a13 + n + 1n a23 + n + 1n 3
a2n + n + 1n n +n+1
with equality for a1 = a2 = · · · = a2n = 1, and also for
1
a1 = n, a2 = · · · = an = 1, an+1 = · · · = a2n = .
n
P 2.3. If
−2n − 1
≤ a1 ≤ · · · ≤ an ≤ 1 ≤ an+1 ≤ · · · ≤ a2n , a1 + a2 + · · · + a2n = 2n,
n−1
then
a13 + a23 + · · · + a2n
3
≥ 2n.
g(x) − g( y) (n − 1)x + 2n + 1
h(x, y) = = x + y +1= ≥ 0.
x−y n−1
From x + n y = 1 + n and h(x, y) = 0, we get
−2n − 1 n+2
x= , y= .
n−1 n−1
In accordance with Note 4, the equality holds for a1 = a2 = · · · = a2n = 1, and also
for
−2n − 1 n+2
a1 = , a2 = · · · = an = 1, an+1 = · · · = a2n = .
n−1 n−1
n−1
(b) if − ≤ a1 ≤ a2 ≤ 1 ≤ · · · ≤ an , then
n−3
f (x) + 2 f ( y) ≥ 3 f (1)
We have
g(u) = u2 ,
x +3
h(x, y) = x + y = ≥ 0.
2
From x + 2 y = 3 and h(x, y) = 0, we get x = −3 and y = 3. Therefore, in
accordance with Note 4, the equality holds for a1 = a2 = · · · = an = 1, and also for
f (x) + (n − 2) f ( y) ≥ (n − 1) f (1)
n−1
for − ≤ x ≤ y satisfying x + (n − 2) y = n − 1. Using Note 1, we only need to
n−3
show that h(x, y) ≥ 0, where
We have
g(u) = u2 − u − 1,
(n − 3)x + n − 1
h(x, y) = x + y − 1 = ≥ 0.
n−1
n−1 n−1
From x + (n − 2) y = n − 1 and h(x, y) = 0, we get x = − and y = .
n−3 n−3
Therefore, in accordance with Note 4, the equality holds for a1 = a2 = · · · = an = 1.
If n ≥ 4, then the equality holds also for
n−1 n−1
a1 = − , a2 = 1, a3 = · · · = a n = .
n−3 n−3
168 Vasile Cîrtoaje
(b) if a1 ≥ a2 ≥ · · · ≥ am ≥ 1, then
hence f is convex on I≥s . Thus, by the RHCF-OV Theorem and Note 1, we only need
to show that h(x, y) ≥ 0 for all nonnegative numbers x, y so that x + (n − m) y =
n − m + 1. We have
f (u) − f (1)
g(u) = = (n − m)(u2 + u + 1) − (2n − 2m + 1)(u + 1)
u−1
= (n − m)u2 − (n − m + 1)u − n + m − 1,
g(x) − g( y)
h(x, y) = = (n − m)(x + y) − n + m − 1 = (n − m − 1)x ≥ 0.
x−y
From x +(n−m) y = 1+n−m and h(x, y) = 0, we get x = 0, y = (n−m+1)/(n−m).
Therefore, in accordance with Note 4, the equality holds for a1 = a2 = · · · = an = 1,
and also for
1
a1 = 0, a2 = · · · = am = 1, am+1 = · · · = an = 1 + .
n−m
(b) Write the inequality as
a1 + a2 + · · · + an
f (a1 ) + f (a2 ) + · · · + f (an ) ≥ n f (s), s= = 1,
n
where
f (u) = (n − m + 2)u2 − u3 , u ∈ I = [0, n].
HCF Method for Ordered Variables 169
For u ≤ 1, we have
hence f is convex on I≤s . By the LHCF-OV Theorem and Note 1, it suffices to show
that h(x, y) ≥ 0 for all x, y ≥ 0 so that x + (n − m) y = 1 + n − m. We have
f (u) − f (1)
g(u) = = (n − m + 2)(u + 1) − (u2 + u + 1)
u−1
= −u2 + (n − m + 1)u + n − m + 1,
g(x) − g( y)
h(x, y) = = −(x + y) + n − m + 1 = (n − m − 1) y ≥ 0.
x−y
From x + (n − m) y = 1 + n − m and h(x, y) = 0, we get x = n − m + 1, y = 0.
Therefore, the equality holds for a1 = a2 = · · · = an = 1, and also for
a1 = n − m + 1, a2 = · · · = am = 1, am+1 = · · · = an = 0.
a1 = n, a2 = a3 = · · · = an = 0
a1 ≤ · · · ≤ an−1 ≤ 1 ≤ an , a1 + a2 + · · · + an = n,
then
a13 + a23 + · · · + an3 + 2n ≥ 3(a12 + a22 + · · · + an2 ),
170 Vasile Cîrtoaje
a1 = 0, a2 = · · · = an−1 = 1, an = 2.
a1 ≥ · · · ≥ an−1 ≥ 1 ≥ an , a1 + a2 + · · · + an = n,
then
a13 + a23 + · · · + an3 + 2n ≤ 3(a12 + a22 + · · · + an2 ),
with equality for a1 = a2 = · · · = an = 1, and also for
a1 = 2, a2 = · · · = an−1 = 1, an = 0.
then
n(a13 + a23 + · · · + a2n
3
− 2n) ≥ (2n + 1)(a12 + a22 + · · · + a2n
2
− 2n),
with equality for a1 = a2 = · · · = a2n = 1, and also for
1
a1 = 0, a2 = · · · = an = 1, an+1 = · · · = a2n = 1 + .
n
then
a13 + a23 + · · · + a2n
3
− 2n ≤ (n + 2)(a12 + a22 + · · · + a2n
2
− 2n),
with equality for a1 = a2 = · · · = a2n = 1, and also for
a1 = n + 1, a2 = · · · = an = 1, an+1 = · · · = a2n = 0.
2(n2 − 3n + 3) 2
a14 + a24 + ··· + an4 −n≥ (a1 + a22 + · · · + an2 − n).
n − 5n + 7
2
and write it as
a1 + a2 + · · · + an
f (a1 ) + f (a2 ) + · · · + f (an ) ≥ n f (s), s= = 1,
n
where
f (u) = u4 − ku2 , u ∈ R.
From f 00 (u) = 2(6u2 − k), it follows that f is convex for u ≥ 1. Therefore, we may
apply the RHCF-OV Theorem for m = n − 1, m = n − 2 and m = 2, respectively. By
Note 1, it suffices to show that h(x, y) ≥ 0 for all real x, y so that x + (n − m) y =
1 + n − m. We have
f (u) − f (1)
g(u) = = u3 + u2 + u + 1 − k(u + 1),
u−1
g(x) − g( y)
h(x, y) = = x 2 + x y + y 2 + x + y + 1 − k.
x−y
2(n2 − 3n + 3)
(c) We have k = , m = 2 and x +(n−2) y = n−1, which involve
n2 − 5n + 7
[(n2 − 5n + 7) y − n2 + 3n − 1]2
h(x, y) = ≥ 0.
n2 − 5n + 7
From x + (n − 2) y = n − 1 and h(x, y) = 0, we get
−n2 + 5n − 5 n2 − 3n + 1
x= , y= .
n2 − 5n + 7 n2 − 5n + 7
Therefore, the equality holds for a1 = a2 = · · · = an = 1, and also for
−n2 + 5n − 5 n2 − 3n + 1
a1 = , a2 = 1, a3 = · · · = a n = .
n2 − 5n + 7 n2 − 5n + 7
21(a2 + b2 + c 2 + d 2 + e2 ) ≥ a4 + b4 + c 4 + d 4 + e4 + 100;
(b) if a ≥ b ≥ c ≥ 1 ≥ d ≥ e, then
13(a2 + b2 + c 2 + d 2 + e2 ) ≥ a4 + b4 + c 4 + d 4 + e4 + 60.
k(a2 + b2 + c 2 + d 2 + e2 − 5) ≥ a4 + b4 + c 4 + d 4 + e4 − 5, k ≥ 6,
and write it as
a+b+c+d+e
f (a) + f (b) + f (c) + f (d) + f (e) ≥ 5 f (s), s= = 1,
5
where
f (u) = ku2 − u4 , u ≥ 0.
From f 00 (u) = 2(k − 6u2 ), it follows that f is convex on [0, 1]. Therefore, we may
apply the LHCF-OV Theorem for m = 2 and m = 3, respectively. By Note 1, it
suffices to show that h(x, y) ≥ 0 for all x, y ≥ 0 so that x + (5 − m) y = 6 − m. We
have
f (u) − f (1)
g(u) = = k(u + 1) − (u3 + u2 + u + 1),
u−1
HCF Method for Ordered Variables 173
g(x) − g( y)
h(x, y) = = k − (x 2 + x y + y 2 + x + y + 1).
x−y
(a) We need to show that h(x, y) ≥ 0 for k = 21, n = 5, m = 2 and x +3 y = 4;
indeed, we have
a = 4, b = 1, c = d = e = 0.
a = 3, b = c = 1, d = e = 0.
and write it as
a1 + a2 + · · · + an
f (a1 ) + f (a2 ) + · · · + f (an ) ≥ n f (s), s= = 1,
n
where
f (u) = ku3 − u4 , u ≥ 0.
174 Vasile Cîrtoaje
From f 00 (u) = 6u(k − 2u2 ), it follows that f is convex on [0, 1]. Therefore, we may
apply the LHCF-OV Theorem for m = n − 1 and m = n − 2, respectively. By Note 1,
it suffices to show that h(x, y) ≥ 0 for x ≥ y ≥ 0 so that x + m y = 1 + m. We have
f (u) − f (1)
g(u) = = k(u2 + u + 1) − (u3 + u2 + u + 1),
u−1
g(x) − g( y)
h(x, y) = = −(x 2 + x y + y 2 ) + (k − 1)(x + y + 1).
x−y
a1 = 2, a2 = · · · = an−1 = 1, an = 0.
a1 = 3, a2 = · · · = an−2 = 1, an−1 = an = 0.
then
1 1 1
(n − m + 1) 2
+ + ··· + − n ≥ 4(n − m)(a12 + a22 + · · · + an2 − n).
a1 a2 an
1 1 1
P 2.10. If a1 , a2 , . . . , an are positive real numbers so that + +···+ = n and
a1 a2 an
a1 ≤ · · · ≤ am ≤ 1 ≤ am+1 ≤ · · · ≤ an , m ∈ {1, 2, . . . , n − 1},
then
p
n−m
a12 + a22 + ··· + an2 −n≥2 1+ (a1 + a2 + · · · + an − n).
n−m+1
a1 ≥ · · · ≥ am ≥ 1 ≥ am+1 ≥ · · · ≥ an , a1 + a2 + · · · + a n = n
involves
a1 + a2 + · · · + an
f (a1 ) + f (a2 ) + · · · + f (an ) ≥ n f (s), s= = 1,
n
where p
1 2k m−n
f (u) = 2 − , k =1+ , u > 0.
u u n−m+1
For u ∈ (0, 1], we have
p
6 − 4ku 6 − 4k 2( n − m − 1)2
f (u) =
00
≥ = ≥ 0.
u4 u4 (n − m + 1)u4
Thus, f is convex on (0, 1]. By the LHCF-OV Theorem and Note 1, it suffices to
show that h(x, y) ≥ 0 for x, y > 0 so that x + (n − m) y = 1 + n − m, where
g(x) − g( y) f (u) − f (1)
h(x, y) = , g(u) = .
x−y u−1
We have
−1 2k − 1
g(u) = +
u2 u
and
1 1 1
h(x, y) = + + 1 − 2k .
xy x y
We only need to show that
p
1 1 2 n−m
+ ≥1+ .
x y n−m+1
Indeed, using the Cauchy-Schwarz inequality, we get
p p p
1 1 (1 + n − m)2 (1 + n − m)2 2 n−m
+ ≥ = =1+ .
x y x + (n − m) y n−m+1 n−m+1
HCF Method for Ordered Variables 177
and write it as
a1 + a2 + · · · + an
f (a1 ) + f (a2 ) + · · · + f (an ) ≥ n f (s), s= = 1,
n
and
1
f (u) = , u ≥ 0.
u2 +k
For u ≥ 1, we have
2(3u2 − k) 2(3 − k)
f (u) =
00
≥ 2 ≥ 0,
(u + k)
2 3 (u + k)3
hence f (u) is convex for u ≥ s. Therefore, we may apply the RHCF-OV Theorem for
m = n−1 and m = n−2, respectively. By Note 1, it suffices to show that h(x, y) ≥ 0
for all x, y ≥ 0 so that x + (n − m) y = 1 + n − m. Since
f (u) − f (1) −u − 1
g(u) = = ,
u−1 (1 + k)(u2 + k)
g(x) − g( y) xy+x + y−k
h(x, y) = = ,
x−y (1 + k)(x 2 + k)( y 2 + k)
we only need to show that
x y + x + y − k ≥ 0.
a1 = 0, a2 = · · · = an−1 = 1, an = 2.
then
1 1 1 2n
+ 2 + ··· + 2 ≤ .
na12 + n + n + 1 na2 + n + n + 1
2 2 na2n + n + n + 1 (n + 1)2
2
hence f is convex on [0, s]. Therefore, we may apply the LHCF-OV Theorem for 2n
numbers and m = n. By Note 1, it suffices to show that h(x, y) ≥ 0 for all x, y ≥ 0
so that x + n y = 1 + n. We have
g(x) − g( y)
h(x, y) =
x−y
n(n2 + n + 1 − nx − n y − nx y)
=
(n + 1)2 (nx 2 + n2 + n + 1)(n y 2 + n2 + n + 1)
n(n y − 1)2
= ≥ 0.
(n + 1)2 (nx 2 + n2 + n + 1)(n y 2 + n2 + n + 1)
a1 = n, a2 = · · · = an = 1, an+1 = · · · = an = f r ac1n.
180 Vasile Cîrtoaje
g(x) − g( y) 3(4 − 3x y)
h(x, y) = =
x−y (3x + 4)(3 y 2 + 4)
2
3(x − 2)2
= ≥ 0.
(3x 2 + 4)(3 y 2 + 4)
From x + 3 y = 4 and h(x, y) = 0, we get x = 2 and y = 2/3. Therefore, in
accordance with Note 4, the equality holds for a = b = c = d = e = f = 1, and
also for
2
a = 2, b = c = 1, d = e = f = .
3
a+b+c+d+e+ f
f (a) + f (b) + f (c) + f (d) + f (e) + f ( f ) ≥ 6 f (s), s= = 1,
6
where
u2 − 1
f (u) = , u ≥ 0.
(2u + 7)2
For u ∈ [0, 1], we have
2(37 − 28u)
f 00 (u) = > 0,
(2u + 7)4
hence f is convex on [0, s]. Therefore, we may apply the LHCF-OV Theorem for
n = 6 and m = 2. By Note 1, it suffices to show that h(x, y) ≥ 0 for all x, y ≥ 0 so
that x + 4 y = 5. We have
g(x) − g( y) 21 − 4x − 4 y − 4x y
h(x, y) = =
x−y (2x + 7)2 (2 y + 7)2
(x − 4)2
= ≥ 0.
(2x + 7)2 (2 y + 7)2
1
a = 4, b = 1, c=d=e= f = .
4
a ≤ b ≤ 1 ≤ c ≤ d ≤ e ≤ f, a + b + c + d + e + f = 6,
then
a2 − 1 b2 − 1 c2 − 1 d2 − 1 e2 − 1 f2−1
+ + + + + ≤ 0.
(2a + 5)2 (2b + 5)2 (2c + 5)2 (2d + 5)2 (2e + 5)2 (2 f + 5)2
a+b+c+d+e+ f
f (a) + f (b) + f (c) + f (d) + f (e) + f ( f ) ≥ 6 f (s), s= = 1,
6
where
1 − u2
f (u) = , u ≥ 0.
(2u + 5)2
For u ≥ 1, we have
2(20u − 13)
f 00 (u) = > 0,
(2u + 5)4
hence f (u) is convex for u ≥ s. Therefore, we may apply the RHCF-OV Theorem
for n = 6 and m = 2. By Note 1, it suffices to show that h(x, y) ≥ 0 for all x, y ≥ 0
so that x + 4 y = 5. We have
f (u) − f (1) −u − 1
g(u) = = ,
u−1 (2u + 5)2
g(x) − g( y)
h(x, y) =
x−y
4x y + 4x + 4 y − 5
=
(2x + 5)2 (2 y + 5)2
4x y + 3x
= ≥ 0.
(2x + 5)2 (2 y + 5)2
a ≤ b ≤ 1 ≤ c, a + b + c = 3,
then v v v
t 2a t 2b t 2c
+ + ≥ 3.
b+c c+a a+b
a+b+c
f (a) + f (b) + f (c) ≥ 3 f (s), s= = 1,
3
where s
u
f (u) = , u ∈ [0, 3).
3−u
From
3(4u − 3)
f 00 (u) = ,
4u3/2 (3 − u)5/2
it follows that f (u) is convex for u ≥ s. Therefore, we may apply the RHCF-OV
Theorem for n = 3 and m = 2. So, it suffices to show that
f (x) + f ( y) ≥ 2 f (1)
Let
2
x1 = 1 − p .
5
Since h(x 1 ) = 0, h(x) > 0 for x ∈ [0, x 1 ) and h(x) < 0 for x ∈ (x 1 , 1), it follows
that g is increasing on [0, x 1 ] and decreasing on [x 1 , 1]. From
a1 ≥ a2 ≥ a3 ≥ a4 ≥ 1 ≥ a5 ≥ a6 ≥ a7 ≥ a8 , a1 + a2 + · · · + a8 = 8,
then
(a12 + 1)(a22 + 1) · · · (a82 + 1) ≥ (a1 + 1)(a2 + 1) · · · (a8 + 1).
a1 + a2 + · · · + a8
f (a1 ) + f (a2 ) + · · · + f (a8 ) ≥ 8 f (s), s= = 1,
8
where
f (u) = ln(u2 + 1) − ln(u + 1), u ≥ 0.
For u ∈ [0, 1], we have
Therefore, f is convex on [0, s]. According to the LHCF-OV Theorem applied for
n = 8 and m = 4, it suffices to show that f (x) + 4 f ( y) ≥ 5 f (1) for x, y ≥ 0 so that
x + 4 y = 5. Using Note 2, we only need to show that H(x, y) ≥ 0 for x, y ≥ 0 so
that x + 4 y = 5, where
f 0 (x) − f 0 ( y) 2(1 − x y) 1
H(x, y) = = 2 + .
x−y (x + 1)( y + 1) (x + 1)( y + 1)
2
8(1 − x y) + (x + 1)( y + 1) ≥ 0.
Indeed,
−1
≤ a ≤ b ≤ 1 ≤ c ≤ d, a + b + c + d = 4,
2
then
1 1 1 1 1 1 1 1
7 2 + 2 + 2 + 2 +3 + + + ≥ 40.
a b c d a b c d
Solution. We have
1 1
d =4−a− b−c ≤4+ + − 1 = 4.
2 2
a+b+c+d
f (a) + f (b) + f (c) + f (d) ≥ 4 f (s), s= = 1,
4
where
7 3 −1
f (u) = 2 + , u∈I= , 4 \ {0}.
u u 2
7 3
Clearly, f (u) is convex for u ≥ 1 (because 2
and are convex). According to Note
u u
3, we may apply the RHCF-OV Theorem for n = 4 and m = 2. By Note 1, we only
need to show that h(x, y) ≥ 0 for x, y ∈ I so that x + 2 y = 3, where
We have
7 10
g(u) = − 2
− ,
u u
7(x + y) + 10x y (2x + 1)(−5x + 21)
h(x, y) = 2 2
= ≥ 0.
x y 2x 2 y 2
From x + 2 y = 3 and h(x, y) = 0, we get x = −1/2, y = 7/3. Therefore, in
accordance with Note 4, the equality holds for a = b = c = d = 1, and also for
−1 7
a= , b = 1, c=d= .
2 4
186 Vasile Cîrtoaje
1 1 1 1 1 1 1 1
3 2 + 2 + 2 + 2 ≥8+ + + + ;
a b c d a b c d
(b) if −1 ≤ a ≤ b ≤ 1 ≤ c ≤ d, then
1 1 1 1 1 1 1 1
2 2 + 2 + 2 + 2 ≥4+ + + + .
a b c d a b c d
d = 4 − a − b − c ≤ 4 + 1 + 1 + 1 = 7.
a+b+c+d
f (a) + f (b) + f (c) + f (d) ≥ 4 f (s), s= = 1,
4
where
3 1
f (u) = − , u ∈ I = [−1, 7] \ {0}.
u2 u
From
2(9 − u)
f 00 (u) = > 0,
u4
it follows that f is convex on I≥s . According to Note 3, we may apply the RHCF-OV
Theorem for n = 4 and m = 3. By Note 1, it suffices to show that h(x, y) ≥ 0 for
all x, y ∈ I so that x + y = 2. We have
f (u) − f (1) 2 3
g(u) = = − − 2,
u−1 u u
g(x) − g( y) 3(x + y) + 2x y
h(x, y) = =
x−y x2 y2
2(x + 1)(3 − x) 2(x + 1)( y + 1)
= = ≥ 0.
x2 y2 x2 y2
a = −1, b = c = 1, d = 3.
(b) We have
d = 4 − a − b − c ≤ 4 + 1 + 1 − 1 = 5.
HCF Method for Ordered Variables 187
a+b+c+d
f (a) + f (b) + f (c) + f (d) ≥ 4 f (s), s= = 1,
4
where
2 1
f (u) = − , u ∈ I = [−1, 5] \ {0}.
u2 u
From
2(6 − u)
f 00 (u) = > 0,
u4
it follows that f is convex on I≥s . According to Note 3, we may apply the RHCF-OV
Theorem for n = 4 and m = 2. By Note 1, it suffices to show that h(x, y) ≥ 0 for
all x, y ∈ I so that x + 2 y = 3. We have
f (u) − f (1) 1 2
g(u) = = − − 2,
u−1 u u
g(x) − g( y) 2(x + y) + x y
h(x, y) = =
x−y x2 y2
(x + 1)(6 − x)
= ≥ 0.
2x 2 y 2
a = −1, b = 1, c = d = 2.
a ≥ b ≥ 1 ≥ c ≥ d, abcd = 1,
then
1 1 1 1
a + b + c + d − 4 ≥ 18 a + b + c + d − − − −
2 2 2 2
.
a b c d
(Vasile C., 2008)
a = ex , b = ey, c = ez , d = ew,
where
x + y +z+w
x ≥ y ≥ 0 ≥ z ≥ w, s= = 0,
4
f (u) = e2u − 1 − 18(eu − e−u ), u ∈ R.
For u ≤ 0, we have
f 00 (u) = 4e2u + 18(e−u − eu ) > 0,
hence f is convex on (−∞, s]. By the LHCF-OV Theorem applied for n = 4 and
m = 2, it suffices to show that f (x) + 2 f ( y) ≥ 3 f (0) for all real x, y so that
x + 2 y = 0; that is, to show that
1 2
a + 2b − 3 − 18 a + 2b − −
2 2
≥0
a b
for all a, b > 0 so that ab2 = 1. This inequality is equivalent to
a = 4, b = 1, c = d = 1/2.
a ≤ b ≤ 1 ≤ c ≤ d, abcd = 1,
then
p p p p
a2 − a + 1 + b2 − b + 1 + c2 − c + 1 + d 2 − d + 1 ≥ a + b + c + d.
a = ex , b = ey, c = ez , d = ew,
where
x + y +z+w
x ≤ y ≤ 0 ≤ z ≤ w, s= = 0,
4
HCF Method for Ordered Variables 189
p
f (u) = e2u − eu + 1 − eu , u ∈ R.
We claim that f is convex for u ≥ 0. Since
and
b2 − 1 2(1 − b)
+p ≥ 0,
b +1
2
b2 − b + 1 + b
which is equivalent to
b+1 2
(b − 1) 2 −p ≥ 0,
b +1 b2 − b + 1 + b
190 Vasile Cîrtoaje
p
(b − 1) (b + 1) b2 − b + 1 − b2 + b − 2 ≥ 0,
(b − 1)2 (3b2 − 2b + 3)
p ≥ 0.
(b + 1) b2 − b + 1 + b2 − b + 2
The last inequality is clearly true. The equality holds for a = b = c = d = 1.
a ≤ b ≤ c ≤ 1 ≤ d, abcd = 1,
then
1 1 1 1 2
+ 3 + 3 + 3 ≥ .
a3 + 3a + 2 b + 3b + 2 c + 3c + 2 d + 3d + 2 3
(Vasile C., 2007)
Solution. Using the substitution
a = ex , b = ey, c = ez , d = ew,
where
x + y +z+w
x ≤ y ≤ z ≤ 0 ≤ w, s= = 0,
4
1
f (u) = , u ∈ R.
+ 3eu + 2
e3u
We claim that f is convex for u ≥ 0. Indeed, denoting t = eu , t ≥ 1, we have
3t(3t 5 + 2t 3 − 6t 2 + 3t − 2)
f 00 (u) =
(t 3 + 3t + 2)3
3t(t − 1)(3t 4 + 3t 3 + 5t 2 − t + 2)
= ≥ 0.
(t 3 + 3t + 2)3
By the RHCF-OV Theorem applied for n = 4 and m = 3, it suffices to show that
f (x) + f ( y) ≥ 2 f (0) for all real x, y so that x + y = 0; that is, to show that
1 1 1
+ 3 ≥
a3 + 3a + 2 b + 3b + 2 3
for all a, b > 0 so that ab = 1. This inequality is equivalent to
(a − 1)4 (a2 + a + 1) ≥ 0.
a1 ≥ · · · ≥ an−1 ≥ 1 ≥ an , a1 a2 · · · an = 1,
then
1 1 1
+ + ··· + ≥ a1 + a2 + · · · + an .
a1 a2 an
(Vasile C., 2007)
Solution. Using the substitution
ai = e x i , i = 1, 2, . . . , n,
f (x 1 ) + f (x 2 ) + · · · + f (x n ) ≥ n f (s),
where
x1 + x2 + · · · + x n
x 1 ≥ · · · ≥ x n−1 ≥ 0 ≥ x n , s= = 0,
n
f (u) = e−u − eu , u ∈ R.
For u ≤ 0, we have
f 00 (u) = e−u − eu ≥ 0,
therefore f (u) is convex for u ≤ s. By the LHCF-OV Theorem applied for m = n − 1,
it suffices to show that f (x) + f ( y) ≥ 2 f (0) for all real x, y so that x + y = 0; that
is, to show that
1 1
−a+ −b≥0
a b
for all a, b > 0 so that ab = 1. This is true since
1 1 1 1
− a + − b = − a + a − = 0.
a b a a
The equality holds for
a1 ≥ 1, a2 = · · · = an−1 = 1, an = 1/a1 .
a1 ≤ · · · ≤ an−1 ≤ 1 ≤ an , a1 a2 · · · an = 1.
If k ≥ 1, then
1 1 1 n
+ + ··· + ≥ .
1 + ka1 1 + ka2 1 + kan 1+k
(Vasile C., 2007)
192 Vasile Cîrtoaje
ai = e x i , i = 1, 2, . . . , n,
f (x 1 ) + f (x 2 ) + · · · + f (x n ) ≥ n f (s),
where
x1 + x2 + · · · + x n
x 1 ≤ · · · ≤ x n−1 ≤ 0 ≤ x n , s= = 0,
n
1
f (u) = , u ∈ R.
1 + keu
For u ≥ 0, we have
keu (keu − 1)
f (u) =
00
≥ 0,
(1 + keu )3
therefore f (u) is convex for u ≥ s. By the RHCF-OV Theorem applied for m = n−1,
it suffices to show that f (x) + f ( y) ≥ 2 f (0) for all real x, y so that x + y = 0; that
is, to show that
1 1 2
+ ≥
1 + ka 1 + kb 1 + k
for all a, b > 0 so that ab = 1. This is true since
a1 ≤ 1, a2 = · · · = an−1 = 1, an = 1/a1 .
a1 ≤ · · · ≤ a8 ≤ 1 ≤ a9 , a1 a2 · · · a9 = 1,
then
1 1 1
+ + ··· + ≥ 1.
(a1 + 2)2 (a2 + 2)2 (a9 + 2)2
ai = e x i , i = 1, 2, . . . , 9,
f (x 1 ) + f (x 2 ) + · · · + f (x 9 ) ≥ 9 f (s),
where
x1 + x2 + · · · + x9
x1 ≤ · · · ≤ x8 ≤ 0 ≤ x9, s= = 0,
9
1
f (u) = , u ∈ R.
(eu + 2)2
For u ∈ [0, ∞), we have
4eu (eu − 1)
f 00 (u) = ≥ 0,
(eu + 2)4
hence f is convex on [s, ∞). According to the RHCF-OV Theorem (case n = 9
and m = 8), it suffices to show that f (x) + f ( y) ≥ 2 f (0) for all real x, y so that
x + y = 0; that is, to show that
1 1 2
+ ≥
(a + 2)2 (b + 2)2 9
for all a, b > 0 so that ab = 1. Write this inequality as
b2 1 2
+ ≥ ,
(2b + 1)2 (b + 2)2 9
which is equivalent to the obvious inequality
(b − 1)4 ≥ 0.
a1 ≤ · · · ≤ an−1 ≤ 1 ≤ an , a1 a2 · · · an = 1.
If p, q ≥ 0 so that
2pq
p+q ≥1+ ,
p + 4q
then
1 1 1 n
+ + ··· + ≥ .
1 + pa1 + qa1 1 + pa2 + qa2
2 2
1 + pan + qan
2 1+p+q
which is equivalent to
(a − 1)2 h(a) ≥ 0,
where
h(a) = q(p + q − 1)(a2 + 1) + (p2 + pq + 2q2 − p − 2q)a
≥ 2q(p + q − 1)a + (p2 + pq + 2q2 − p − 2q)a
= (p2 + 3pq + 4q2 − p − 4q)a ≥ 0.
The equality holds for a1 = a2 = · · · = an = 1.
Remark. For p = 1, q = 1/4 and n = 9, we get the preceding P 2.25.
HCF Method for Ordered Variables 195
a1 ≤ · · · ≤ an−1 ≤ 1 ≤ an , a1 a2 · · · an = 1.
1 1 1 n
+ + ··· + ≥ .
(a1 + k) m (a2 + k) m (an + k) m (1 + k)m
ai = e x i , i = 1, 2, . . . , n,
f (x 1 ) + f (x 2 ) + · · · + f (x n ) ≥ n f (s),
where
x1 + x2 + · · · + x n
x 1 ≤ · · · ≤ x n−1 ≤ 0 ≤ x n , s= = 0,
n
1
f (u) = , u ∈ R.
(eu + k)m
For u ∈ [0, ∞), we have
meu (meu − k)
f (u) =
00
≥ 0,
(eu + k)m+2
hence f is convex on [s, ∞). According to the RHCF-OV Theorem (case m = n−1),
it suffices to show that f (x) + f ( y) ≥ 2 f (0) for all real x, y so that x ≤ y and
x + y = 0; that is, to show that
1 1 2
+ ≥
(a + k) m (b + k) m (1 + k)m
for all a, b > 0 so that a ∈ (0, 1] and ab = 1. Write this inequality as g(a) ≥ 0,
where
1 am 2
g(a) = + − ,
(a + k) m (ka + 1) m (1 + k)m
with
g 0 (a) a m−1 (a + k)m+1 − (ka + 1)m+1
= .
m (a + k)m+1 (ka + 1)m+1
If g 0 (a) ≤ 0 for a ∈ (0, 1], then g is decreasing, hence g(a) ≥ g(1) = 0. Thus, it
suffices to show that
ka + 1 m+1
m−1
a ≤ .
a+k
196 Vasile Cîrtoaje
Since
ka + 1 ma + 1 (m − k)(1 − a2 )
− = ≥ 0,
a+k a+m (a + k)(a + m)
we only need to show that
ma + 1
m+1
m−1
a ≤ ,
a+m
which is equivalent to h(a) ≤ 0 for a ∈ (0, 1], where
with
m − 1 m + 1 m(m + 1) m(m − 1)(a − 1)2
h (a) =
0
+ − = .
a a+m ma + 1 a(a + m)(ma + 1)
Since h0 (a) ≥ 0, h(a) is increasing for a ∈ (0, 1], therefore h(a) ≤ h(1) = 0. The
equality holds for a1 = a2 = · · · = an = 1.
Remark. For k = m = 2 and n = 9, we get the inequality in P 2.25.
a1 ≤ · · · ≤ an−1 ≤ 1 ≤ an , a1 a2 · · · an = 1,
then
1 1 1 n
+p + ··· + p ≥ .
1 + 3a1 1 + 3a2 1 + 3an
p
2
ai = e x i , i = 1, 2, . . . , n,
f (x 1 ) + f (x 2 ) + · · · + f (x n ) ≥ n f (s),
where
x1 + x2 + · · · + x n
x 1 ≤ · · · ≤ x n−1 ≤ 0 ≤ x n , s= = 0,
n
1
f (u) = p , u ∈ R.
1 + 3eu
For u ≥ 0, we have
3eu (3eu − 2)
f 00 (u) = > 0,
4(1 + 3eu )5/2
HCF Method for Ordered Variables 197
hence f is convex on [s, ∞). According to the RHCF-OV Theorem (case m = n−1),
it suffices to show that f (x) + f ( y) ≥ 2 f (0) for all real x, y so that x + y = 0; that
is, to show that
1 1
p +p ≥1
1 + 3a 1 + 3b
for all a, b > 0 so that ab = 1. Write this inequality as
s
1 a
p + ≥ 1.
1 + 3a a+3
1
Substituting p = t, 0 < t < 1, the inequality becomes
1 + 3a
v
t 1 − t2
≥ 1 − t.
8t 2 + 1
By squaring, we get
t(1 − t)(2t − 1)2 ≥ 0,
which is true. The equality holds for a1 = a2 = · · · = an = 1.
a1 ≤ · · · ≤ an−1 ≤ 1 ≤ an , a1 a2 · · · an = 1.
1
If 0 < m < 1 and 0 < k ≤ , then
21/m −1
1 1 1 n
+ + ··· + ≥ .
(a1 + k) m (a2 + k) m (an + k) m (1 + k)m
f (x 1 ) + f (x 2 ) + · · · + f (x n ) ≥ n f (s),
198 Vasile Cîrtoaje
where
x1 + x2 + · · · + x n
x 1 ≤ · · · ≤ x n−1 ≤ 0 ≤ x n , s= = 0,
n
1
f (u) = , u ∈ R.
(eu + k)m
For u ∈ [0, ∞), we have
meu (meu − k)
f 00 (u) = ≥ 0,
(eu + k)m+2
hence f is convex on [s, ∞). According to the RHCF-OV Theorem (case m = n−1),
it suffices to show that f (x) + f ( y) ≥ 2 f (0) for all real x, y so that x + y = 0; that
is, to show that
1 1 2
+ ≥
(a + k)m (b + k)m (1 + k)m
for all a, b > 0 so that ab = 1. Write this inequality as g(a) ≥ 0 for a ≥ 1, where
1 am 2
g(a) = + − .
(a + k) m (ka + 1) m (1 + k)m
The derivative
g 0 (a) a m−1 (a + k)m+1 − (ka + 1)m+1
=
m (a + k)m+1 (ka + 1)m+1
has the same sign as the function
We have
where m
h1 (a) = (m − 1)(a2 + 1) − 2 k − a.
k
The discriminant D of the quadratic function h1 (a) is
2
m 2
D m
= k− − (m − 1)2 = (1 − k2 ) − 1 .
4 k k2
Since D > 0, the roots a1 and a2 of h1 (a) are real and unequal. If a1 < a2 , then
h1 (a) ≥ 0 for a ∈ [a1 , a2 ] and h1 (a) ≤ 0 for a ∈ (−∞, a1 ] ∪ [a2 , ∞). Since
2(k + 1)(m − k)
h1 (1) = > 0,
k
HCF Method for Ordered Variables 199
it follows that a1 < 1 < a2 , therefore h1 (a) and h0 (a) are positive for a ∈ [1, a2 )
and negative for a ∈ (a2 , ∞), h is increasing on [1, a2 ] and decreasing on [a2 , ∞).
From h(1) = 0 and
lim h(a) = −∞,
a→∞
it follows that there is a3 > a2 so that h(a) and g 0 (a) are positive for a ∈ (1, a3 ) and
negative for a ∈ (a3 , ∞). As a result, g is increasing on [1, a3 ] and decreasing on
[a3 , ∞). Since g(1) = 0 and
1 2
lim g(a) = − ≥ 0,
a→∞ k m (1 + k)m
it follows that g(a) ≥ 0 for a ≥ 1. This completes the proof. The equality holds for
a1 = a2 = · · · = an = 1.
1 1
Remark. For k = and m = , we get the preceding P 2.28.
3 2
a1 ≥ a2 ≥ a3 ≥ 1 ≥ a4 ≥ · · · ≥ an , a1 a2 · · · an = 1,
then
1 1 1 n
+ + ··· + ≥ .
3a1 + 1 3a2 + 1 3an + 1 4
(Vasile C., 2007)
ai = e x i , i = 1, 2, . . . , n,
f (x 1 ) + f (x 2 ) + · · · + f (x n ) ≥ n f (s),
where
x1 + x2 + · · · + x n
x1 ≥ x2 ≥ x3 ≥ 0 ≥ x4 ≥ · · · ≥ x n, s= = 0,
n
1
f (u) = , u ∈ R.
3eu+1
For u ∈ [0, ∞), we have
3eu (3eu − 1)
f 00 (u) = > 0,
(3eu + 1)3
200 Vasile Cîrtoaje
hence f is convex on [s, ∞). According to the RHCF-OV Theorem (case m = n−3),
it suffices to show that f (x) + 3 f ( y) ≥ 4 f (0) for all real x, y so that x + 3 y = 0;
that is, to show that
1 3
+ ≥1
3a + 1 3b + 1
for all a, b > 0 so that ab3 = 1. The inequality is equivalent to
b3 3
+ ≥ 1,
b + 3 3b + 1
3
(b − 1)2 (b + 2) ≥ 0.
The equality holds for a1 = a2 = · · · = an = 1.
a1 ≥ a2 ≥ a3 ≥ 1 ≥ a4 ≥ · · · ≥ an , a1 a2 · · · an = 1,
then
1 1 1 n
+ + ··· + ≥ .
(a1 + 1)2 (a2 + 1)2 (an + 1)2 4
(Vasile C., 2007)
Solution. Using the substitution
ai = e x i , i = 1, 2, . . . , n,
f (x 1 ) + f (x 2 ) + · · · + f (x n ) ≥ n f (s),
where
x1 + x2 + · · · + x n
x1 ≥ x2 ≥ x3 ≥ 0 ≥ x4 ≥ · · · ≥ x n, s= = 0,
n
1
f (u) = , u ∈ R.
(eu + 1)2
For u ∈ [0, ∞), we have
2eu (2eu − 1)
f 00 (u) = > 0,
(eu + 1)4
hence f is convex on [s, ∞). According to the RHCF-OV Theorem (case m = 3), it
suffices to show that f (x) + 3 f ( y) ≥ 4 f (0) for all real x, y so that x + 3 y = 0; that
is, to show that
1 3
+ ≥1
(a + 1)2 (b + 1)2
HCF Method for Ordered Variables 201
b6 3
+ ≥ 1.
(b + 1)
3 2 (b + 1)2
Using the Cauchy-Schwarz inequality, it suffices to show that
(b3 + 3)2
≥ 1,
(b3 + 1)2 + 3(b + 1)2
which is equivalent to the obvious inequality
(b − 1)2 (4b + 5) ≥ 0.
a1 ≥ · · · ≥ an−1 ≥ 1 ≥ an , a1 a2 · · · an = 1,
then
1 1 1 n
+ + · · · + ≤ .
(a1 + 3)2 (a2 + 3)2 (an + 3)2 16
(Vasile C., 2007)
Solution. Using the substitution
ai = e x i , i = 1, 2, . . . , n,
f (x 1 ) + f (x 2 ) + · · · + f (x n ) ≥ n f (s),
where
x1 + x2 + · · · + x n
x 1 ≥ · · · ≥ x n−1 ≥ 0 ≥ x n , s= = 0,
n
−1
f (u) = , u ∈ R.
(eu + 3)2
For u ∈ (−∞, 0], we have
2eu (3 − 2eu )
f 00 (u) = > 0,
(eu + 3)4
hence f is convex on (−∞, s]. According to the LHCF-OV Theorem (case m =
n−1), it suffices to show that f (x)+ f ( y) ≥ 2 f (0) for all real x, y so that x + y = 0;
that is, to show that
1 1 1
+ ≤
(a + 3)2 (b + 3)2 8
202 Vasile Cîrtoaje
b2 1 1
+ ≤ ,
(3b + 1)2 (b + 3)2 8
a1 ≥ · · · ≥ an−1 ≥ 1 ≥ an , a1 a2 · · · an = 1,
p
If k ≥ 1 + 2, then
1 1 1 n
+ + · · · + ≤ ,
(a1 + k)2 (a2 + k)2 (an + k)2 (1 + k)2
a1 ≥ · · · ≥ an−1 ≥ 1 ≥ an , a1 a2 · · · an = 1.
If p, q ≥ 0 so that p + q ≤ 1, then
1 1 1 n
+ + ··· + ≤ .
1 + pa1 + qa1 1 + pa2 + qa2
2 2
1 + pan + qan2 1+p+q
ai = e x i , i = 1, 2, . . . , n,
f (x 1 ) + f (x 2 ) + · · · + f (x n ) ≥ n f (s),
where
x1 + x2 + · · · + x n
x 1 ≥ · · · ≥ x n−1 ≥ 0 ≥ x n , s= = 0,
n
HCF Method for Ordered Variables 203
−1
f (u) = , u ∈ R.
1 + peu + qe2u
For u ≤ 0, we have
1 1 2
+ ≤
1 + pa + qa 2 1 + pb + qb 2 1+p+q
a1 ≥ · · · ≥ an−1 ≥ 1 ≥ an , a1 a2 · · · an = 1.
1
If m > 1 and k ≥ , then
21/m −1
1 1 1 n
+ + · · · + ≤ .
(a1 + k)m (a2 + k)m (an + k)m (1 + k)m
1
21/m < 1 + ,
m
204 Vasile Cîrtoaje
hence
1
k≥ > m > 1.
21/m − 1
Using the substitution
ai = e x i , i = 1, 2, . . . , n,
we can write the inequality as
f (x 1 ) + f (x 2 ) + · · · + f (x n ) ≥ n f (s),
where
x1 + x2 + · · · + x n
x 1 ≥ · · · ≥ x n−1 ≥ 0 ≥ x n , s= = 0,
n
−1
f (u) = , u ∈ R.
(eu + k)m
For u ≤ 0, we have
meu (k − meu )
f 00 (u) = ≥ 0,
(eu + k)m+2
hence f is convex u ≤ s. By the LHCF-OV Theorem (case m = n − 1), it suffices to
show that f (x) + f ( y) ≥ 2 f (0) for all real x, y so that x + y = 0; that is, to show
that
1 1 2
+ ≤
(a + k) m (b + k) m (1 + k)m
for all a, b > 0 so that ab = 1. Write this inequality as g(a) ≤ 0 for a ≥ 1, where
1 am 2
g(a) = + − .
(a + k)m (ka + 1)m (1 + k)m
The derivative
g 0 (a) a m−1 (a + k)m+1 − (ka + 1)m+1
=
m (a + k)m+1 (ka + 1)m+1
has the same sign as the function
We have
m−1 1 kh1 (a)
k
h (a) =
0
+ (m + 1) − = ,
a a + k ka + 1 a(a + k)(ka + 1)
where m
h1 (a) = (m − 1)(a2 + 1) − 2 k − a.
k
The discriminant D of the quadratic function h1 (a) is
m 2 m2
D
= k− − (m − 1) = (k − 1) 1 − 2 .
2 2
4 k k
HCF Method for Ordered Variables 205
Since D > 0, the roots a1 and a2 of h1 (a) are real and unequal. If a1 < a2 , then
h1 (a) ≤ 0 for a ∈ [a1 , a2 ] and h1 (a) ≥ 0 for a ∈ (−∞, a1 ] ∪ [a2 , ∞). Since
2(k + 1)(m − k)
h1 (1) = < 0,
k
it follows that a1 < 1 < a2 , therefore h1 (a) and h0 (a) are negative for a ∈ [1, a2 )
and positive for a ∈ (a2 , ∞), h(a) is decreasing for a ∈ [1, a2 ] and increasing for
a ∈ [a2 , ∞). From h(1) = 0 and
lim h(a) = ∞,
a→∞
it follows that there is a3 > a2 so that h(a) and g 0 (a) are negative for a ∈ (1, a3 )
and positive for a ∈ (a3 , ∞). As a result, g is decreasing on [1, a3 ] and increasing
on [a3 , ∞). Since g(1) = 0 and
1 2
lim g(a) = − ≤ 0,
a→∞ k m (1 + k)m
it follows that g(a) ≤ 0 for a ≥ 1. This completes the proof. The equality holds for
a1 = a2 = · · · = an = 1.
a1 ≥ · · · ≥ an−1 ≥ 1 ≥ an , a1 a2 · · · an = 1,
then
1 1 1 n
+p + ··· + p ≤p .
1 + 2a1 1 + 2a2 1 + 2an
p
3
(Vasile C., 2007)
ai = e x i , i = 1, 2, . . . , n,
f (x 1 ) + f (x 2 ) + · · · + f (x n ) ≥ n f (s),
where
x1 + x2 + · · · + x n
x 1 ≥ · · · ≥ x n−1 ≥ 0 ≥ x n , s= = 0,
n
−1
f (u) = p , u ∈ R.
1 + 2eu
206 Vasile Cîrtoaje
For u ≤ 0, we have
eu (1 − eu )
f 00 (u) = > 0,
(1 + 2eu )5/2
hence f is convex on (−∞, s]. According to the LHCF-OV Theorem (case m =
n−1), it suffices to show that f (x)+ f ( y) ≥ 2 f (0) for all real x, y so that x + y = 0;
that is, to show that v v
t 3 t 3
+ ≤2
1 + 2a 1 + 2b
for all a, b > 0 so that ab = 1. By the Cauchy-Schwarz inequality, we get
v v v
3 t 3 3 3
t t
+ ≤ +1 1+ = 2.
1 + 2a 1 + 2b 1 + 2a 1 + 2b
a1 ≥ · · · ≥ an−1 ≥ 1 ≥ an , a1 a2 · · · an = 1.
1 1 1 n
+ + · · · + ≤ .
(a1 + k)m (a2 + k)m (an + k)m (1 + k)m
ai = e x i , i = 1, 2, . . . , n,
f (x 1 ) + f (x 2 ) + · · · + f (x n ) ≥ n f (s),
where
x1 + x2 + · · · + x n
x 1 ≥ · · · ≥ x n−1 ≥ 0 ≥ x n , s= = 0,
n
−1
f (u) = , u ∈ R.
(eu + k)m
For u ≤ 0, we have
meu (k − meu )
f 00 (u) = ≥ 0,
(eu + k)m+2
HCF Method for Ordered Variables 207
then
1 1 1 n
+ + ··· + ≤ .
3 − a1 3 − a2 3 − an 2
(Vasile C., 2007)
Solution. From
we get p
a1 ≤ 2.
p p p
Replacing a1 , a2 , . . . , an by a1 , a2 , . . . , an , we have to prove that
where
a1 + a2 + · · · + an
2 ≥ a1 ≥ · · · ≥ an−1 ≥ 1 ≥ an , s= = 1,
n
1
f (u) = p , u ∈ [0, 2].
u−3
For u ∈ [0, 1], we have
p
3(1 − u)
f (u) = p
00
p ≥ 0.
4u u(3 − u)3
Therefore, f is convex on [0, s]. According to the LHCF-OV Theorem and Note 1
(case m = n − 1), it suffices to show that h(x, y) ≥ 0 for x, y ≥ 0 so that x + y = 2.
Since
f (u) − f (1) −1
g(u) = = p p
u−1 2(3 − u)(1 + u)
and
p p
g(x) − g( y) 2− x − y
h(x, y) = = p p p p p p ,
x−y 2( x + y)(1 + x)(1 + y)(3 − x)(3 − y)
a1 ≤ · · · ≤ an−1 ≤ 1 ≤ an , a1 + a2 + · · · + an = n.
Prove that
n − a1 3 n − a2 3 n − a 3
n
a13 + a23 + ··· + an3 − n ≥ (n − 1) 2
+ + ··· + −n .
n−1 n−1 n−1
a1 ≤ 1, a2 = · · · = an−1 = 1, a n = 2 − a1 .
Chapter 3
a1 + a2 + · · · + an = ns
if and only if
f (x) + (n − 1) f ( y) ≥ n f (s)
for all x, y ∈ I so that x ≤ s ≤ y and x + (n − 1) y = ns.
Proof. For
a1 = x, a2 = a3 = · · · = an = y,
the inequality
f (a1 ) + f (a2 ) + · · · + f (an ) ≥ f (s)
becomes
f (x) + (n − 1) f ( y) ≥ n f (s);
therefore, the necessity is obvious.
The proof of sufficiency is based on Lemma below. According to this lemma, it
suffices to consider that a1 , a2 , . . . , an ∈ J, where
J = I≤s0 .
211
212 Vasile Cîrtoaje
Because f (u) is convex on J≥s , the desired inequality follows from the RHCF The-
orem (see Chapter 1) applied to the interval J.
Lemma. Let f be a real function defined on an interval I. In addition, f is decreasing
on I≤s0 , and f (u) ≥ f (s0 ) for u ∈ I, where s, s0 ∈ I, s < s0 . If the inequality
holds for all a1 , a2 , . . . , an ∈ I≤s0 so that a1 + a2 + · · · + an = ns, then it holds for all
a1 , a2 , . . . , an ∈ I so that a1 + a2 + · · · + an = ns.
Proof. For i = 1, 2, . . . , n, define the numbers
¨
ai , ai ≤ s0
bi =
s0 , ai > s0 .
Clearly, bi ∈ I≤s0 and bi ≤ ai . Since f (u) ≥ f (s0 ) for u ∈ I≥s0 , it follows that
f (bi ) ≤ f (ai ) for i = 1, 2, . . . , n. Therefore,
b1 + b2 + · · · + bn ≤ a1 + a2 + · · · + an = ns
and
f (b1 ) + f (b2 ) + · · · + f (bn ) ≤ f (a1 ) + f (a2 ) + · · · + f (an ).
Thus, it suffices to show that
a1 + a2 + · · · + an = ns
if and only if
f (x) + (n − 1) f ( y) ≥ n f (s)
for all x, y ∈ I so that x ≥ s ≥ y and x + (n − 1) y = ns.
Partially Convex Function Method 213
From the RPCF-Theorem and the LPCF-Theorem, we find the PCF-Theorem (Par-
tially Convex Function Theorem).
Partially Convex Function Theorem (Vasile Cîrtoaje, 2012). Let f be a real function
defined on an interval I and convex on [s0 , s] or [s, s0 ], where s0 , s ∈ I. In addition, f
is decreasing on I≤s0 and increasing on I≥s0 . The inequality
a + a + ··· + a
1 2 n
f (a1 ) + f (a2 ) + · · · + f (an ) ≥ n f
n
holds for all a1 , a2 , . . . , an ∈ I satisfying
a1 + a2 + · · · + an = ns
if and only if
f (x) + (n − 1) f ( y) ≥ n f (s)
for all x, y ∈ I so that x + (n − 1) y = ns.
f (x) + (n − 1) f ( y) ≥ n f (s),
by the condition
h(x, y) ≥ 0 for all x, y ∈ I so that x + (n − 1) y = ns.
f 0 (x) − f 0 ( y)
H(x, y) = .
x−y
As shown in Note 2 from Chapter 1, the inequalities in the RPCF-Theorem and the
LPCF-Theorem hold true by replacing the hypothesis
f (x) + (n − 1) f ( y) ≥ n f (s)
Note 3. The desired inequalities in the RPCF-Theorem and the LPCF-Theorem be-
come equalities for
a1 = a2 = · · · = an = s.
214 Vasile Cîrtoaje
a1 = x, a2 = · · · = an = y
(or any cyclic permutation). Notice that these equality conditions are equivalent to
x + (n − 1) y = ns, h(x, y) = 0
Note 4. From the proof of the RPCF-Theorem, it follows that this theorem is also
valid in the case when f is defined on I \ {u0 }, where u0 ∈ I>s0 . Similarly, the LPCF-
Theorem is also valid in the case when f is defined on I \ {u0 }, where u0 ∈ I<s0 .
where
s, s0 ∈ I, s < s0 , ns − (n − 1)s0 ≤ inf I.
If
f (x) + (n − 1) f ( y) ≥ n f (s)
for all x, y ∈ I so that x ≤ s ≤ y and x + (n − 1) y = ns, then
x + x + ··· + x
1 2 n
f (x 1 ) + f (x 2 ) + · · · + f (x n ) ≥ n f
n
for all x 1 , x 2 , . . . , x n ∈ I satisfying x 1 + x 2 + · · · + x n = ns .
In order to prove Theorem 1, we define the function
f (u), u ≤ s0 , u ∈ I
¨
f0 (u) =
f (s0 ), u ≥ s0 , u ∈ I,
which is convex on I≥s . Taking into account that f0 (s) = f (s) and f0 (u) ≤ f (u) for
all u ∈ I, it suffices to prove that
f0 (x 1 ) + f0 (x 2 ) + · · · + f0 (x n ) ≥ n f0 (s)
Partially Convex Function Method 215
f0 (x) + (n − 1) f0 ( y) ≥ n f0 (s)
ns − (n − 1)s0 ≥ sup I.
More precisely, the following theorem holds:
Theorem 2. Let f be a function defined on a real interval I, convex on [s0 , s] and
satisfying
min f (u) = f (s0 ),
u∈I≤s
where
s, s0 ∈ I, s > s0 , ns − (n − 1)s0 ≥ sup I.
If
f (x) + (n − 1) f ( y) ≥ n f (s)
for all x, y ∈ I so that x ≥ s ≥ y and x + (n − 1) y = ns, then
x + x + ··· + x
1 2 n
f (x 1 ) + f (x 2 ) + · · · + f (x n ) ≥ n f
n
for all x 1 , x 2 , . . . , x n ∈ I satisfying x 1 + x 2 + · · · + x n = ns.
The proof of Theorem 2 is similar to the proof of Theorem 1.
Note 6. From the proof of Theorem 1, it follows that Theorem 1 is also valid in
the case in which f is defined on I \ {u0 }, where u0 is an interior point of I so that
u0 ∈/ [s, s0 ]. Similarly, Theorem 2 is also valid in the case in which f is defined on
/ [s0 , s].
I \ {u0 }, where u0 is an interior point of I so that u0 ∈
Note 7. In the same manner, we can extend weighted Jensen’s inequality to right
and left partially convex functions establishing the WRPCF-Theorem, the WLPCF-
Theorem and the WPCF-Theorem (Vasile Cîrtoaje, 2014).
216 Vasile Cîrtoaje
p1 + p2 + · · · + pn = 1, p = min{p1 , p2 , . . . , pn },
and let f be a real function defined on an interval I and convex on [s, s0 ], where
s, s0 ∈ I, s < s0 . In addition, f is decreasing on I≤s0 and f (u) ≥ f (s0 ) for u ∈ I. The
inequality
p1 a1 + p2 a2 + · · · + pn an = s,
if and only if
p f (x) + (1 − p) f ( y) ≥ f (s)
for all x, y ∈ I so that x ≤ s ≤ y and px + (1 − p) y = s.
p1 + p2 + · · · + pn = 1, p = min{p1 , p2 , . . . , pn },
and let f be a real function defined on an interval I and convex on [s0 , s], where
s0 , s ∈ I, s0 < s. In addition, f is increasing on I≥s0 and f (u) ≥ f (s0 ) for u ∈ I. The
inequality
p1 a1 + p2 a2 + · · · + pn an = s,
if and only if
p f (x) + (1 − p) f ( y) ≥ f (s)
for all x, y ∈ I so that x ≥ s ≥ y and px + (1 − p) y = s.
Partially Convex Function Method 217
3.2 Applications
5a − 1 5b − 1 5c − 1 5d − 1 5e − 1 5f − 1
+ + + + + ≤ 4.
5a2 + 1 5b2 + 1 5c 2 + 1 5d 2 + 1 5e2 + 1 5 f 2 + 1
1 1 1 3
+ 2 + 2 ≤ .
9a2 − 10a + 9 9b − 10b + 9 9c − 10c + 9 8
1 1 1 1 4
+ 2 + 2 + 2 ≤ .
4a2 − 5a + 4 4b − 5b + 4 4c − 5c + 4 4d − 5d + 4 3
218 Vasile Cîrtoaje
1 25
k∈ , ,
6 14
then X 1 5
≤ .
ka12 + a2 + a3 + a4 + a5 k+4
5 2 5 2 5 2 5 2 5 2
7− + 7− + 7− + 7− + 7− ≥ 20.
a b c d e
220 Vasile Cîrtoaje
n2
3.20. Let a1 , a2 , . . . , an be real numbers so that a1 +a2 +· · ·+an = n. If k ≥ ,
4(n − 1)
then
(a12 + k)(a22 + k) · · · (an2 + k) ≥ (1 + k)n .
(1 − a + a4 )(1 − b + b4 )(1 − c + c 4 ) ≥ 1.
1 1 1 1 1 1 1 1 1 1
5 2 + 2 + 2 + 2 + 2 + 45 ≥ 14 + + + + .
a b c d e a b c d e
7 − 6a 7 − 6b 7 − 6c
+ + ≥ 1.
2 + a2 2 + b2 2 + c 2
Partially Convex Function Method 221
1 1 1 1
+ + ≤ .
a + 5bc b + 5ca c + 5ab 2
1 1 1 3
+ + ≤ .
4 − 3a + 4a 2 4 − 3b + 4b 2 4 − 3c + 4c 2 5
1 1 1 3
+ + ≤ .
(3a + 1)(3a2 − 5a + 3) (3b + 1)(3b2 − 5b + 3) (3c + 1)(3c 2 − 5c + 3) 4
1 − a1 1 − a2 1 − an
+ + ··· + ≥ 0.
1 + pa1 + qa1 1 + pa2 + qa2
2 2
1 + pan + qan2
1−a 1− b 1−c
+ + ≥ 0.
17 + 4a + 6a 2 17 + 4b + 6b 2 17 + 4c + 6c 2
1 − a1 1 − a2 1 − a8
+ + · · · + ≥ 0.
(1 + a1 )2 (1 + a2 )2 (1 + a8 )2
−13 13
3.33. Let a, b, c be positive real numbers so that abc = 1. If k ∈ p , p ,
3 3 3 3
then
a+k b+k c+k 3(1 + k)
+ + ≤ .
a2 + 1 b2 + 1 c 2 + 1 2
222 Vasile Cîrtoaje
p
3.34. If a, b, c are positive real numbers and 0 < k ≤ 2 + 2 2, then
a3 b3 c3 a+b+c
+ + ≥ .
ka + bc kb + ca kc + ab
2 2 2 k+1
a1 , a2 , . . . , an ≤ π, a1 + a2 + · · · + an = π,
then
π
cos a1 + cos a2 + · · · + cos an ≤ n cos .
n
−(3n − 2)
a1 , a2 , . . . , an ≥ , a1 + a2 + · · · + an = n,
n−2
then
1 − a1 1 − a2 1 − an
+ + · · · + ≥ 0.
(1 + a1 )2 (1 + a2 )2 (1 + an )2
3.3 Solutions
a+b+c
f (a) + f (b) + f (c) ≥ 3 f (s), s= = 1,
3
where
5 − 16u
f (u) = , u ∈ R.
32u2 + 1
From
16(32u2 − 20u − 1)
f 0 (u) = ,
(32u2 + 1)2
it follows that f is increasing on
p
5 − 33
−∞, ∪ [s0 , ∞)
16
and decreasing on p
5 − 33
, s0 ,
16
where p
5 + 33
s0 = ≈ 0.6715.
16
Also, from
lim f (u) = 0
u→−∞
and
f (s0 ) < 0,
it follows that f (u) ≥ f (s0 ) for u ∈ R. In addition, for u ∈ [s0 , 1], we have
hence f is convex on [s0 , s]. According to the LPCF-Theorem, we only need to show
that f (x) + 2 f ( y) ≥ 3 f (1) for all real x, y so that x + 2 y = 3. Using Note 1, it
suffices to prove that h(x, y) ≥ 0, where
a+b+c+d
f (a) + f (b) + f (c) + f (d) ≥ 4 f (s), s= = 1,
4
where
5 − 18u
f (u) = , u ∈ R.
12u2 + 1
From
6(36u2 − 20u − 3)
f 0 (u) = ,
(12u2 + 1)2
it follows that f is increasing on
p
5 − 52
−∞, ∪ [s0 , ∞)
18
Partially Convex Function Method 227
and decreasing on
p p
5 + 52
5 − 52
, s0 , s0 = ≈ 0.678.
18 18
Also, from
lim f (u) = 0
u→−∞
and
f (s0 ) < 0,
it follows that f (u) ≥ f (s0 ) for u ∈ R. In addition, for u ∈ [s0 , 1], we have
hence f is convex on [s0 , s]. According to the LPCF-Theorem and Note 1, we only
need to show that h(x, y) ≥ 0 for x, y ∈ R so that x + 3 y = 4. We have
5a − 1 5b − 1 5c − 1 5d − 1 5e − 1 5f − 1
+ + + + + ≤ 4.
5a2 + 1 5b2 + 1 5c 2 + 1 5d 2 + 1 5e2 + 1 5 f 2 + 1
a+b+c+d+e+ f
f (a) + f (b) + f (c) + f (d) + f (e) + f ( f ) ≥ 4 f (s), s= = 1,
6
228 Vasile Cîrtoaje
where
1 − 5u
f (u) = , u ∈ R.
5u2 + 1
From
5(5u2 − 2u − 1)
f (u) =
0
,
(5u2 + 1)2
it follows that f is increasing on
p
1− 6
−∞, ∪ [s0 , ∞)
5
and decreasing on
p p
1+ 6
1− 6
, s0 , s0 = ≈ 0.69.
5 5
Also, from
lim f (u) = 0
u→−∞
and
f (s0 ) < 0,
it follows that f (u) ≥ f (s0 ) for u ∈ R. In addition, for u ∈ [s0 , 1], we have
hence f is convex on [s0 , s]. According to the LPCF-Theorem and Note 1, we only
need to show that h(x, y) ≥ 0 for x, y ∈ R so that x + 5 y = 6. We have
n(n − 2)ai2 + 2(n − 1)ai + n ≥ (n − 1)ai2 + 2(n − 1)ai + n > (n − 1)(ai + 1)2 ≥ 0.
f1 (u) = 2(n − 2)u3 − 3n(n + 1)(n − 2)u2 − 2n2 (2n − 3)u + n3 (n + 1).
therefore,
min f (u) = f (s0 ).
u∈I
hence f 00 (u) > 0. Since [s0 , s] ⊂ [−1, 1], f is convex on [s0 , s]. By the LPCF-
Theorem and Note 1, we only need to show that h(x, y) ≥ 0 for x, y ∈ R and
x + (n − 1) y = n, where
Indeed, we have
(n − 2)u + n
g(u) =
(n − 2)u2 + n2
and
h(x, y) n2 − n(x + y) − (n − 2)x y
=
n−2 [(n − 2)x 2 + n2 ][(n − 2) y 2 + n2 ]
(n − 1)(n − 2) y 2
= ≥ 0.
[(n − 2)x 2 + n2 ][(n − 2) y 2 + n2 ]
a+b+c+d
f (a) + f (b) + f (c) + f (d) ≥ 4 f (s), s= = 1,
4
where
u2 − u
f (u) = , u ∈ R.
3u2 + 4
Partially Convex Function Method 231
From
3u2 + 8u − 4
f 0 (u) = ,
(3u2 + 4)2
p
−4 − 2 7
it follows that f is increasing on −∞, ∪ [s0 , ∞) and decreasing on
3
p
−4 − 2 7
, s0 , where
3
p
−4 + 2 7
s0 = ≈ 0.43.
3
Since
1
lim f (u) =
u→−∞ 3
and f (s0 ) < 0, it follows that
g(x) − g( y) 4 − 3x y
h(x, y) = =
x−y (3x + 4)(3 y 2 + 4)
2
(x − 2)2
= ≥ 0.
(3x 2 + 4)(3 y 2 + 4)
The proof is completed. From x + 3 y = 4 and h(x, y) = 0, we get x = 2 and
y = 2/3. By Note 3, the equality holds for a = b = c = d = 1, and also for
2
a = 2, b=c=d=
3
(or any cyclic permutation).
Remark. In the same manner, we can prove the following generalization:
• If a1 , a2 , . . . , an are real numbers so that a1 + a2 + · · · + an = n, then
a1 (a1 − 1) a2 (a2 − 1) an (an − 1)
+ + ··· + ≥ 0,
4(n − 1)a1 + n
2 2 4(n − 1)a2 + n
2 2 4(n − 1)an2 + n2
232 Vasile Cîrtoaje
a+b+c+d
f (a) + f (b) + f (c) + f (d) ≥ 4 f (s), s= = 1,
4
where
−1
f (u) = , u ∈ R.
4u2 − 5u + 4
From
2(8u − 5)
f 0 (u) = ,
(4u2 − 5u + 4)2
it follows that f is decreasing on (−∞, s0 ] and increasing on [s0 , ∞), where
5
s0 = < 1 = s.
8
For u ∈ [s0 , s] = [5/8, 1], we have
f (u) − f (1) 4u − 1)
g(u) = = ,
u−1 3(4u2 − 5u + 4)
234 Vasile Cîrtoaje
7 3
x= , y= .
4 4
In accord with Note 3, the equality holds for a = b = c = 1, and also for
7 3
a= , b=c=d=
4 4
2(n − 1)
k =1− ,
n2
then
1 1 1 n
+ 2 + ··· + 2 ≥ ,
a12 − 2ka1 + 1 a2 − 2ka2 + 1 an − 2kan + 1 2(1 − k)
with equality for a1 = a2 = · · · = an = 1, and also for
3n2 − 6n + 4 n2 − 2n + 4
a1 = , a2 = a3 = · · · = an =
n2 n2
n
k≥ p .
2 n−1
Then,
a1 (a1 − 1) a2 (a2 − 1) an (an − 1)
+ + ··· + ≥ 0.
(a1 + k) 2 (a2 + k)2 (an + k)2
hence f is convex on [s0 , 1]. According to the LPCF-Theorem, Note 4 and Note 1,
it suffices to show that h(x, y) ≥ 0 for x, y ∈ I which satisfy x + (n − 1) y = n. We
have
f (u) − f (1) u+1
g(u) = = ,
u−1 (u + k)2
g(x) − g( y) (k − 1)2 − 1 − x − y − x y
h(x, y) = = ≥ 0,
x−y (x + k)2 ( y + k)2
since
n2 [(n − 1) y − 1]2
(k − 1)2 − 1 − x − y − x y ≥ −1− x − y − xy = ≥ 0.
n−1 n−1
Partially Convex Function Method 237
n
The equality holds for a1 = a2 = · · · = an = 1. If k = 1 + p , then the equality
n−1
holds also for
1
a1 = n − 1, a2 = · · · = an =
n−1
(or any cyclic permutation).
1 25
k∈ , ,
6 14
then X 1 5
≤ .
ka12 + a2 + a3 + a4 + a5 k+4
(Vasile C., 2006)
a1 + a2 + a3 + a4 + a5 = 5,
For
1 25
≤k≤ ,
2 14
we have
1
s0 = ≤ 1 = s,
2k
and for u ∈ [s0 , s], that is
1
≤ u ≤ 1,
2k
we have
(1 − u)(2ku − 1) ≥ 0,
−2ku2 ≥ (2k + 1)u + 1,
−2k2 u2 ≥ k(2k + 1)u + k,
therefore
3 −3k(2k − 1)u + 13k − 2
g(u) ≥ [k(2k + 1)u + k] + 3ku + 5k − 1 =
2 2
−3k(2k − 1) + 13k − 2
≥ = −3k2 + 8k − 1 = 3k(2 − k) + (2k − 1) > 0.
2
Consequently, f is convex on [s0 , s].
For
1 1
≤k≤ ,
6 2
we have
1
s0 = ≥ 1 = s,
2k
and for u ∈ [s, s0 ], that is
1
1≤u≤ ,
2k
we have
1 4 5
+ ≤
kx 2 − x +5 ky − y +5 k +4
2
for
x + 4 y = 5, x, y ∈ R.
Partially Convex Function Method 239
1 1 1 1
− +4 − ≥ 0,
k + 4 kx 2 − x + 5 k + 4 k y2 − y + 5
(x − 1)(kx + k − 1) 4( y − 1)(k y + k − 1)
+ ≥ 0.
kx 2 − x + 5 k y2 − y + 5
Since
4( y − 1) = 1 − x,
the inequality is equivalent to
kx + k − 1 ky + k −1
(x − 1) − ≥ 0,
kx 2 − x + 5 k y 2 − y + 5
1
The equality holds for a1 = a2 = a3 = a4 = a5 = 1. If k = , then the equality
6
holds also for
5
a1 = −5, a2 = a3 = a4 = a5 =
2
25
(or any cyclic permutation). If k = , then the equality holds also for
14
79 23
a1 = , a2 = a3 = a4 = a5 =
25 50
(or any cyclic permutation).
then X 1 n
≤ ,
ka12 + a2 + · · · + an k+n−1
with equality for a1 = a2 = · · · = an = 1. If k = k1 , then the equality holds also for
2n
a1 = −n, a2 = · · · = a n =
n−1
(or any cyclic permutation). If k = k2 , then the equality holds also for
(2k − 1)(n − 1) + 1 2k + n − 2
a1 = , a2 = · · · = an =
2k 2k(n − 1)
(or any cyclic permutation).
We have
where
3k(1 − k) −5k2 + 29k − 4
A= , B= .
4 4
Since B ≥ 0, it suffices to show that A − k2 y ≥ 0. Indeed, we have
(2k − 1)(n − 1) + 1 2k + n − 2
a1 = , a2 = · · · = an =
2k 2k(n − 1)
1 1 1
+ + ··· + ≤ 1.
a1k + a2 + · · · + an a1 + a2k + · · · + an a1 + a2 + · · · + ank
We will show that f is convex on [s0 , 1]. For u ∈ [s0 , 1], we have
where
g(u) = −k(k + 1)u2k−2 + k(k + 3)uk−1 − 2.
Denoting
t = kuk−1 , 1 ≤ t ≤ k,
Partially Convex Function Method 243
we get
1 n−1
+ k ≤1
xk −x +n y − y+n
yk − y + n
xk − x + n ≥ ,
yk − y + 1
(n − 1)( y − y k )
xk − x ≥ ,
yk − y + 1
xk − x y − yk
≥ .
x −1 (1 − y)( y k − y + 1)
xk − x
Let h(x) = , x > 1. By the weighted AM-GM inequality, we have
x −1
(k − 1)x k + 1 − kx k−1
h0 (x) = > 0.
(x − 1)2
x − 1 = (n − 1)(1 − y) ≥ 1 − y, x ≥ 2 − y > 1,
we get
(2 − y)k + y − 2
h(x) ≥ h(2 − y) = .
1− y
Thus, it suffices to show that
y − yk
(2 − y) + y − 2 ≥ k
k
,
y − y +1
which is equivalent to
1
(2 − y)k + y − 1 ≥ .
yk − y +1
1 + t ≥ 1 + t 2 + t(1 − t)k ,
x 1k − x 1 x 2k − x 2 x nk − x n
+ + ··· + ≥ 0.
x 1k + x 2 + · · · + x n x 1 + x 2k + · · · + x n x 1 + x 2 + · · · + x nk
x5 − x2 y5 − y2 z5 − z2
+ + ≥ 0.
x 5 + y 2 + z2 x 2 + y 5 + z2 x 2 + y 2 + z5
x1 x5 5
+ ··· + ≤ .
ksx 12 + x2 + x3 + x4 + x5 x 1 + x 2 + x 3 + x 4 + ksx 5
2
k+4
Since s ≥ 1, it suffices to prove the inequality for s = 1; that is, to show that
a1 a a5 5
+ 2 2 + ··· + 2 ≤
ka12 − a1 + 5 ka2 − a1 + 5 ka5 − an + 5 k + 4
for
a1 + a2 + a3 + a4 + a5 = 5.
Write the desired inequality as
where
a1 + a2 + a3 + a4 + a5
s= =1
5
and
−u
f (u) = , u ∈ [0, 5].
ku2 − u + 5
From
ku2 − 5
f 0 (u) = ,
(ku2 − u + 5)2
it follows that f is decreasing on [0, s0 ] and increasing on [s0 , 5], where
v
t5
s0 = .
k
We have
2g(u)
f 00 (u) = , g(u) = −k2 u3 + 15ku − 5, g 0 (u) = 3k(5 − ku2 ).
(u2 − u + 5)3
4
Case 1: ≤ k ≤ 5. We have
9
v
t5
s0 = ≥ 1 = s.
k
4 86k − 25
g(u) ≥ g(1) = −k + 15k − 5 = k −
2
(5 − k) + > 0.
9 9
Consequently, f 00 (u) > 0 for u ∈ [s0 , 1], hence f is convex on [s0 , s].
x 4y 5
+ ≤
kx 2 − x +5 ky − y +5 k +4
2
Partially Convex Function Method 247
for
x + 4 y = 5, x, y ≥ 0.
Write this inequality as follows:
1 1
x y
− +4 − ≥ 0,
k + 4 kx 2 − x + 5 k + 4 k y2 − y + 5
(x − 1)(kx − 5) 4( y − 1)(k y − 5)
+ ≥ 0.
kx 2 − x + 5 k y2 − y + 5
Since
4( y − 1) = 1 − x,
the inequality is equivalent to
kx − 5 ky −5
(x − 1) − ≥ 0,
kx 2 − x + 5 k y 2 − y + 5
(x − 1)2 h(x, y)
≥ 0,
(kx 2 − x + 5)(k y 2 − y + 5)
where
4 61 4
We need to show that h(x, y) ≥ 0 for k ∈ , . For k ∈ , 1 , we have
9 5 9
15k 5(9k − 4)
h(x, y) = (5 − 4 y) −k y +
2
+
4 4
kx(15 − 4k y) 5(9k − 4)
= +
4 4
kx(kx + 15 − 5k) 5(9k − 4)
= + ≥ 0,
4 4
61
while for k ∈ 1, , we have
5
5k + 15 2 (61 − 5k)(k − 1)
h(x, y) = 2k y − + ≥ 0.
4 16
4
The equality holds for a1 = a2 = a3 = a4 = a5 = 1. If k = , then the equality
9
holds also for
5
a1 = 0, a2 = a3 = a4 = a5 =
4
248 Vasile Cîrtoaje
61
(or any cyclic permutation). If k = , then the equality holds also for
5
115 95
a1 = , a2 = a3 = a4 = a5 =
61 122
(or any cyclic permutation).
n(k − n + 2) n(k + n − 2)
a1 = , a2 = · · · = a n =
2k 2k(n − 1)
Since s k−1 ≥ 1, it suffices to prove the inequality for s = 1; that is, to show that
a1 a2 an
+ + ··· + ≤1
a1k − a1 + n a2k − a2 + n ank − an + n
for
a1 + a2 + · · · + an = n.
Case 1: 1 < k ≤ n + 1. By Bernoulli’s inequality, we have
(k − 1)uk − n ≥ (k − 1)s0k − n = 0,
hence
Since f 00 (u) > 0, it follows that f is convex on [s0 , s]. By the LPCF-Theorem, we
need to show that
f (x) + (n − 1) f ( y) ≥ n f (1)
for
x ≥ 1 ≥ y ≥ 0, x + (n − 1) y = n.
Consider the nontrivial case where x > 1 > y ≥ 0 and write the required inequality
as follows:
x (n − 1) y
+ ≤ 1,
xk − x + n yk − y + n
x( y k − y + n)
xk − x + n ≥ ,
yk − ny + n
(n − 1) y( y − y k )
xk − x ≥ .
yk − ny + n
Since y < 1 and y k − n y + n > y k − y + 1, it suffices to show that
(n − 1)( y − y k )
xk − x ≥ ,
yk − y + 1
which has been proved at P 3.12.
The equality holds for a1 = a2 = · · · = an = 1.
Solution. Let
a1 + a2 + · · · + an
s= , s ≤ 1.
n
We have three cases to consider.
1
Case 1: s ≤ . The inequality is trivial because
n
ai ≤ a1 + a2 + · · · + an = ns ≤ 1
for i = 1, 2, . . . , n.
1
Case 2: < s < 1. Without loss of generality, assume that
n
a1 ≤ · · · ≤ a j < 1 ≤ a j+1 · · · ≤ an , j ∈ {1, 2, . . . , n}.
a1 ≤ b1 ≤ 1, . . . , a j ≤ b j ≤ 1, b j+1 = a j+1 , . . . , bn = an .
where
1−u
f (u) = , u ∈ [0, ns].
ku2 − u + ns
For u ∈ [0, 1], we have
Since
f (b1 ) + f (b2 ) + · · · + f (bn ) ≤ f (a1 ) + f (a2 ) + · · · + f (an ),
it suffices to show that f (b1 ) + f (b2 ) + · · · + f (bn ) ≥ 0 for b1 + b2 + · · · + bn = n.
This inequality is proved at Case 3.
Case 3: s = 1. Write the inequality as
a1 + a2 + · · · + an
f (a1 ) + f (a2 ) + · · · + f (an ) ≥ n f (s), s= = 1,
n
where
1−u
f (u) = , u ∈ [0, n].
ku2 −u+n
252 Vasile Cîrtoaje
From
k[(u − 1)2 − 1] − (n − 1)
f 0 (u) = ,
(ku2 − u + n)2
it follows that f is decreasing on [0, s0 ] and increasing on [s0 , n], where
v
t n−1
s0 = 1 + 1 + > 1 = s, s0 < n.
k
We will show that f is convex on [1, s0 ]. We have
2g(u)
f 00 (u) = ,
(ku2− u + n)3
where
1− x (n − 1)(1 − y) 1 1
+ = (x − 1) − 2 +
kx 2 − x + n k y2 − y + n kx − x + n k y 2 − y + n
(x − 1)(x − y)(kx + k y − 1)
=
(kx 2 − x + n)(k y 2 − y + n)
n(x − 1)2 (kx + k y − 1)
= ≥ 0,
(n − 1)(kx 2 − x + n)(k y 2 − y + n)
because
n−1 (n − 2)x
k(x + y) − 1 ≥ (x + y) − 1 = ≥ 0.
n n
1
The proof is completed. The equality holds for a1 = a2 = · · · = an = 1. If k = 1 − ,
n
then the equality holds also for
n
a1 = 0, a2 = a3 = · · · = an =
n−1
Partially Convex Function Method 253
s0 = 2 > s.
1− x (n − 1)(1 − y) 1 1
+ = (1 − x) −
1 − x + kx 2 1 − y + k y2 1 − x + kx 2 1 − y + k y 2
(1 − x)( y − x)(kx + k y − 1)
=
(1 − x + kx 2 )(1 − y + k y 2 )
n(x − 1)2 (kx + k y − 1)
= .
(n − 1)(1 − x + kx 2 )(1 − y + k y 2 )
Since
n−1 (n − 2)x
k(x + y) − 1 ≥ (x + y) − 1 = ≥ 0,
n n
1
the conclusion follows. The equality holds for a1 = a2 = · · · = an = 1. If k = 1 − ,
n
then the equality holds also for
n
a1 = 0, a2 = a3 = · · · = an =
n−1
(or any cyclic permutation).
p/a1 p/a2
1/p q/a1 q/a2
1/q
+ a2 + · · · + anp/an + a2 + · · · + anq/an
a1 a1
≥
n n
for all p ≥ q > 0. Thus, it suffices to prove the desired inequality for
n
k= , 1 < k ≤ 2.
n−1
Rewrite the desired inequality as
a1 + a2 + · · · + an
f (a1 ) + f (a2 ) + · · · + f (an ) ≥ n f (s), s= = 1,
n
where
f (u) = −uk/u , u ∈ I = (0, n).
Partially Convex Function Method 255
We have
k
f 0 (u) = ku u −2 (ln u − 1),
k
f 00 (u) = ku u −4 [u + (1 − ln u)(2u − k + k ln u)].
For n = 2, when k = 2 and I = (0, 2), f is convex on [1, 2) because
1 − ln u > 0, 2u − k + k ln u = 2u − 2 + 2 ln u ≥ 2u − 2 ≥ 0.
Therefore, we may apply the RHCF-Theorem. Consider now that n ≥ 3. From the
expression of f 0 , it follows that f is decreasing on (0, s0 ] and increasing on [s0 , n),
where
s0 = e > 1 = s.
In addition, we claim that f is convex on [1, s0 ]. Indeed, since
1 − ln u ≥ 0, 2u − k + k ln u ≥ 2 − k > 0,
x k/x + (n − 1) y k/ y ≤ n
for
0 < x ≤ 1 ≤ y, x + (n − 1) y = n.
We have
k
≥ k > 1.
x
Also, from
k n n k n 2
= > = 1, = ≤ ≤ 2,
y (n − 1) y x + (n − 1) y y (n − 1) y y
we get
k
0<
− 1 ≤ 1.
y
Therefore, by Bernoulli’s inequality, we have
1
x k/x + (n − 1) y k/ y − n = + (n − 1) y · y
k/ y−1
−n
1 k/x
x
1
k
≤ 1
+ (n − 1) y 1 + − 1 ( y − 1) − n
1+ k
x x −1 y
x2
= − (k − 1)x 2 − (2 − k)x
x 2 − kx + k
−x(x − 1)2 [(k − 1)x + k(2 − k)]
= ≤ 0.
x 2 − kx + k
The proof is completed. The equality holds for a1 = a2 = · · · = an = 1.
256 Vasile Cîrtoaje
5 2 5 2 5 2 5 2 5 2
7− + 7− + 7− + 7− + 7− ≥ 20.
a b c d e
a+b+c+d+e
f (a) + f (b) + f (c) + f (d) + f (e) ≥ 5 f (s), s= = 1,
5
where
5 2
f (u) = 7 − , u ∈ I = R \ {0}.
u
From
10(7u − 5)
f 0 (u) = ,
u3
it follows that f is increasing on (−∞, 0)∪[s0 , ∞) and decreasing on (0, s0 ], where
5
s0 = < 1 = s.
7
Since
lim f (u) = 49
u→−∞
10(15 − 14u)
f 00 (u) = > 0.
u4
According to the LPCF-Theorem and Note 4, we only need to show that
f (x) + 4 f ( y) ≥ 5 f (1)
for all nonzero real x, y so that x + 4 y = 5. Using Note 1, it suffices to prove that
h(x, y) ≥ 0, where
k 2 k 2 k 2
1− + 1− + ··· + 1 − ≥ n(1 − k)2 ,
a1 a2 an
a1 + a2 + · · · + a7
f (a1 ) + f (a2 ) + · · · + f (a7 ) ≥ 7 f (s), s= = 1,
7
where
f (u) = ln(u2 + 2), u ∈ R.
From
2u
f 0 (u) = ,
u2 + 2
it follows that f is decreasing on (−∞, s0 ] and increasing on [s0 , ∞], where
s0 = 0.
From
2(2 − u2 )
f 00 (u) = ,
(u2 + 2)2
258 Vasile Cîrtoaje
it follows that f 00 (u) > 0 for u ∈ [0, 1], therefore f is convex on [s0 , s]. By the
LPCF-Theorem, it suffices to prove that
f (x) + 6 f ( y) ≥ 7 f (1)
g( y) = ln [(7 − 6 y)2 + 2] + 6 ln ( y 2 + 2) − 7 ln 3, y ∈ R.
From
4(6 y − 7) 12 y
g 0 ( y) = +
12 y 2 − 28 y + 17 y 2 + 2
28(6 y 3 − 13 y 2 + 9 y − 2)
=
(12 y 2 − 28 y + 17)( y 2 + 2)
28(2 y − 1)(3 y − 2)( y − 1)
= ,
(12 y 2 − 28 y + 17)( y 2 + 2)
1 2 1 2
it follows that g is decreasing on −∞, ∪ , 1 and increasing on , ∪
2 3 2 3
[1, ∞); therefore,
g ≥ min{g(1/2), g(1)}.
Since g(1) = 0, we only need to show that g(1/2) ≥ 0; that is, to show that x = 4
and y = 1/2 involve
(x 2 + 2)( y 2 + 2)6 ≥ 37 .
Indeed, we have
37 139 · 37
(x + 2)( y + 2) − 3 = 3
2 2 6 7 7
− 1 = > 0.
211 211
n2
P 3.20. Let a1 , a2 , . . . , an be real numbers so that a1 +a2 +· · ·+an = n. If k ≥ ,
4(n − 1)
then
(a12 + k)(a22 + k) · · · (an2 + k) ≥ (1 + k)n .
a1 + a2 + · · · + an
f (a1 ) + f (a2 ) + · · · + f (an ) ≥ n f (s), s= = 1,
n
Partially Convex Function Method 259
where
f (u) = ln(u2 + k), u ∈ R.
From
2u
f 0 (u) =
,
+k u2
it follows that f is decreasing on (−∞, s0 ] and increasing on [s0 , ∞], where
s0 = 0.
From
2(k − u2 )
f 00 (u) = ,
(u2 + k)2
it follows that f 00 (u) ≥ 0 for u ∈ [0, 1], therefore f is convex on [s0 , s]. By the
LPCF-Theorem and Note 2, it suffices to prove that H(x, y) ≥ 0 for x, y ∈ R so that
x + (n − 1) y = n, where
f 0 (x) − f 0 ( y)
H(x, y) = .
x−y
We have
1 k−xy
H(x, y) = 2
2 (x + k)( y 2 + k)
n2 − 4(n − 1)x y
≥ ,
4(n − 1)(x 2 + k)( y 2 + k)
[x + (n − 1) y]2 − 4(n − 1)x y
=
4(n − 1)(x 2 + k)( y 2 + k)
[x − (n − 1) y)]2
= ≥ 0.
4(n − 1)(x 2 + k)( y 2 + k)
The equality holds for a1 = a2 = · · · = an = 1.
From
2u − 1
f 0 (u) = ,
u2 − u + 1
it follows that f is decreasing on (−∞, s0 ] and increasing on [s0 , ∞), where
1
s0 = < 1 = s.
2
In addition, from
1 + 2u(1 − u)
f 00 (u) = ,
(u2 − u + 1)2
it follows that f 00 (u) > 0 for u ∈ [s0 , 1], hence f is convex on [s0 , s]. According to
LPCF-Theorem, we only need to show that
f (x) + (n − 1) f ( y) ≥ n f (1)
g(x) ≥ g(1) = 0.
min{g(1), g(x 2 )} ≥ 0.
Partially Convex Function Method 261
f (x) + (n − 1) f ( y) ≥ n f (1)
262 Vasile Cîrtoaje
g(x) ≥ g(1) = 0.
2x 2 − (n + 2)x + 4n − 3 = 0
min{g(1), g(x 2 )} ≥ 0.
We have g(1) = 0 and g(x 2 ) > 0 for 27 ≤ n ≤ 38. For n = 39, the inequality does
not hold.
Remark 2. By replacing a1 , a2 , . . . , an respectively with 1 − a1 , 1 − a2 , . . . , 1 − an , we
get the following statement:
• Let a1 , a2 , . . . , an be real numbers such that a1 + a2 + · · · + an = 0. If n ≤ 38, then
(1 − a + a4 )(1 − b + b4 )(1 − c + c 4 ) ≥ 1.
a+b+c
f (a) + f (b) + f (c) ≥ 3 f (s), s= = 1,
3
where
f (u) = ln(1 − u + u4 ), u ∈ [0, 3].
From
4u3 − 1
f 0 (u) = ,
1 − u + u4
it follows that f is decreasing on [0, s0 ] and increasing on [s0 , 3], where
1
s0 = p
3
< 1 = s.
4
Also, f is convex on [s0 , 1] because
f (x) + 2 f ( y) ≥ 3 f (1)
for all x, y ≥ 0 so that x+2 y = 3. Using Note 2, it suffices to prove that H(x, y) ≥ 0,
where
f 0 (x) − f 0 ( y)
H(x, y) = .
x−y
We have
(x + y)(x − y)2 − 1 + 4(x 2 + y 2 + x y) − 2x y(x + y) − 4x 3 y 3
H(x, y) =
(1 − x + x 4 )(1 − y + y 4 )
−1 + 4(x 2 + y 2 + x y) − 2x y(x + y) − 4x 3 y 3
≥
(1 − x + x 4 )(1 − y + y 4 )
h(x, y)
= ,
(1 − x + x 4 )(1 − y + y 4 )
where
h(x, y) = −1 + 2(x + y)[2(x + y) − x y] − 4x y − 4x 3 y 3 .
From 3 = x + 2 y ≥ 2 2x y and (1 − x)(1 − y) ≤ 0, we get
p
9
xy ≤ , x + y ≥ 1 + x y.
8
264 Vasile Cîrtoaje
Therefore,
−3u4 + 6u − 1 −3u + 6u − 1 3u − 1
f 00 (u) = ≥ = > 0.
(1 − u + u )
3 2 (1 − u + u )
3 2 (1 − u + u3 )2
According to the LPCF-Theorem, we only need to show that
f (x) + 3 f ( y) ≥ 4 f (1)
for all x, y ≥ 0 so that x+3 y = 4. Using Note 2, it suffices to prove that H(x, y) ≥ 0,
where
f 0 (x) − f 0 ( y)
H(x, y) = .
x−y
We have
(x − y)2 + 3(x + y) − 1 − 3x 2 y 2 3(x + y) − 1 − 3x 2 y 2
H(x, y) = ≥ .
(1 − x + x 3 )(1 − y + y 3 ) (1 − x + x 3 )(1 − y + y 3 )
Partially Convex Function Method 265
4
xy ≤ , x + y ≥ 1 + x y.
3
Therefore,
3(x + y) − 1 − 3x 2 y 2 ≥ 3(1 + x y) − 1 − 3x 2 y 2
≥ 3(1 + x y) − 1 − 4x y = 2 − x y > 0,
1 1 1 1 1 1 1 1 1 1
5 2 + 2 + 2 + 2 + 2 + 45 ≥ 14 + + + + .
a b c d e a b c d e
a+b+c+d+e
f (a) + f (b) + f (c) + f (d) + f (e) ≥ 5 f (s), s= = 1,
5
where
5 14
f (u) = 2
− + 9, u ∈ I = R \ {0}.
u u
From
2(7u − 5)
f 0 (u) = ,
u3
it follows that f is increasing on (−∞, 0)∪[s0 , ∞) and decreasing on (0, s0 ], where
5
s0 = < 1 = s.
7
Since
lim f (u) = 9
u→−∞
From
2(15 − 14u)
f 00 (u) = ,
u4
266 Vasile Cîrtoaje
it follows that f is convex on [s0 , 1]. By the LPCF-Theorem, Note 4 and Note 1, it
suffices to show that h(x, y) ≥ 0 for all x, y ∈ I which satisfy x + 4 y = 5, where
Indeed, we have
9 5
g(u) = − 2,
u u
5x + 5 y − 9x y (6 y − 5)2
h(x, y) = = ≥ 0.
x2 y2 x2 y2
In accordance with Note 3, the equality holds for a = b = c = d = e = 1, and also
for
5 5
a= , b=c=d=e=
3 6
(or any cyclic permutation).
7 − 6a 7 − 6b 7 − 6c
+ + ≥ 1.
2 + a2 2 + b2 2 + c 2
a = e x , b = e y , c = ez ,
s0 = ln 3 > s.
Partially Convex Function Method 267
We have
2t · h(t)
f 00 (u) = , h(t) = −3t 4 + 14t 3 + 36t 2 − 28t − 12, t = eu .
(2 + t 2 )3
We will show that h(t) > 0 for t ∈ [1, 3], hence f is convex on [0, s0 ]. We have
f (x) + 2 f ( y) ≥ 3 f (0)
for all real x, y so that x + 2 y = 0. That is, to show that the original inequality
holds for b = c and a = 1/c 2 . Write this inequality as
1
a= , b=c=2
4
(or any cyclic permutation).
1 1 1 1
+ + ≤ .
a + 5bc b + 5ca c + 5ab 2
a b c 1
+ 2 + 2 ≤ .
a2 +5 b +5 c +5 2
Using the substitution
a = e x , b = e y , c = ez ,
we need to show that
f (x) + f ( y) + f (z) ≥ 3 f (s),
268 Vasile Cîrtoaje
where
x + y +z
s= =0
3
and
−eu
f (u) = , u ∈ R.
e2u + 5
From
eu (e2u − 5)
f 0 (u) = ,
(e2u + 5)2
it follows that f is decreasing on (−∞, s0 ] and increasing on [s0 , ∞), where
ln 5
s0 = > 0 = s.
2
Also, from
eu (−e4u + 30e2u − 25)
f 00 (u) = ,
(e2u + 5)3
p
it follows that f is convex on [s, s0 ], because u ∈ [0, s0 ] involves eu ∈ [1, 5 ] and
e2u ∈ [1, 5], hence
By the RPCF-Theorem, we only need to prove the original inequality for b = c and
a = 1/c 2 . Write this inequality as
c2 2c 1
+ 2 ≤ ,
5c + 1 c + 5 2
4
1 1 1 3
+ + ≤ .
4 − 3a + 4a 2 4 − 3b + 4b 2 4 − 3c + 4c 2 5
Solution. Let
a = ex , b = ey, c = ez .
We need to show that
f (x) + f ( y) + f (z) ≥ 3 f (s),
Partially Convex Function Method 269
where
x + y +z
s= =0
3
and
−1
f (u) = , u ∈ R.
4 − 3eu + 4e2u
From
eu (8eu − 3)
f 0 (u) = ,
(4 − 3eu + 4e2u )2
it follows that f is decreasing on (−∞, s0 ] and increasing on [s0 , ∞), where
3
s0 = ln < 0 = s.
8
We claim that f is convex on [s0 , 0]. Since
By the LPCF-Theorem, we only need to prove the original inequality for b = c and
a = 1/c 2 . Write this inequality as follows:
c4 2 3
+ ≤ ,
4c − 3c + 4 4 − 3c + 4c
4 2 2 5
7(4c 4 + 5c 3 − c 2 − 3c + 1) ≥ 0.
Indeed,
4c 4 + 5c 3 − c 2 − 3c + 1 = c 2 (2c − 1)2 + 9c 3 − 2c 2 − 3c + 1
and
9c 3 − 2c 2 − 3c + 1 = c(3c − 1)2 + (2c − 1)2 > 0.
270 Vasile Cîrtoaje
Solution. Let
a = ex , b = ey, c = ez .
We need to show that
f (x) + f ( y) + f (z) ≥ 3 f (s),
where
x + y +z
s= =0
3
and
−1
f (u) = , u ∈ R.
(3eu + 1)(3e2u − 5eu + 3)
From
(3eu − 2)(9eu − 2)
f 0 (u) = ,
(3eu + 1)2 (3e2u − 5eu + 3)2
it follows that f is increasing on (−∞, s1 ] ∪ [s0 , ∞) and decreasing on [s1 , s0 ],
where
s1 = ln 2 − ln 9, s0 = ln 2 − ln 3, s1 < s0 < 0 = s.
Since
−1
lim f (u) = f (s0 ) = ,
u→−∞ 3
we get
min f (u) = f (s0 ).
u∈R
t · h(t)
f 00 (u) = ,
(3t + 1)3 (3t 2 − 5t + 3)3
Partially Convex Function Method 271
where
2
t=e ∈u
,1 , h(t) = −729t 5 + 1188t 4 − 648t 3 + 387t 2 − 160t + 12.
3
it follows that h(t) > 0 for t ∈ [2/3, 1] ⊂ [t 2 , t 3 ], hence f is convex on [s0 , 0]. By
the LPCF-Theorem, we only need to prove the original inequality for b = c ≤ 1 and
a = 1/c 2 . Write this inequality as follows:
c6 2 3
+ ≤ .
(c 2 + 3)(3c 4 − 5c 2 + 3) (3c + 1)(3c 2 − 5c + 3) 4
Since
c 2 + 3 ≥ 2(c + 1)
and
3c 4 − 5c 2 + 3 ≥ c(3c 2 − 5c + 3),
it suffices to prove that
c5 2 3
+ ≤ .
2(c + 1)(3c 2 − 5c + 3) (3c + 1)(3c 2 − 5c + 3) 4
1 − a1 1 − a2 1 − an
+ + ··· + ≥ 0.
1 + pa1 + qa1 1 + pa2 + qa2
2 2
1 + pan + qan2
Solution. For q = 0, we get a known inequality (see Remark 2 from the proof
of P 1.63). Consider further that q > 0. Using the substitutions ai = e x i for i =
1, 2, . . . , n, we need to show that
f (x 1 ) + f (x 2 ) + · · · + f (x n ) ≥ n f (s),
272 Vasile Cîrtoaje
where
x1 + x2 + · · · + x n
s= =0
n
and
1 − eu
f (u) = , u ∈ R.
1 + peu + qe2u
From
eu (qe2u − 2qeu − p − 1)
f 0 (t) = ,
(1 + peu + qe2u )2
it follows that f is decreasing on (−∞, s0 ] and increasing on [s0 , ∞), where
v
t p+1
s0 = ln r0 > 0 = s, r0 = 1 + 1 + .
q
Also, we have
t · h(t)
f 00 (u) = ,
(1 + pt + qt 2 )3
where
We will show that h(t) ≥ 0 for t ∈ [1, r0 ], hence f is convex on [0, s0 ]. We have
h0 (t) is increasing,
h is increasing, hence
Since
p + 4q p + 2q
p + 3q − 1 ≥ p + 3q − = > 0,
n−1 2
f 00 (u) > 0 for u ∈ [0, s0 ], therefore f is convex on [s, s0 ]. By the RPCF-Theorem, we
only need to prove the original inequality for
a2 = · · · = an := t, a1 = 1/t n−1 , t ≥ 1.
Partially Convex Function Method 273
A = t n−1 (t n − nt + n − 1),
B = t 2n − t n+1 − (n − 1)(t − 1),
C = t n−1 [(n − 1)t n + 1 − nt n−1 ].
(p + 4q)C
pA + qB ≥ ,
n−1
which is equivalent to
This is true if
(n − 1)A − C ≥ 0
and
(n − 1)B − 4C ≥ 0
for t ≥ 1. By the AM-GM inequality, we have
For n = 3, we have
B = (t − 1)2 (t 4 + 2t 3 + 2t 2 + 2t + 2),
B − 2C = (t − 1)2 (t 4 − 2t 3 + 2t + 2)
= (t − 1)2 [(t − 1)2 (t 2 − 1) + 3] ≥ 0.
we have
We have
g10 (t) = n(n + 1)t n−1 − 4n(n − 1)t n−2 + 4n(n − 2)t n−3 − 2.
Since Æ
n(n + 1)t n−1 + 4n(n − 2)t n−3 ≥ 4n (n + 1)(n − 2)t n−2 ,
we get
Æ
g10 (t) ≥ 4n (n + 1)(n − 2) − n + 1 t n−2 − 2
Æ
≥ 4n (n + 1)(n − 2) − n + 1 − 2
4n(n − 3)
=p −2
(n + 1)(n − 2) + n − 1
4n(n − 3)
> − 2 = 2(n − 4) ≥ 0.
(n + 1) + n − 1
1 − a1 1 − a2 1 − a3 1 − a4 1 − a5 1 − a6
+ + + + + ≥0
1 + a12 1 + a22 1 + a32 1 + a42 1 + a52 1 + a62
1 − a1
− 1 ≥ 0,
1 + a12
1−a 1− b 1−c
+ + ≥ 0.
17 + 4a + 6a2 17 + 4b + 6b2 17 + 4c + 6c 2
a = e x , b = e y , c = ez ,
5
In addition, since p + 3q − 1 = > 0 (see the proof of P 3.30), f is convex on
17
[0, s0 ]. By the RPCF-Theorem, we only need to prove the original inequality for
b = c ≥ 1 and a = 1/c 2 . Write this inequality as follows:
c 2 (c 2 − 1) 2(1 − c)
+ ≥ 0,
c 4 + pc 2 + q 1 + pc + qc 2
pA + qB ≥ C,
where
A = c 2 (c − 1)2 (c + 2),
B = (c − 1)2 (c 4 + 2c 3 + 2c 2 + 2c + 2),
C = c 2 (c − 1)2 (2c + 1).
Indeed, we have
f (x 1 ) + f (x 2 ) + · · · + f (x 8 ) ≥ 8 f (s),
where
x1 + x2 + · · · + x8
s= =0
8
and
1 − eu
f (u) = , u ∈ R.
(1 + eu )2
From
eu (eu − 3)
f 0 (t) = ,
(1 + eu )3
Partially Convex Function Method 277
s0 = ln 3 > 1 = s.
We have
eu (8eu − e2u − 3)
f 00 (u) = .
(1 + eu )4
For u ∈ [0, ln 3], that is eu ∈ [1, 3], we have
(t − 1)6 ≥ 0.
−13 13
P 3.33. Let a, b, c be positive real numbers so that abc = 1. If k ∈ p , p ,
3 3 3 3
then
a+k b+k c+k 3(1 + k)
+ 2 + 2 ≤ .
a +1 b +1 c +1
2 2
(Vasile C., 2012)
where
h(t) = −t 4 − 4kt 3 + 6t 2 + 4kt − 1, t = eu .
We will show that h(t) > 0 for t ∈ [r0 , 1], hence f is convex on [s0 , s]. Indeed,
since
52t 52
4kt = p ≥ > 1,
3 3 27
we have
3t 6 − 4t 5 + t 4 + t 2 − 4t + 3 − k(1 − t 2 )3 ≥ 0,
(t − 1)2 [(3 + k)t 4 + 2(1 + k)t 3 + 2t 2 + 2(1 − k)t + 3 − k] ≥ 0,
p 2 p p p
(t − 1)2 t − 2 + 3 (27 + 13 3)t 2 + 24(2 + 3)t + 33 + 17 3 ≥ 0.
13
The equality holds for a = b = c = 1. If k = p , then the equality holds also for
3 3
p p
a = 7 + 4 3, b = c =2− 3
−13
(or any cyclic permutation). If k = p , then the equality holds also for
3 3
p p
a = 7 − 4 3, b = c =2+ 3
p
P 3.34. If a, b, c are positive real numbers and 0 < k ≤ 2 + 2 2, then
a3 b3 c3 a+b+c
+ 2 + 2 ≥ .
ka + bc kb + ca kc + ab
2 k+1
a4 b4 b4 a b c
+ + ≥ + + ,
ka + 1 kb + 1 kb + 1 k + 1 k + 1 k + 1
3 3 3
280 Vasile Cîrtoaje
a4 − a b4 − b c4 − c
+ + ≥ 0.
ka3 + 1 kb3 + 1 kc 3 + 1
Using the substitution
a = ex , b = ey, c = ez ,
we need to show that
f (x) + g( y) + g(z) ≥ 3 f (s),
where
x + y +z
s= =0
3
and
e4u − eu
f (u) = , u ∈ R.
ke3u + 1
From
ke6u + 2(k + 2)e3u − 1
f 0 (t) = ,
(ke3u + 1)2
it follows that f is decreasing on (−∞, s0 ] and increasing on [s0 , ∞), where
v
p
3 −k − 2 + (k + 1)(k + 4)
u
t
s0 = ln r0 < 0, r0 = ∈ (0, 1).
k
Also, we have
t · h(t)
f 00 (u) = ,
(kt 3 + 1)3
where
h(t) = k2 t 9 − k(4k + 1)t 6 + (13k + 16)t 3 − 1, t = eu .
If h(t) > p
0 for t ∈ [r0 , 1], then f is convex on [s0 , 0]. We will prove this only for
k = 2 + 2 2, when r0 ≈ 0.415 and h(t) ≥ 0 for t ∈ [t 1 , t 2 ], where t 1 ≈ 0.2345 and
t 2 ≈ 1.02. Since [r0 , 1] ⊂ [t 1 , t 2 ], the conclusion follows. By the LPCF-Theorem,
we only need to prove the original inequality for b = c. Due to homogeneity, we
may consider that b = c = 1. Thus, we need to show that
a3 2 a+2
+ ≥ ,
ka + 1 a + k
2 k+1
which is equivalent to the obvious inequality
(a − 1)2 [a2 − (k − 2)a + 2] ≥ 0.
p
For k = 2 + 2 2, this inequality has the form
p
(a − 1)2 (a − 2)2 ≥ 0.
p
The equality holds for a = b = c. If k = 2 + 2 2, then the equality holds also for
a
p =b=c
2
(or any cyclic permutation).
Partially Convex Function Method 281
1 1 1 1 1 1
2 + + ··· + ≥3 + + ··· + .
a+1 b+1 e+1 a+2 b+2 e+2
a = ex , b = ey, c = ez , d = et , e = ew,
where
x + y +z+ t +w
s= =0
5
and
1 − eu
f (u) = u , u ∈ R.
(e + 1)(eu + 2)
From
eu (e2u − 2eu − 5)
f 0 (u) = ,
(eu + 1)2 (eu + 2)2
it follows that f is decreasing on (−∞, s0 ] and increasing on [s0 , ∞), where
p
s0 = ln(1 + 6) < 2, s < s0 .
Also, we have
t · h(t)
f 00 (u) = , t = eu ,
(t + 1)3 (t + 2)3
where
h(t) = −t 4 + 7t 3 + 21t 2 + 7t − 10.
We will show that h(t) > 0 for t ∈ [1, 2], hence f is convex on [0, s0 ]. We have
b = c = d = e := t, a = 1/t 4 , t ≥ 1.
282 Vasile Cîrtoaje
t 4 (t 4 − 1) 4(t − 1)
≥ ,
(t 4 + 1)(2t 4 + 1) (t + 1)(t + 2)
which is true if
Since
(t 4 + 1)(2t 4 + 1) = 2t 8 + 3t 4 + 1 ≤ 2t 4 (t 4 + 2),
it suffices to show that
t 5 − 4t 4 + 6t 3 + 6t 2 + 5t − 14 ≥ 0,
1 1 1 1 1 1
3 + + ··· + ≥2 + + ··· + .
2a1 + 1 2a2 + 1 2a14 + 1 a1 + 1 a2 + 1 a14 + 1
1 − a1 1 − a2 1 − a14
+ + ··· + ≥ 0.
(a1 + 1)(2a1 + 1) (a2 + 1)(2a2 + 1) (a14 + 1)(2a14 + 1)
f (x 1 ) + f (x 2 ) + · · · + f (x 14 ) ≥ 14 f (s),
where
x 1 + x 2 + · · · + x 14
s= =0
14
and
1 − eu
f (u) = , u ∈ R.
(eu + 1)(2eu + 1)
Partially Convex Function Method 283
From
2eu (e2u − 2eu − 2)
f 0 (u) = ,
(eu + 1)2 (2eu + 1)2
it follows that f is decreasing on (−∞, s0 ] and increasing on [s0 , ∞), where
p
s0 = ln(1 + 3) < 2, s < s0 .
Also, we have
2t · h(t)
f 00 (u) = , t = eu ,
(t + 1)3 (2t + 1)3
where
h(t) = −2t 4 + 11t 3 + 15t 2 + 2t − 2.
We will show that h(t) > 0 for t ∈ [1, 2], hence f is convex on [0, s0 ]. We have
a2 = a3 = · · · = a14 := t, a1 = 1/t 13 , t ≥ 1.
t 13 (t 13 − 1) 13(t − 1)
≥ .
(t + 1)(t + 2) (t + 1)(2t + 1)
13 13
Since
(t 13 + 1)(t 13 + 2) = t 26 + 3t 13 + 2 ≤ t 13 (t 13 + 5),
it suffices to show that
t 13 − 1 13(t − 1)
≥ ,
t 13 + 5 (t + 1)(2t + 1)
which is equivalent to
t 13 (t 2 − 5t + 7) − t 2 − 34t + 32 ≥ 0.
Substituting
t = 1 + x, x ≥ 0,
the inequality becomes
(1 + x)13 (x 2 − 3x + 3) − x 2 − 36x − 3 ≥ 0.
Since
(1 + x)13 ≥ 1 + 13x + 78x 2 ,
it suffices to show that
is true since
1 − a1 1 − a2 1 − a8
+ + ··· + ≥ 0.
(a1 + 1)(ka1 + 1) (a2 + 1)(ka2 + 1) (a8 + 1)(ka8 + 1)
f (x 1 ) + f (x 2 ) + · · · + f (x 8 ) ≥ 8 f (s),
where
x1 + x2 + · · · + x8
s= =0
8
and
1 − eu
f (u) = , u ∈ R.
(eu + 1)(keu + 1)
From
eu (ke2u − 2keu − k − 2)
f 0 (u) = ,
(eu + 1)2 (keu + 1)2
it follows that f is decreasing on (−∞, s0 ] and increasing on [s0 , ∞), where
v
t 2
s0 = ln 1 + 2+ < 2, s < s0 .
k
Also, we have
t · h(t)
f 00 (u) = , t = eu ,
(t + 1)3 (kt + 1)3
Partially Convex Function Method 285
where
We will show that h(t) > 0 for t ∈ [1, 2], hence f is convex on [0, s0 ]. We have
a2 = a3 = · · · = a8 := t, a1 = 1/t 7 , t ≥ 1.
t 7 (t 7 − 1) 7(t − 1)
≥ .
(t + 1)(t + k) (t + 1)(kt + 1)
7 7
Since
(t 7 + 1)(t 7 + k) = t 14 + (k + 1)t 7 + k ≤ t 7 (t 7 + 2k + 1),
it suffices to show that
t7 − 1 7(t − 1)
≥ ,
t 7 + 2k + 1 (t + 1)(kt + 1)
which is equivalent to
k(t − 1)P(t) + Q(t) ≥ 0,
where
P(t) = t(t + 1)(t 6 + t 5 + t 4 + t 3 + t 2 + t + 1) − 14,
Q(t) = (t + 1)(t 7 − 1) − 7(t − 1)(t 7 + 1).
Since (t − 1)P(t) ≥ 0 for t ≥ 1, it suffices to consider the case k = 1. So, we need
to show that
t 7 − 1 7(t − 1)
≥ ,
t7 + 3 (t + 1)2
which is equivalent to
t 7 (t 2 − 5t + 8) − t 2 − 23t + 20 ≥ 0.
Substituting
t = 1 + x, x ≥ 0,
the inequality becomes
(1 + x)7 (x 2 − 3x + 4) − x 2 − 25x − 4 ≥ 0.
286 Vasile Cîrtoaje
Since
(1 + x)7 ≥ 1 + 7x + 21x 2 ,
it suffices to show that
is true since
1 1 1 1 1 1
+ + ··· + ≥ + + ··· + .
2a1 + 1 2a2 + 1 2a9 + 1 a1 + 2 a2 + 2 a9 + 2
1 − a1 1 − a2 1 − a9
+ + ··· + ≥ 0.
(2a1 + 1)(a1 + 2) (2a2 + 1)(a2 + 2) (2a9 + 1)(a9 + 2)
f (x 1 ) + f (x 2 ) + · · · + f (x 9 ) ≥ 9 f (s),
where
x1 + x2 + · · · + x9
s= =0
9
and
1 − eu
f (u) = , u ∈ R.
(2eu + 1)(eu + 2)
From
eu (2e2u − 4eu − 7)
f 0 (u) = ,
(2eu + 1)2 (eu + 2)2
it follows that f is decreasing on (−∞, s0 ] and increasing on [s0 , ∞), where
p
3 2
s0 = ln 1 + < 2, s < s0 .
2
Partially Convex Function Method 287
Also, we have
t · h(t)
f 00 (u) = , t = eu ,
(2t + 1)3 (t + 2)3
where
h(t) = −4t 4 + 26t 3 + 54t 2 + 19t − 14.
We will show that h(t) > 0 for t ∈ [1, 2], hence f is convex on [0, s0 ]. We have
a2 = a3 = · · · = a9 := t, a1 = 1/t 8 , t ≥ 1.
which is equivalent to
Substituting
t = 1 + x, x ≥ 0,
the inequality becomes
Since
(1 + x)8 ≥ 1 + 8x + 28x 2 ,
it suffices to show that
is true since
−1
a1 , a2 , . . . , an ≥ , a1 + a2 + · · · + an = n,
n−2
then
a12 a22 an2
+ + ··· + ≤ n.
a12 − a1 + 1 a22 − a2 + 1 an2 − an + 1
a1 + a2 + · · · + an
f (a1 ) + f (a2 ) + · · · + f (an ) ≥ n f (s), s= = 1,
n
where
1−u −1 n2 − n − 1
f (u) = 2 , u∈I= , .
u −u+1 n−2 n−2
Let s0 = 2. We have s < s0 and
because
1−u 1 (u − 2)2
f (u) − f (2) = + = ≥ 0.
u2 − u + 1 3 3(u2 − u + 1)
From
u(u − 2)
f 0 (u) = ,
(u2− u + 1)2
2(3u2 − u3 − 1) 2u2 (2 − u) + 2(u2 − 1)
f 00 (u) = = ,
(u2 − u + 1)3 (u2 − u + 1)3
it follows that f is convex on [1, s0 ]. However, we can’t apply the RPCF-Theorem
in its original form because f is not decreasing on I≤s0 . According to Theorem 1,
we may replace this condition with ns − (n − 1)s0 ≤ inf I. Indeed, we have
−1
ns − (n − 1)s0 = n − 2(n − 1) = −n + 2 ≤ = inf I.
n−2
So, it suffices to show that f (x) + (n − 1) f ( y) ≥ n f (1) for all x, y ∈ I so that
x + (n − 1) y = n.
We have
−1
g(u) = ,
u2 −u+1
x + y −1 (n − 2)x + 1
h(x, y) = = ≥ 0.
(x 2 − x + 1)( y 2 − y + 1) (n − 1)(x 2 − x + 1)( y 2 − y + 1)
The equality holds for a1 = a2 = · · · = an = 1, and also for
−1 n−1
a1 = , a2 = a3 = · · · = a n =
n−2 n−2
(or any cyclic permutation).
−n
a1 , a2 , . . . , an ≥ , a1 + a2 + · · · + an = n,
n−2
then
1 1 1 1 1 1
2
+ 2 + ··· + 2 ≥ + + ··· + .
a1 a2 an a1 a2 an
a1 + a2 + · · · + an
f (a1 ) + f (a2 ) + · · · + f (an ) ≥ n f (s), s= = 1,
n
where
1 1 −n n(2n − 3)
f (u) = 2 − , u∈I= , \ {0}.
u u n−2 n−2
Let
s0 = 2, s < s0 .
From
1 1 1 (u − 2)2
f (u) − f (2) = − + = ≥ 0,
u2 u 4 4u2
it follows that
min f (u) = f (s0 ),
u∈I
while from
u−2 2(3 − u)
f 0 (u) = , f 00 (u) = ,
u3 u4
Partially Convex Function Method 291
−n
ns − (n − 1)s0 = n − 2(n − 1) = −n + 2 ≤ = inf I.
n−2
So, according to Note 1, it suffices to show that h(x, y) ≥ 0 for all x, y ∈ I so that
x + (n − 1) y = n. We have
f (u) − f (1) −1
g(u) = = 2,
u−1 u
1 − a1 1 − a2 1 − an
+ + ··· + ≥ 0,
k + a1 k + a2
2 2
k + an2
−n n
a1 = , a2 = a3 = · · · = a n =
n−2 n−2
a1 + a2 + · · · + an
f (a1 ) + f (a2 ) + · · · + f (an ) ≥ n f (s), s= = 1,
n
where
n+1 n−1
f (u) = − , u ∈ I = [−1, 2n − 1] \ {0}.
u2 u
Let
2(n + 1)
s0 = ∈ I, s < s0 .
n−1
Since
[(n − 1)u − 2(n + 1)]2
f (u) − f (s0 ) = ≥ 0,
4(n + 1)u2
we have
min f (u) = f (s0 ).
u∈I
From
(n − 1)u − 2(n + 1) 6(n + 1) − 2(n − 1)u
f 0 (u) = , f 00 (u) = ,
u3 u4
it follows that f is convex on [1, s0 ]. Since f is not decreasing on I≤s0 , according
to Theorem 1 and Note 6, we may replace this condition in RPCF-Theorem with
ns − (n − 1)s0 ≤ inf I. We have
n+1
a1 = −1, a2 = · · · = an =
n−1
(or any cyclic permutation).
Partially Convex Function Method 293
−(3n − 2)
a1 , a2 , . . . , an ≥ , a1 + a2 + · · · + an = n,
n−2
then
1 − a1 1 − a2 1 − an
+ + ··· + ≥ 0.
(1 + a1 )2 (1 + a2 )2 (1 + an )2
(Vasile C., 2014)
From
u−3 2(5 − u)
f 0 (u) = , f 00 (u) = ,
(u + 1)3 (u + 1)4
it follows that f is convex on [1, s0 ]. We can’t apply the RPCF-Theorem in its original
form because f is not decreasing on I≤s0 . However, according to Theorem 1 and
Note 6, we may replace this condition with ns − (n − 1)s0 ≤ inf I. Indeed, for n ≥ 4,
we have
−(3n − 2)
ns − (n − 1)s0 = n − 3(n − 1) = −2n + 3 ≤ = inf I.
n−2
According to Note 1, it suffices to show that h(x, y) ≥ 0 for all x, y ∈ I so that
x ≤ 1 ≤ y and x + (n − 1) y = n. We have
f (u) − f (1) −1
g(u) = = ,
u−1 (u + 1)2
g(x) − g( y) x + y +2 (n − 2)x + 3n − 2
h(x, y) = = = ≥ 0.
x−y (x + 1) ( y + 1)
2 2 (n − 1)(x + 1)2 ( y + 1)2
294 Vasile Cîrtoaje
a1 + a2 + · · · + an
f (a1 ) + f (a2 ) + · · · + f (an ) ≥ n f (s), s= = 1,
n
where
1−u
f (u) = , u ∈ I = [0, n] \ {1/k}.
(1 − ku)2
Let
s0 = 2 − 1/k, 1 = s < s0 .
Since
we have
min f (u) = f (s0 ).
u∈I
From
ku − 2k + 1 2k(−ku + 3k − 2)
f 0 (u) = , f 00 (u) = ,
(ku − 1)3 (1 − ku)4
it follows that f is convex on [1, s0 ]. We can’t apply the RPCF-Theorem because f
is not decreasing on I≤s0 . According to Theorem 1 and Note 6, we may replace this
condition with ns − (n − 1)s0 ≤ inf I. Indeed, we have
3n − 4 4−n
ns − (n − 1)s0 ≤ n − (n − 1) · = ≤ 0 = inf I.
2(n − 1) 2
Partially Convex Function Method 295
Since
−1 k[k(x + y) − 2]
g(u) = , h(x, y) = ,
(1 − ku)2 (1 − kx)2 (1 − k y)2
we need to show that k(x + y) − 2 ≥ 0. Indeed, we have
a1 + a2 + · · · + an = ns
and
a1 ≤ a2 ≤ · · · ≤ am ≤ s, m ∈ {1, 2, . . . , n − 1},
if and only if
f (x) + (n − m) f ( y) ≥ (1 + n − m) f (s)
for all x, y ∈ I so that x ≤ s ≤ y and x + (n − m) y = (1 + n − m)s.
Proof. For
a1 = x, a2 = · · · = am = s, am+1 = · · · = an = y,
the inequality
f (a1 ) + f (a2 ) + · · · + f (an ) ≥ n f (s)
becomes
f (x) + (n − m) f ( y) ≥ (1 + n − m) f (s);
297
298 Vasile Cîrtoaje
therefore, the necessity is obvious. By Lemma from Chapter 3, to prove the suffi-
ciency, it suffices to consider that a1 , a2 , . . . , an ∈ J, where
J = I≤s0 .
Because f is convex on J≥s , the desired inequality follows from HCF-OV Theorem
applied to the interval J.
Similarly, we can prove Left Partially Convex Function Theorem for Ordered Vari-
ables (LPCF-OV Theorem).
LPCF-OV Theorem. Let f be a real function defined on an interval I and convex on
[s0 , s], where s0 , s ∈ I, s0 < s. In addition, f is increasing on I≥s0 and f (u) ≥ f (s0 )
for u ∈ I. The inequality
a + a + ··· + a
1 2 n
f (a1 ) + f (a2 ) + · · · + f (an ) ≥ n f
n
holds for all a1 , a2 , . . . , an ∈ I satisfying
a1 + a2 + · · · + an = ns
and
a1 ≥ a2 ≥ · · · ≥ am ≥ s, m ∈ {1, 2, . . . , n − 1},
if and only if
f (x) + (n − m) f ( y) ≥ (1 + n − m) f (s)
for all x, y ∈ I so that x ≥ s ≥ y and x + (n − m) y = (1 + n − m)s.
The RPCF-OV Theorem and the LPCF-OV Theorems are respectively generaliza-
tions of the RPCF Theorem and LPCF Theorem, because the last theorems can be
obtained from the first theorems for m = 1.
We may replace the hypothesis condition in the RPCF-OV Theorem and the LPCF-OV
Theorem, namely
f (x) + m f ( y) ≥ (1 + m) f (s),
by the condition
h(x, y) ≥ 0 for all x, y ∈ I so that x + m y = (1 + m)s.
f 0 (x) − f 0 ( y)
H(x, y) = .
x−y
PCF Method for Ordered Variables 299
The desired inequality of Jensen’s type in the RPCF-OV Theorem and the LPCF-OV
Theorem holds true by replacing the hypothesis
f (x) + m f ( y) ≥ (1 + m) f (s)
Note 3. The desired inequalities in the RPCF-OV Theorem and the LPCF-OV Theo-
rem become equalities for
a1 = a2 = · · · = an = s.
a1 = x, a2 = · · · = am = s, am+1 = · · · = an = y
(or any cyclic permutation). Notice that these equality conditions are equivalent to
x + (n − m) y = (1 + n − m)s, h(x, y) = 0
Note 4. The RPCF-OV Theorem is also valid in the case where f is defined on
I \ {u0 }, where u0 is an interior point of I so that u0 > s0. Similarly, LPCF Theorem
is also valid in the case in which f is defined on I \ {u0 }, where u0 is an interior
point of I so that u0 < s0.
where
s, s0 ∈ I, s < s0 , (1 + n − m)s − (n − m)s0 ≤ inf I.
The inequality
a + a + ··· + a
1 2 n
f (a1 ) + f (a2 ) + · · · + f (an ) ≥ n f
n
300 Vasile Cîrtoaje
a1 + a2 + · · · + an = ns
and
a1 ≤ a2 ≤ · · · ≤ am ≤ s, m ∈ {1, 2, . . . , n − 1},
if and only if
f (x) + (n − m) f ( y) ≥ (1 + n − m) f (s)
for all x, y ∈ I so that x ≤ s ≤ y and x + (n − m) y = (1 + n − m)s.
The proof of this theorem is similar to the one of Theorem 1 from chapter 3.
ns − (n − 1)s0 ≥ sup I.
More precisely, the following theorem holds:
Theorem 2. Let f be a function defined on a real interval I, convex on [s0 , s] and
satisfying
min f (u) = f (s0 ),
u∈I≤s
where
s, s0 ∈ I, s > s0 , (1 + n − m)s − (n − m)s0 ≥ sup I.
The inequality
a + a + ··· + a
1 2 n
f (a1 ) + f (a2 ) + · · · + f (an ) ≥ n f
n
holds for all a1 , a2 , . . . , an ∈ I satisfying
a1 + a2 + · · · + an = ns
and
a1 ≥ a2 ≥ · · · ≥ am ≥ s, m ∈ {1, 2, . . . , n − 1},
if and only if
f (x) + (n − m) f ( y) ≥ (1 + n − m) f (s)
for all x, y ∈ I so that x ≥ s ≥ y and x + (n − m) y = (1 + n − m)s.
Note 6. Theorem 1 is also valid in the case in which f is defined on I \ {u0 }, where
/ [s, s0 ]. Similarly, Theorem 2 is also valid in
u0 is an interior point of I so that u0 ∈
the case in which f is defined on I \ {u0 }, where u0 is an interior point of I so that
/ [s0 , s].
u0 ∈
PCF Method for Ordered Variables 301
Note 7. We can extend weighted Jensen’s inequality to right and left partially con-
vex functions with ordered variables establishing the WRPCF-OV Theorem and the
WLPCF-OV Theorem (Vasile Cirtoaje, 2014).
WRPCF-OV Theorem. Let p1 , p2 , . . . , pn be positive real numbers so that
p1 + p2 + · · · + pn = 1,
and let f be a real function defined on an interval I and convex on [s, s0 ], where
s, s0 ∈ int(I), s < s0 . In addition, f is decreasing on I≤s0 and f (u) ≥ f (s0 ) for u ∈ I.
The inequality
p1 f (x 1 ) + p2 f (x 2 ) + · · · + pn f (x n ) ≥ f (p1 x 1 + p2 x 2 + · · · + pn x n )
x1 ≤ x2 ≤ · · · ≤ x n, x m ≤ s, m ∈ {1, 2, . . . , n − 1},
if and only if
f (x) + k f ( y) ≥ (1 + k) f (s)
for all x, y ∈ I satisfying
x ≤ s ≤ y, x + k y = (1 + k)s,
where
pm+1 + pm+2 + · · · + pn
k= .
p1
p1 + p2 + · · · + pn = 1,
and let f be a real function defined on an interval I and convex on [s0 , s], where
s0 , s ∈ I, s0 < s. In addition, f is increasing on I≥s0 and f (u) ≥ f (s0 ) for u ∈ I. The
inequality
p1 f (x 1 ) + p2 f (x 2 ) + · · · + pn f (x n ) ≥ f (p1 x 1 + p2 x 2 + · · · + pn x n )
x1 ≥ x2 ≥ · · · ≥ x n, x m ≥ s, m ∈ {1, 2, . . . , n − 1},
if and only if
f (x) + k f ( y) ≥ (1 + k) f (s)
for all x, y ∈ I satisfying
x ≥ s ≥ y, x + k y = (1 + k)s,
302 Vasile Cîrtoaje
where
pm+1 + pm+2 + · · · + pn
k= .
p1
1
p1 = p2 = · · · = pn = ,
n
the WRPCF-OV Theorem and the WLPCF-OV Theorem yield the RPCF-OV Theorem
and the LPCF-OV Theorem, respectively.
PCF Method for Ordered Variables 303
4.2 Applications
a ≤ 1 ≤ b ≤ c ≤ d, a + b + c + d = 4,
then
a b c d
+ 2 + 2 + 2 ≤ 1.
3a2 + 1 3b + 1 3c + 1 3d + 1
a ≥ b ≥ 1 ≥ c ≥ d, a + b + c + d = 4,
then
16a − 5 16b − 5 16c − 5 16d − 5 4
+ + + ≤ .
32a + 1 32b + 1 32c + 1 32d + 1 3
2 2 2 2
a ≥ b ≥ 1 ≥ c ≥ d ≥ e, a + b + c + d + e = 5,
then
18a − 5 18b − 5 18c − 5 18d − 5 18e − 5
+ + + + ≤ 5.
12a2 + 1 12b2 + 1 12c 2 + 1 12d 2 + 1 12e2 + 1
a ≥ b ≥ 1 ≥ c ≥ d ≥ e, a + b + c + d + e = 5,
then
a(a − 1) b(b − 1) c(c − 1) d(d − 1) e(e − 1)
+ + 2 + + 2 ≥ 0.
3a2 + 4 3b2 + 4 3c + 4 3d 2 + 4 3e + 4
n+1
If k ≥ p , then
2 n
n+1
If k ≥ 1 + p , then
n
a12 − 1 a22 − 1 2
a2n −1
+ + ··· + ≥ 0.
(a1 + k)2 (a2 + k)2 (a2n + k)2
a1 ≥ 1 ≥ a2 ≥ · · · ≥ an , a1 + a2 + · · · + an = n,
then
3/a1 3/a2
a1 + a2 + · · · + an3/an ≤ n.
then
(1 − a1 + a12 )(1 − a2 + a22 ) · · · (1 − a11 + a11
2
) ≥ 1.
a1 ≥ a2 ≥ a3 ≥ a4 ≥ 1 ≥ a5 ≥ a6 ≥ a7 ≥ a8 , a1 + a2 + · · · + a8 = 8,
then
1 1 1 1 1 1
5 2 + 2 + · · · + 2 + 72 ≥ 14 + + ··· + .
a1 a2 a8 a1 a2 a8
a ≤ b ≤ 1 ≤ c ≤ d, abcd = 1,
then
7 − 6a 7 − 6b 7 − 6c 7 − 6d 4
+ + + ≥ .
2+a 2 2+ b 2 2+c 2 2+d 2 3
PCF Method for Ordered Variables 305
a ≤ b ≤ 1 ≤ c, abc = 1,
then
7 − 4a 7 − 4b 7 − 4c
+ + ≥ 3.
2 + a2 2 + b2 2 + c 2
a ≥ 1 ≥ b ≥ c, abc = 1,
then
23 − 8a 23 − 8b 23 − 8c
+ + ≥ 9.
3 + 2a2 3 + 2b2 3 + 2c 2
a1 ≤ · · · ≤ an−1 ≤ 1 ≤ an , a1 a2 · · · an = 1.
If p, q ≥ 0 so that p + 3q ≥ 1, then
1 − a1 1 − a2 1 − an
+ + ··· + ≥ 0.
1 + pa1 + qa1 1 + pa2 + qa2
2 2
1 + pan + qan2
−2 ≤ a ≤ b ≤ 1 ≤ c ≤ d ≤ e, a + b + c + d + e = 5,
then
1 1 1 1 1 1 1 1 1 1
2
+ 2+ 2+ 2+ 2≥ + + + + .
a b c d e a b c d e
306 Vasile Cîrtoaje
PCF Method for Ordered Variables 307
4.3 Solutions
a ≤ 1 ≤ b ≤ c ≤ d, a + b + c + d = 4,
then
a b c d
+ + + ≤ 1.
3a2 + 1 3b2 + 1 3c 2 + 1 3d 2 + 1
a+b+c+d
f (a) + f (b) + f (c) + f (d) ≥ 4 f (s), s= = 1,
4
where
−u
f (u) = , u ∈ R.
3u2 + 1
From
3u2 − 1
f 0 (u) = ,
(3u2 + 1)2
it follows that pf is increasing on (−∞, −s0 ] ∪ [s0 , ∞) and decreasing on [−s0 , s0 ],
where s0 = 1/ 3. Since
lim f (u) = 0
u→−∞
From
18u(1 − u2 )
f 00 (u) = ,
(3u2 + 1)3
it follows that f is convex on [0, 1], hence on [s0 , 1]. Therefore, we may apply the
LPCF-OV Theorem for n = 4 and m = 1. We only need to show that f (x) + f ( y) ≥
2 f (1) for all real x, y so that x + y = 2. Using Note 1, it suffices to prove that
h(x, y) ≥ 0, where
Indeed, we have
3u − 1
g(u) = ,
4(3u2 + 1)
3(1 + x + y − 3x y) 9(1 − x y)
h(x, y) = = ≥ 0,
4(3x + 1)(3 y + 1) 4(3x + 1)(3 y 2 + 1)
2 2 2
308 Vasile Cîrtoaje
since
4(1 − x y) = (x + y)2 − 4x y = (x − y)2 ≥ 0.
Thus, the proof is completed. The equality holds for a = b = c = d = 1.
Remark. Similarly, we can prove the following generalization:
• If a1 , a2 , . . . , an are real numbers so that
a1 ≤ 1 ≤ a2 ≤ · · · ≤ an , a1 + a2 + · · · + an = n,
then
a1 a an n
+ 22 + ··· + 2 ≤ ,
3a12+ 1 3a2 + 1 3an + 1 4
with equality for a1 = a2 = · · · = an = 1.
a ≥ b ≥ 1 ≥ c ≥ d, a + b + c + d = 4,
then
16a − 5 16b − 5 16c − 5 16d − 5 4
+ + + ≤ .
32a + 1 32b + 1 32c + 1 32d + 1 3
2 2 2 2
a+b+c+d
f (a) + f (b) + f (c) + f (d) ≥ 4 f (s), s= = 1,
4
where
5 − 16u
f (u) = , u ∈ R.
32u2 + 1
As shown in the proof of P 3.1, f is convex on [s0 , 1], increasing for u ≥ s0 and
where p
5 + 33
s0 = ≈ 0.6715.
16
Therefore, we may apply the LPCF-OV Theorem for n = 4 and m = 2. We only need
to show that f (x) + 2 f ( y) ≥ 3 f (1) for all real x, y so that x + 2 y = 3. Using Note
1, it suffices to prove that h(x, y) ≥ 0, where
Indeed, we have
32(2u − 1)
g(u) = ,
3(32u2 + 1)
64(1 + 16x + 16 y − 32x y) 64(4x − 5)2
h(x, y) = = ≥ 0.
3(32x 2 + 1)(32 y 2 + 1) 3(32x 2 + 1)(32 y 2 + 1)
From x + 2 y = 3 and h(x, y) = 0, we get x = 5/4 and y = 7/8. Therefore, in
accordance with Note 3, the equality holds for a = b = c = d = 1, and also for
5 7
a= , b = 1, c=d= .
4 8
a1 ≥ · · · ≥ an−2 ≥ 1 ≥ an−1 ≥ an , a1 + a2 + · · · + an = n,
then
16a1 − 5 16a2 − 5 16an − 5 n
+ + · · · + ≤ ,
32a12 + 1 32a22 + 1 32an2 + 1 3
with equality for a1 = a2 = · · · = an = 1, and also for
5 7
a1 = , a2 = · · · = an−2 = 1, an−1 = an = .
4 8
a ≥ b ≥ 1 ≥ c ≥ d ≥ e, a + b + c + d + e = 5,
then
18a − 5 18b − 5 18c − 5 18d − 5 18e − 5
+ + + + ≤ 5.
12a + 1 12b + 1 12c + 1 12d + 1 12e2 + 1
2 2 2 2
where p
5 + 52
s0 = ≈ 0.678.
18
Therefore, applying the LPCF-OV Theorem for n = 5 and m = 3, we only need to
show that f (x) + 3 f ( y) ≥ 4 f (1) for all real x, y so that x + 3 y = 4. Using Note 1,
it suffices to prove that h(x, y) ≥ 0, where
Indeed, we have
6(2u − 1)
g(u) = ,
12u2 + 1
12(1 + 6x + 6 y − 12x y) 12(2x − 3)2
h(x, y) = = ≥ 0.
(12x 2 + 1)(12 y 2 + 1) (12x 2 + 1)(12 y 2 + 1)
From x + 3 y = 4 and h(x, y) = 0, we get x = 3/2 and y = 5/6. Therefore, in
accordance with Note 3, the equality holds for a = b = c = d = e = 1, and also for
3 5
a= , b = 1, c=d=e= .
2 6
then
18a1 − 5 18a2 − 5 18an − 5
+ + ··· + ≤ n,
12a1 + 1 12a2 + 1
2 2
12an2 + 1
with equality for a1 = a2 = · · · = an = 1, and also for
3 5
a1 = , a2 = · · · = an−3 = 1, an−2 = an−1 = an = .
2 6
a ≥ b ≥ 1 ≥ c ≥ d ≥ e, a + b + c + d + e = 5,
then
a(a − 1) b(b − 1) c(c − 1) d(d − 1) e(e − 1)
+ + 2 + + 2 ≥ 0.
3a2 + 4 3b2 + 4 3c + 4 3d 2 + 4 3e + 4
(Vasile C., 2012)
PCF Method for Ordered Variables 311
a+b+c+d+e
f (a) + f (b) + f (c) + f (d) + f (e) ≥ 5 f (s), s= = 1,
5
where
u2 − u
f (u) = , u ∈ R.
3u2 + 4
As shown in the proof of P 3.5, f is convex on [s0 , 1], increasing for u ≥ s0 and
where p
−4 + 2 7
s0 = ≈ 0.43.
3
Therefore, we may apply the LPCF-OV Theorem for n = 5 and m = 2. We only need
to show that f (x) + 3 f ( y) ≥ 4 f (1) for all real x, y so that x + 3 y = 4. Using Note
1, it suffices to prove that h(x, y) ≥ 0. Indeed, we have
f (u) − f (1) u
g(u) = = 2 ,
u−1 3u + 4
g(x) − g( y) 4 − 3x y (x − 2)2
h(x, y) = = = ≥ 0.
x−y (3x 2 + 4)(3 y 2 + 4) (12x 2 + 1)(12 y 2 + 1)
From x + 3 y = 4 and h(x, y) = 0, we get x = 2 and y = 2/3. Therefore, in
accordance with Note 3, the equality holds for
a = b = c = d = e = 1,
then
a1 (a1 − 1) a2 (a2 − 1) an (an − 1)
+ + ··· + ≥ 0,
3a1 + 4
2
3a2 + 4
2
3an2 + 4
with equality for a1 = a2 = · · · = an = 1, and also for
2
a1 = 2, a2 = · · · = an−3 = 1, an−2 = an−1 = an = .
3
312 Vasile Cîrtoaje
a1 ≥ a2 ≥ 1 ≥ a3 ≥ · · · ≥ an , a1 + a2 + · · · + an = n,
then
a1 (a1 − 1) a2 (a2 − 1) an (an − 1)
+ + · · · + ≥ 0,
4(n − 2)a12 + (n − 1)2 4(n − 2)a22 + (n − 1)2 4(n − 2)an2 + (n − 1)2
where
k
s0 = < 1.
2k + 1
Having in view Note 4, we may apply the LPCF-OV Theorem for 2n real numbers
and m = n. We only need to show that f (x) + n f ( y) ≥ (n + 1) f (1) for x, y ∈ I so
that x + n y = n + 1. Using Note 1, it suffices to prove that h(x, y) ≥ 0. We have
f (u) − f (1) u
g(u) = = ,
u−1 (u + k)2
PCF Method for Ordered Variables 313
g(x) − g( y) k2 − x y
h(x, y) = = ≥ 0,
x−y (x + k)2 ( y + k)2
because
(n + 1)2 (x + n y)2 (x − n y)2
k2 − x y ≥ −xy = −xy = ≥ 0.
4n 4n 4n
n+1
The equality holds for a1 = a2 = · · · = an = 1. If k = p , then the equality holds
2 n
also for
n+1 n+1
a1 = , a2 = · · · = an = 1, an+1 = · · · = a2n = .
2 2n
where
−1
s0 =
∈ (−1, 0).
k
According to Note 4, we may apply the LPCF-OV Theorem for 2n real numbers and
m = n. Thus, we only need to show that f (x) + n f ( y) ≥ (n + 1) f (1) for x, y ∈ I
so that x + n y = n + 1. Using Note 1, it suffices to prove that h(x, y) ≥ 0. We have
f (u) − f (1) u+1
g(u) = = ,
u−1 (u + k)2
314 Vasile Cîrtoaje
g(x) − g( y) (k − 1)2 − 1 − x − y − x y
h(x, y) = = ≥ 0,
x−y (x + k)2 ( y + k)2
because
(n + 1)2 (n y − 1)2
(k − 1)2 − 1 − x − y − x y ≥ −1− x − y − xy = ≥ 0.
n n
n+1
The equality holds for a1 = a2 = · · · = an = 1. If k = 1 + p , then the equality
n
holds also for
1
a1 = n, a2 = · · · = an = 1, an+1 = · · · = a2n = .
n
a1 ≥ 1 ≥ a2 ≥ · · · ≥ an , a1 + a2 + · · · + an = n,
then
3/a1 3/a2
a1 + a2 + · · · + an3/an ≤ n.
x 3/x + y 3/ y ≤ 2,
which is just the inequality in P 3.32 from Volume 2. The equality holds for
a1 = a2 = · · · = an = 1.
then
(1 − a1 + a12 )(1 − a2 + a22 ) · · · (1 − a11 + a11
2
) ≥ 1.
s0 = 1/2.
Also, from
1 + 2u(1 − u)
f 00 (u) = ,
(1 − u + u2 )2
it follows that f 00 (u) > 0 for u ∈ [s0 , 1], hence f is convex on [s0 , 1]. Therefore,
applying the LPCF-OV Theorem for n = 11 and m = 2, we only need to show that
f (x) + 9 f ( y) ≥ 9 f (1) for all real x, y so that x + 9 y = 10. Using Note 2, it suffices
to prove that H(x, y) ≥ 0, where
f 0 (x) − f 0 ( y) 1 + x + y − 2x y
H(x, y) = = .
x−y (1 − x + x 2 )(1 − y + y 2 )
316 Vasile Cîrtoaje
Since
1 + x + y − 2x y = 18 y 2 − 8 y + 1 = 2 y 2 + (4 y − 1)2 > 0,
the conclusion follows. The equality holds for a1 = a2 = · · · = a11 = 1.
Remark. By replacing a1 , a2 , . . . , a11 respectively with 1 − a1 , 1 − a2 , . . . , 1 − a11 , we
get the following statement.
• If a1 , a2 , . . . , a11 are real numbers so that
a1 ≤ a2 ≤ 0 ≤ a3 ≤ · · · ≤ a11 , a1 + a2 + · · · + a11 = 0,
then
(1 − a1 + a12 )(1 − a2 + a22 ) · · · (1 − a11 + a11
2
) ≥ 1,
with equality for a1 = a2 = · · · = an = 0.
a1 ≥ a2 ≥ a3 ≥ a4 ≥ 1 ≥ a5 ≥ a6 ≥ a7 ≥ a8 , a1 + a2 + · · · + a8 = 8,
then
1 1 1 1 1 1
5 2 + 2 + · · · + 2 + 72 ≥ 14 + + ··· + .
a1 a2 a8 a1 a2 a8
(Vasile C., 2012)
where
5
s0 = .
7
Taking into account Note 4, we may apply the LPCF-OV Theorem for n = 8 and
m = 4. We only need to show that f (x) + 4 f ( y) ≥ 5 f (1) for x, y ∈ I so that
x + 4 y = 5. Using Note 1, it suffices to prove that h(x, y) ≥ 0. Indeed, we have
f (u) − f (1) 9 5
g(u) = = − 2,
u−1 u u
PCF Method for Ordered Variables 317
g(x) − g( y) 5(x + y) − 9x y
h(x, y) = =
x−y x2 y2
(x + 4 y)(x + y) − 9x y (x − 2 y)2
= = ≥ 0.
x2 y2 x2 y2
In accordance with Note 3, the equality holds for a1 = a2 = · · · = a8 = 1, and also
for
5 5
a1 = , a2 = a3 = a4 = 1, a5 = a6 = a7 = a8 = .
3 6
a ≤ b ≤ 1 ≤ c ≤ d, abcd = 1,
then
7 − 6a 7 − 6b 7 − 6c 7 − 6d 4
+ + + ≥ .
2+a 2 2+ b 2 2+c 2 2+d 2 3
(Vasile C., 2012)
a = e x , b = e y , c = ez , d = e w ,
where
x + y +z+w
x ≤ y ≤ 0 ≤ z ≤ w, s= = 0,
4
7 − 6eu
f (u) = , u ∈ R.
2 + e2u
As shown in the proof of P 3.26, f is convex on [0, s0 ], is decreasing on (−∞, s0 ]
and increasing on [s0 , ∞), where
s0 = ln 3.
which is clearly true. In accordance with Note 3, the equality holds for a = b =
c = d = 1, and also for
1
a= , b = 1, c = d = 2.
4
a ≤ b ≤ 1 ≤ c, abc = 1,
then
7 − 4a 7 − 4b 7 − 4c
+ + ≥ 3.
2 + a2 2 + b2 2 + c 2
(Vasile C., 2012)
a = e x , b = e y , c = ez ,
s0 = ln 4.
Also, we have
4t · h(t)
f 00 (u) = , t = eu ,
(2 + t 2 )3
where
h(t) = −t 4 + 7t 3 + 12t 2 − 14t − 4.
We will show that h(t) ≥ 0 for t ∈ [1, 4], hence f is convex on [0, s0 ]. Indeed,
(a − 1)4 ≥ 0.
a ≥ 1 ≥ b ≥ c, abc = 1,
then
23 − 8a 23 − 8b 23 − 8c
+ + ≥ 9.
3 + 2a2 3 + 2b2 3 + 2c 2
(Vasile C., 2012)
a = e x , b = e y , c = ez ,
(a − 1)4 ≥ 0.
a1 ≤ · · · ≤ an−1 ≤ 1 ≤ an , a1 a2 · · · an = 1.
If p, q ≥ 0 so that p + 3q ≥ 1, then
1 − a1 1 − a2 1 − an
+ + ··· + ≥ 0.
1 + pa1 + qa1 1 + pa2 + qa2
2 2
1 + pan + qan2
f (x 1 ) + f (x 2 ) + · · · + f (x n ) ≥ n f (s),
where
x1 + x2 + · · · + x n
x 1 ≤ · · · ≤ x n−1 ≤ 0 ≤ x n , s= = 0,
n
1 − eu
f (u) = , u ∈ R.
1 + peu + qe2u
As shown in the proof of P 3.30, if p + 3q − 1 ≥ 0, then f is convex on [0, s0 ], where
v
t p+1
s0 = ln r0 > 0, r0 = 1 + 1 + .
q
In addition, f is decreasing on (−∞, s0 ] and increasing on [s0 , ∞). Therefore,
we may apply the RPCF-OV Theorem for m = n − 1. We only need to show that
f (x) + f ( y) ≥ 2 f (0) for all real x, y so that x + y = 0; that is, to prove that
1−a 1− b
+ ≥0
1 + pa + qa 2 1 + pb + qb2
PCF Method for Ordered Variables 321
−2 ≤ a ≤ b ≤ 1 ≤ c ≤ d ≤ e, a + b + c + d + e = 5,
then
1 1 1 1 1 1 1 1 1 1
2
+ 2+ 2+ 2+ 2≥ + + + + .
a b c d e a b c d e
a+b+c+d+e
f (a) + f (b) + f (c) + f (d) + f (e) ≥ 5 f (s), s= = 1,
5
where
1 1
f (u) = 2
− , u ∈ I = [−2, 7] \ {0}.
u u
Let
s0 = 2, s < s0 .
From
1 1 1 (u − 2)2
f (u) − f (2) = − + = ≥ 0,
u2 u 4 4u2
it follows that
min f (u) = f (s0 ),
u∈I
while from
u−2 2(3 − u)
f 0 (u) =3
, f 00 (u) = ,
u u4
it follows that f is convex on [s, s0 ]. We can’t apply the the RPCF-OV Theorem
because f is not decreasing on I≤s0 . According to Theorem 1 (applied for n = 5 and
m = 2) and Note 6, we may replace this condition with (1+n−m)s−(n−m)s0 ≤ inf I.
Indeed, we have
So, according to Note 1, it suffices to show that h(x, y) ≥ 0 for all x, y ∈ I so that
x + 3 y = 4. We have
f (u) − f (1) −1
g(u) = = 2,
u−1 u
g(x) − g( y) x+y 2(x + 2)
h(x, y) = = 2 2 = ≥ 0.
x−y x y 3x 2 y 2
The proof is completed. By Note 3, the equality holds for a = b = c = d = e = 1,
and also for
a = −2, b = 1, c = d = e = 2.
Chapter 5
Sn = f (x 1 ) + f (x 2 ) + · · · + f (x n )
EV-Lemma. Let a, b, c be fixed nonnegative real numbers, not all equal and, for k ≥ 0,
at most one of them equal to zero, and let x ≤ y ≤ z be nonnegative real numbers so
that
x + y + z = a + b + c, x k + y k + z k = ak + bk + c k ,
323
324 Vasile Cîrtoaje
where k is a real number (k 6= 1); for k = 0, the second equation is x yz = abc. Then,
the range of y is an interval [m, M ] with m < M ; in addition,
(1) y = m if and only if x = y < z;
(2) y = M if and only if 0 = x < y ≤ z or 0 < x ≤ y = z.
a+b+c k
a +b +c ≥3
k k k
, k ∈ (−∞, 0) ∪ (1, ∞),
3
a+b+c k
a +b +c ≤3
k k k
, k ∈ (0, 1),
3
a+b+c 3
abc ≤ , k = 0,
3
where the equality holds if and only if a = b = c.
According to the relations
x + z = a + b + c − y, x k + z k = ak + bk + c k − y k ,
we get
y k−1 − z k−1 y k−1 − x k−1
x0 = ≤ 0, z0 = ≤ 0. (*)
z k−1 − x k−1 x k−1 − z k−1
Let us define the nonnegative functions
f1 ( y) = y − x( y), f2 ( y) = z( y) − y. f3 ( y) = x( y).
Since
these functions are strictly increasing, decreasing and decreasing, respectively. Thus,
the inequality f1 ( y) ≥ 0 (with f1 increasing) involves y ≥ m, where m is a root of
the equation x( y) = y, and the inequality f2 ( y) ≥ 0 (with f2 decreasing) involves
involves y ≤ y2 , where y2 is a root of the equation z( y) = y. If x( y2 ) ≥ 0, then
EV Method for Nonnegative Variables 325
S3 = f (x) + f ( y) + f (z)
Proof. If a = b = c, then
a+b+c
k
a +b +c =3
k k k
,
3
hence
x + y + z k
x k + y k + zk = 3 ,
3
which involves x = y = z. If k > 0 and two of a, b, c are equal to zero, then
a k + b k + c k = (a + b + c)k ,
hence
x k + y k + z k = (x + y + z)k ,
which involves x = y = 0. In both cases, the extremum conditions in the statement
(x = y and either x = 0 or y = z) are satisfied. Consider further that a, b, c are
not all equal and at most one of them is equal to zero. As shown in the proof of the
EV-Lemma, we have x < z. According to the relations
x + z = a + b + c − y, x k + z k = ak + bk + c k − y k ,
F 0 ( y) = x 0 f 0 (x) + f 0 ( y) + z 0 f 0 (z)
y k−1 − z k−1 y k−1 − x k−1
= g(x k−1
) + g( y k−1
) + g(z k−1 ),
z k−1 − x k−1 x k−1 − z k−1
which, for x < y < z, is equivalent to
F 0 ( y) g(x k−1 )
=
( y k−1 − x k−1 )( y k−1 − z k−1 ) (x k−1 − y k−1 )(x k−1 − z k−1 )
we have F 0 ( y) < 0 for y ∈ (m, M ) (see the EV-Lemma), hence F is strictly decreas-
ing on [m, M ]. Therefore, F is maximal for y = m (when 0 ≤ x = y ≤ z) and is
minimal for y = M (when x = 0 or 0 < x ≤ y = z.
This result contradicts the assumption that Sn attains its maximum at (b1 , b2 , . . . , bn )
with b1 < bn−1 .
(b) Similarly, we can prove that Sn is minimal for n ≥ 4 when either x 1 = 0 or
0 < x1 ≤ x2 = · · · = x n.
0 ≤ x1 ≤ x2 ≤ · · · ≤ x n
EV Method for Nonnegative Variables 327
so that
x 1 + x 2 + · · · + x n = a1 + a2 + · · · + a n ,
x 12 + x 22 + · · · + x n2 = a12 + a22 + · · · + an2 .
Let f be a real-valued function, continuous on [0, ∞) and differentiable on (0, ∞),
so that the joined function
g(x) = f 0 (x)
is strictly convex on (0, ∞). The sum
Sn = f (x 1 ) + f (x 2 ) + · · · + f (x n )
0 < x1 ≤ x2 ≤ · · · ≤ x n
so that
x 1 + x 2 + · · · + x n = a1 + a2 + · · · + a n ,
1 1 1 1 1 1
+ + ··· + = + + ··· + .
x1 x2 xn a1 a2 an
Let f be a real-valued function, continuous and differentiable on (0, ∞), so that the
joined function
1
g(x) = f p
0
x
is strictly convex on (0, ∞). The sum
Sn = f (x 1 ) + f (x 2 ) + · · · + f (x n )
x1 ≤ x2 = x3 = · · · = x n.
0 ≤ x1 ≤ x2 ≤ · · · ≤ x n
so that
x 1 + x 2 + · · · + x n = a1 + a2 + · · · + an , x 1 x 2 · · · x n = a1 a2 · · · an .
328 Vasile Cîrtoaje
Sn = f (x 1 ) + f (x 2 ) + · · · + f (x n )
0 ≤ x1 ≤ x2 ≤ · · · ≤ x n
so that
0 < x1 ≤ x2 = x3 = · · · = x n,
0 < x 1 = x 2 = · · · = x n−1 ≤ x n ;
x 1 = x 2 = · · · = x n−1 ≤ x n ,
Note 1. The EV-Theorem, Corollary 1 and Corollary 3 are also valid for the cases
when x 1 , x 2 , . . . , x n > 0, f is continuous and differentiable on (0, ∞), f (0+) =
±∞ and the sum Sn has a global maximum (minimum).
From the EV-Theorem and Note 1, we can obtain some interesting particular
results, which are useful in many applications.
0 ≤ x1 ≤ x2 ≤ · · · ≤ x n
so that
Let us denote
Sn = x 1m + x 2m + · · · + x nm .
Case 1 : k < 0.
(a) If m ∈ (k, 0) ∪ (1, ∞), then Sn is maximal only for
0 < x 1 = x 2 = · · · = x n−1 ≤ x n ,
0 < x1 ≤ x2 = x3 = · · · = x n.
0 < x 1 = x 2 = · · · = x n−1 ≤ x n ,
0 < x1 ≤ x2 = x3 = · · · = x n.
0 ≤ x 1 = x 2 = · · · = x n−1 ≤ x n ,
0 < x 1 = x 2 = · · · = x n−1 ≤ x n ,
0 < x1 ≤ x2 = x3 = · · · = x n.
0 ≤ x 1 = x 2 = · · · = x n−1 ≤ x n ,
Case 3 : k > 1.
(a) If m ∈ (0, 1) ∪ (k, ∞), then Sn is maximal only for
0 ≤ x 1 = x 2 = · · · = x n−1 ≤ x n ,
330 Vasile Cîrtoaje
0 < x 1 = x 2 = · · · = x n−1 ≤ x n ,
0 < x1 ≤ x2 = x3 = · · · = x n.
0 ≤ x 1 = x 2 = · · · = x n−1 ≤ x n ,
We have
f 0 (u) = m2 (m − 2)(m − k)um−1
and
0 ≤ x1 ≤ x2 ≤ · · · ≤ x n
so that
p p p p q q q q
x 1 + x 2 + · · · + x np = a1 + a2 + · · · + anp , x 1 + x 2 + · · · + x nq = a1 + a2 + · · · + anq ,
where
p, q ∈ {1, 2, 3}, p 6= q.
The symmetric sum X
Sn = x i1 x i2 x i3
1≤i1 <i2 <i3 ≤n
Corollary 6 is a consequence
P of Corollary 5. For p = 2 and q = 3, according
P to
this identity, the sum 1≤i1 <i2 <i3 ≤n x i1 x i2 x i3 is maximal/minimal when x 1 is max-
imal/minimal. Therefore, we need to show that if
0 ≤ x 1 = x 2 = · · · = x n−1 ≤ x n ,
Note 2. The EV-Theorem and Corollaries 1-3 can be extended to the cases where:
(a) x 1 , x 2 , . . . , x n ≥ m ≥ 0, f is continuous on [m, ∞) and differentiable on
1
(m, ∞), and g(x) is strictly convex for x k−1 > m; so, the sum
Sn = f (x 1 ) + f (x 2 ) + · · · + f (x n )
Sn = f (x 1 ) + f (x 2 ) + · · · + f (x n )
Note 3. The EV-Theorem and Corollaries 1-3 can be extended to the cases where:
(a) x 1 , x 2 , . . . , x n > m ≥ 0, f is continuous and differentiable on (m, ∞),
1
f (m+) = ±∞, g(x) is strictly convex for x k−1 > m and the sum Sn has a global
maximum (minimum);
(b) 0 ≤ x 1 , x 2 , . . . , x n < M , f is continuous and differentiable on [0, M ),
1
f (M −) = ±∞, g(x) is strictly convex for x k−1 < M and the sum Sn has a global
maximum (minimum).
332 Vasile Cîrtoaje
EV Method for Nonnegative Variables 333
5.2 Applications
a + b + c + d = a3 + b3 + c 3 + d 3 = 2,
then
7
≤ a2 + b2 + c 2 + d 2 ≤ 2.
4
then
14
3 ≤ a13 + a23 + · · · + a93 ≤ .
3
27
a + b + c + d = a2 + b2 + c 2 + d 2 = ,
7
then
5427 1377
≤ a3 + b3 + c 3 + d 3 ≤ .
1372 343
bc ca ab 3
+ 2 + 2 ≤ .
3a2 +b +c
2 2 3b + c + a
2 2 3c + a + b
2 2 5
then
abc + bcd + cda + dab ≤ 1.
then
a12 + a22 + · · · + an2 ≤ p2 + q2 .
1 1 1 3 a+b+c
+ + ≤ + .
a + b b + c c + a 2(a + b + c) 3
EV Method for Nonnegative Variables 335
a2 + b2 + c 2 = 3,
then
1 1 1 a+b+c 11
+ + + ≥ .
a+b b+c c+a 9 2(a + b + c)
a + b + c = 4,
then
1 1 1 15
+ + ≥ .
a + b b + c c + a 8 + ab + bc + ca
5.16. If a, b, c are nonnegative real numbers, no two of which are zero, then
1 1 1 1 2
+ + ≥ +p .
a+b b+c c+a a+b+c ab + bc + ca
5.17. If a, b, c are nonnegative real numbers, no two of which are zero, then
p p
1 1 1 3− 3 2+ 3
+ + ≥ + p .
a+b b+c c+a a + b + c 2 ab + bc + ca
5.18. Let a, b, c be nonnegative real numbers, no two of which are zero, so that
ab + bc + ca = 3.
If p
9+5 3
0≤k≤ ≈ 2.943,
6
then
2 2 2 9(1 + k)
+ + ≥ .
a+b b+c c+a a + b + c + 3k
336 Vasile Cîrtoaje
5.19. If a, b, c are nonnegative real numbers, no two of which are zero, then
1 1 1 20
+ + ≥ p .
a+b b+c c+a a + b + c + 6 ab + bc + ca
then
51 a b c
≤ + + ≤ 2.
28 b+c c+a a+b
176
abc + bcd + cda + dab ≤ 1 + abcd.
81
a2 b2 c 2 + b2 c 2 d 2 + c 2 d 2 a2 + d 2 a2 b2 + 2(abcd)3/2 ≤ 6.
EV Method for Nonnegative Variables 337
a2 b2 + b2 c 2 + c 2 a2 + a3 b3 + b3 c 3 + c 3 a3 ≥ 6abc.
0 ≤ k ≤ 15,
then
1 1 1 k 9+k
+ + + ≥ .
(a + b)2 (b + c)2 (c + a)2 (a + b + c)2 4(ab + bc + ca)
5.31. If a, b, c are nonnegative real numbers, no two of which are zero, then
1 1 1 24 8
+ + + ≥ .
(a + b)2 (b + c)2 (c + a)2 (a + b + c)2 ab + bc + ca
5.32. If a, b, c are nonnegative real numbers, no two of which are zero, so that
then
1 1 1 9k 3
+ + + ≥ + k.
(a + b)2 (b + c)2 (c + a)2 (a + b + c)2 4
338 Vasile Cîrtoaje
5.33. If a, b, c are nonnegative real numbers, no two of which are zero, then
2 2 2 8 1
(a) + 2 + 2 ≥ 2 + ;
a2 +b 2 b +c 2 c +a 2 a +b +c
2 2 ab + bc + ca
2 2 2 7 6
(b) + 2 + 2 ≥ 2 + ;
a2 +b 2 b +c 2 c +a 2 a +b +c
2 2 (a + b + c)2
2 2 2 45
(c) + 2 + 2 ≥ .
a2 +b 2 b +c 2 c +a 2 4(a + b + c ) + ab + bc + ca
2 2 2
5.34. If a, b, c are nonnegative real numbers, no two of which are zero, then
1 1 1 3 4
+ 2 + 2 + 2 ≥ .
a2 +b 2 b +c 2 c +a 2 a +b +c
2 2 ab + bc + ca
5.35. If a, b, c are nonnegative real numbers, no two of which are zero, then
3 3 3 5 4
(a) + + ≥ + ;
a2 + ab + b2 b2 + bc + c 2 c 2 + ca + a2 ab + bc + ca a2 + b2 + c 2
3 3 3 1 24
(b) + 2 + 2 ≥ + ;
a2 + ab + b 2 b + bc + c 2 c + ca + a 2 ab + bc + ca (a + b + c)2
1 1 1 21
(c) + + ≥ .
a2 + ab + b2 b2 + bc + c 2 c 2 + ca + a2 2(a2 + b2 + c 2 ) + 5(ab + bc + ca)
1 1 1 85
+ + ≤ .
(a + b)2 (b + c)2 (c + a)2 36(ab + bc + ca)
1 1 1 3(a2 + b2 + c 2 )
+ + ≤ .
(a + b)2 (b + c)2 (c + a)2 4(ab + bc + ca)
EV Method for Nonnegative Variables 339
2
5.39. Let a, b, c ≥ so that a + b + c = 3. Then,
5
1 1 1 3
+ + ≤ .
3 + 2(a + b ) 3 + 2b + c ) 3 + 2(c + a ) 7
2 2 2 2 2 2
5.46. If a, b, c are nonnegative real numbers, no two of which are zero, so that
a4 + b4 + c 4 = 3, then
1 1 1 3
+ + ≥ .
a5 + b5 b5 + c 5 c 5 + a5 2
340 Vasile Cîrtoaje
then
1 1 1
+ + ··· + ≥ 1.
1 + (n − 1)a1 1 + (n − 1)a2 1 + (n − 1)an
1 1 1 50
4 + + + ≥ 27.
a b c a+b+c
1 1 1
a + b + c + 15 ≥ 6
3 3 3
+ + .
a b c
1 1 1
5.63. If a, b, c are positive real numbers so that + + = 3, then
a b c
4(a2 + b2 + c 2 ) + 9 ≥ 21abc.
1 1 1
5.64. If a1 , a2 , . . . , an are positive real numbers so that + + ··· + = n,
a1 a2 an
then,
a1 + a2 + · · · + an − n ≤ en−1 (a1 a2 · · · an − 1),
where n−1
1
en−1 = 1 + .
n−1
1 1 1 1
(a1 + a2 + · · · + an − n) + + ··· + − n + a1 a2 · · · an + ≥ 2.
a1 a2 an a1 a2 · · · an
EV Method for Nonnegative Variables 343
1 1
−Ç < 1.
a1 + a2 + · · · + an − n
p
1 1 1
a1 + a2 + ··· + an −n
n2 (n − 2) 1 1 1
a1n−1 + a2n−1 + ··· + ann−1 + ≥ (n − 1) + + ··· + .
a1 + a2 + · · · + an a1 a2 an
abc − 1 2
(a + b + c − 3)2 ≥ (a + b2 + c 2 − 3).
abc + 1
5.74. If a1 , a2 , . . . , an are positive real numbers so that a13 + a23 + · · · + an3 = n, then
a1 + a2 + · · · + an ≥ n n+1 a1 a2 · · · an .
p
ln 4
k ≥2− ≈ 0.738,
ln 3
then
a k + b k + c k ≥ 3.
344 Vasile Cîrtoaje
ln 9 − ln 8
k≥ ≈ 0.29,
ln 3 − ln 2
then
a k + b k + c k ≥ ab + bc + ca.
a + b + c ≥ 2, ab + bc + ca ≥ 1,
then p p p
3
a+ b+
3 3
c ≥ 2.
then
a16 + a26 + · · · + an6 ≤ n.
EV Method for Nonnegative Variables 345
a + b + c + d = 4,
then
(a4 + b4 + c 4 + d 4 )2 ≥ (a2 + b2 + c 2 + d 2 )(a5 + b5 + c 5 + d 5 ).
a + b + c + d = 4,
then
13(a2 + b2 + c 2 + d 2 )2 ≥ 12(a4 + b4 + c 4 + d 4 ) + 160.
346 Vasile Cîrtoaje
then v v p
t3 s a 1+ 7
t b s
c
3 ≤ + + ≤ p .
5 b+c c+a a+b 2
then
19 a b c 141
≤ + + ≤ .
12 b+c c+a a+b 88
x 3 + y 3 + z 3 = a3 + b3 + c 3 .
Then,
(a + b + c)(x + y + z) p
3
≥ 3.
ab + bc + ca + x y + yz + z x
a1 + a2 + · · · + an = n,
then v
1 X
p u
a1 + a2 + · · · + an ≤ 2n − 1 + 2 1 −
p p t ai a j .
n i< j
then
(n − 1)(n − 2) Xp
(a1 + a2 + · · · + an ) + ai a j ≥ n(n − 1).
2 i< j
5.99. Let
Then,
1 1 1
F (a1 , a2 , . . . , an ) ≥ F , ,..., .
a1 a2 an
5.100. Let
F (a1 , a2 , . . . , an ) = a1 + a2 + · · · + an − n n a1 a2 · · · an ,
p
a1 (a2 + a3 + · · · + an ) ≥ n − 1.
Then,
1 1 1
F (a1 , a2 , . . . , an ) ≥ F , ,..., .
a1 a2 an
348 Vasile Cîrtoaje
5.101. Let
v
t a2 + a2 + · · · + a2
1 2 n a1 + a2 + · · · + an
F (a1 , a2 , . . . , an ) = − ,
n n
where a1 , a2 , . . . , an are positive real numbers such that a1 ≤ a2 ≤ · · · ≤ an and
a1n−1 (a2 + a3 + · · · + an ) ≥ n − 1.
Then,
1 1 1
F (a1 , a2 , . . . , an ) ≥ F , ,..., .
a1 a2 an
a1 + a2 + · · · + an = n, an = max{a1 , a2 , . . . , an },
then
1 1 1
n + + ··· + ≥ 4(a12 + a22 + · · · + an2 ) + n(n − 5).
a1 a2 an−1
1 1 1
+ + ≤ 1.
a+ b+1 b+c+1 c+a+1
EV Method for Nonnegative Variables 349
5.3 Solutions
a + b + c + d = a3 + b3 + c 3 + d 3 = 2,
then
7
≤ a2 + b2 + c 2 + d 2 ≤ 2.
4
(Vasile C., 2010)
(a2 + b2 + c 2 + d 2 )2 ≤ (a + b + c + d)(a3 + b3 + c 3 + d 3 ).
a+b+c+d =2 , a3 + b3 + c 3 + d 3 = 2, a ≤ b ≤ c ≤ d,
then
S4 = a2 + b2 + c 2 + d 2
is minimal for a = b = c.
So, we only need to prove that the equations
3a + d = 3a3 + d 3 = 2, a, d ≥ 0,
imply
7
≤ 3a2 + d 2 .
4
Indeed, from 3a + d = 3a3 + d 3 = 2, we get a = 1/4 and d = 5/4, when
7
3a2 + d 2 = .
4
The left inequality is an equality for
1 5
a=b=c= , d=
4 4
(or any cyclic permutation).
350 Vasile Cîrtoaje
then
14
3 ≤ a13 + a23 + · · · + a93 ≤ .
3
(Vasile C., 2010)
a1 ≤ a2 ≤ · · · ≤ a9 ,
then
S9 = a13 + a23 + · · · + a93
is maximal for a1 = a2 = · · · = a8 ≤ a9 .
Thus, we only need to prove that the equations
8a + b = 3, 8a2 + b2 = 3, a, b ≥ 0,
involve
14
8a3 + b3 ≤
.
3
Indeed, from the equations above, we get a = 1/6 and b = 5/3, when
1 125 14
8a3 + b3 = + = .
27 27 3
The equality holds for
1 5
a1 = a2 = · · · = a8 = , a9 =
6 3
(or any cyclic permutation).
EV Method for Nonnegative Variables 351
3(b2 + c 2 + d 2 ) ≥ b + c + d)2 .
Solution. Substituting
x + y + z = const ant , x yz = 1, x ≤ y ≤ z,
then
S3 = x 7/5 + y 7/5 + z 7/5
is maximal for x = y.
So, it suffices to prove the original inequality for a = b. Write this inequality in
the homogeneous form
(a5 + b5 + c 5 )2 ≥ 3abc(a7 + b7 + c 7 ).
We only need to prove this inequality for a = b = 1; that is, to show that f (c) ≥ 0,
where
f (c) = (c 5 + 2)2 − 3c(c 7 + 2), c > 0.
We have
f 0 (c) = 10c 4 (c 5 + 2) − 24c 7 − 6,
f 00 (c) = 2c 3 g(t), g(t) = 45c 5 − 84c 3 + 40.
By the AM-GM inequality, we get
Æ
5
g(t) = 15c 5 + 15c 5 + 15c 5 + 20 + 20 − 84c 3 ≥ 5 (15c 5 )3 · 202 − 84c 3
p5
p5
= 27 · 16 25 − 14 18 c 3 > 0,
hence f 00 (c) > 0, f 0 (c) is increasing. Since f 0 (0) = 1, it follows that f 0 (c) ≤ 0 for
c ≤ 1, f 0 (c) ≥ 0 for c ≥ 1, therefore f is decreasing on (0, 1] and increasing on
[1, ∞); consequently, f (c) ≥ f (1) = 0. The equality occurs for a = b = c = 1.
354 Vasile Cîrtoaje
Solution. Substituting
x + y + z + t = const ant , x yz t = 1, x ≤ y ≤ z ≤ t,
then
S4 = x 4/3 + y 4/3 + z 4/3 + t 4/3
is maximal for x = y = z.
Therefore, it suffices to prove the original inequality for a = b = c. Write the
original inequality in the homogeneous form
p
(a3 + b3 + c 3 + d 3 )2 ≥ 4 abcd (a4 + b4 + c 4 + d 4 ).
We only need to prove this inequality for a = b = c = 1; that is, to show that
p
(d 3 + 3)2 ≥ 4 d (d 4 + 3).
p
Putting u = d, we have
p
(d 3 + 3)2 − 4 d (d 4 + 3) = (u6 + 3)2 − 4u(u8 + 3)
= (u3 − 1)4 + 4(u + 2)(u − 1)2 ≥ 0.
3a = 2b = 2c
Solution. For the nontrivial case abc 6= 0, we can write the desired inequalities in
the form
m
f (a) + f (b) + f (c) + ≥ 0,
abc
where
−1
f (u) = , k ∈ {2, 3, 4}, u > 0.
u(u2 + k)
We have f (0+) = −∞ and
3u2 + k
f 0 (u) = ,
u2 (u2 + k)2
kx 6 + 3x 4
g(x) = f 0 (1/x) = ,
(kx 2 + 1)2
2x 2 (k3 x 6 + 4k2 x 4 − 3kx 2 + 18)
g 00 (x) = .
(kx 2 + 1)4
358 Vasile Cîrtoaje
Since
k3 x 6 + 4k2 x 4 − 3kx 2 + 18 > 4k2 x 4 − 3kx 2 + 18 > 0,
we have g 00 (x) > 0, hence g is strictly convex on (0, ∞). According to Corollary 3
and Note 1, if
bc ca ab 1
+ + ≤
9a2 + 2(a + b + c)2 9b2 + 2(a + b + c)2 9c 2 + 2(a + b + c)2 8
for b = c = 1, that is
1 2a 1
+ ≤ ,
11a2 + 8a + 8 2a2 + 8a + 17 8
2a a(11a + 8)
≤ ,
2a2 + 8a + 17 8(11a2 + 8a + 8)
a(22a3 − 72a2 + 123a + 8) ≥ 0.
Since
the conclusion follows. The equality holds for a = 0 and b = c = 3/2 (or any cyclic
permutation).
(b) Let
p p
11 33 − 45 33 − 5
m= ≈ 0.253, r= ≈ 0.186.
72 4
We only need to prove the homogeneous inequality
bc ca ab
+ 2 + 2 ≤m
3a2 + (a + b + c)2 3b + (a + b + c)2 3c + (a + b + c)2
1 2a
f (a) = + .
4(a2 + a + 1) a2 + 4a + 7
EV Method for Nonnegative Variables 359
We have
Since f 0 (a) ≥ 0 for a ∈ [0, r], and f 0 (a) ≤ 0 for a ∈ [r, ∞), f is increasing on
[0, r] and decreasing on [r, ∞); therefore,
f (a) ≥ f (r) = m.
bc ca ab 1
+ + ≤
9a2 + 4(a + b + c)2 9b2 + 4(a + b + c)2 9c 2 + 4(a + b + c)2 15
for b = c = 1, that is
1 2a 1
+ 2 ≤ ,
13a2 + 16a + 16 4a + 16a + 25 15
then
abc + bcd + cda + dab ≤ 1.
hence abc ≤ 1. Consider further that a, b, c, d > 0 and use the contradiction
method. Assume that
abc + bcd + cda + dab > 1,
and prove that
(3a + 1)(3b + 1)(3c + 1) > 64.
It suffices to show that
abc + bcd + cda + dab ≥ 1
involves
(3a + 1)(3b + 1)(3c + 1) ≥ 64.
Replacing a, b, c, d by 1/a, 1/b, 1/c, 1/d, we need to show that
a + b + c + d = abcd
involves
3 3 3 3
+1 +1 +1 + 1 ≥ 64,
a b c d
which is equivalent to
where
3
f (u) = − ln +1 , u > 0.
u
We have f (0+) = −∞ and
3x 2 6
g(x) = f 0 (1/x) = , g 00 (x) = > 0,
3x + 1 (3x + 1)3
then
S4 = f (a) + f (b) + f (c) + f (d)
is maximal for a = b = c.
EV Method for Nonnegative Variables 361
for 3a + d = a3 d, that is
3 a3 − 1
= , 1 < a ≤ d.
d a
Write this inequality as
(3 + a)3 (3 + d) ≥ 64a3 d,
(3 + a)4 (3 + d) ≥ 64a3 d(3 + a),
a−1 4
4 1+ (3 + d) ≥ a3 d(3 + a).
4
By Bernoulli’s inequality, we have
a−1 4 a−1
1+ ≥1+4· = a.
4 4
which is equivalent to
12
≥ a3 + 3a2 − 4,
d
4(a3 − 1)
≥ a3 + 3a2 − 4,
a
a4 − a3 − 4a + 4 ≤ 0,
(a − 1)(a3 − 4) ≤ 0.
This is true if a3 ≤ 4. Indeed, we have
3 3 3 a3 − 1 4 − a3
0≤ − = − = .
a d a a a
The proof is completed. The original inequality is an equality for
a = b = c = 1, d =0
then
a12 + a22 + · · · + an2 ≤ p2 + q2 .
(Vasile C., 2013)
Solution. For n = 2, the inequality is an equality. Consider now that n ≥ 3 and
a1 ≤ a2 ≤ · · · ≤ an . We will apply Corollary 5 for k = 3 and m = 2:
• If a1 , a2 , . . . , an are nonnegative real numbers so that a1 ≤ a2 ≤ · · · ≤ an and
then
Sn = a12 + a22 + · · · + an2
is maximal for either a1 = 0 or a2 = a3 = · · · = an .
In the first case a1 = 0, the conclusion follows by induction method. In the
second case, for
a1 = a, a2 = a3 = · · · = an = b,
we need to show that
a2 + (n − 1)b2 ≤ p2 + q2
for
a + (n − 1)b = p + q, a3 + (n − 1)b3 = p3 + q3 .
Since
2(p3 + q3 )
3(p2 + q2 ) = (p + q)2 + ,
p+q
the inequality can be written as
2[a3 + (n − 1)b3 ]
3a2 + 3(n − 1)b2 ≤ [a + (n − 1)b]2 + ,
a + (n − 1)b
which is equivalent to
1
f (a) + f (b) + f (c) + p (a + b + c)2 ≤ 0,
3
where p p
f (u) = −u 4 − u2 , 0≤u≤ 3.
We have
2(x 2 − 2)
g(x) = f 0 (x) = p ,
4 − x2
48
g 00 (x) = .
(4 − x 2 )5/2
p
Since g 00 (x) > 0 for x ∈ (0, 2), g is strictly convex on [0, 3]. According to Corol-
lary 1, if
a + b + c = const ant , a2 + b2 + c 2 = 3 , 0 ≤ a ≤ b ≤ c,
This is true if
c 2 (4c 2 + 5)(c 2 + 8) ≥ (2c 3 + 8c − 1)2 ,
which is equivalent to
5c 4 + 4c 3 − 24c 2 + 16c − 1 ≥ 0,
1 1 1 3 a+b+c
+ + ≤ + .
a + b b + c c + a 2(a + b + c) 3
1 1 1 3 a+b+c
+ + ≤ + .
a + b b + c c + a 2(a + b + c) ab + bc + ca
a + b + c = 1, 0 ≤ a ≤ b ≤ c, ab + bc + ca > 0.
3 1
f (a) + f (b) + f (c) ≤ + ,
2 ab + bc + ca
where
1
f (u) = , 0 ≤ u < 1.
1−u
We will apply Corollary 1 to the function f , which satisfies f (1−) = ∞ and
1
g(x) = f 0 (x) = ,
(1 − x)2
6
g 00 (x) = .
(1 − x)4
Since g 00 (x) > 0, g is strictly convex on [0, 1). According to Corollary 1 and Note
3, if
a+b+c =1 , ab + bc + ca = const ant , 0 ≤ a ≤ b ≤ c,
then the sum
S3 = f (a) + f (b) + f (c)
is maximal for a = b ≤ c. Thus, we only need to prove the homogeneous inequality
for a = b = 1 and c ≥ 1; that is,
4 3 2(c + 2)
1+ ≤ + ,
c+1 c+2 2c + 1
which reduces to
(c − 1)2 ≥ 0.
The original inequality is an equality for a = b = c = 1.
EV Method for Nonnegative Variables 365
Solution. Proceeding in the same manner as in the proof of the preceding P 5.12,
we only need to prove the homogeneous inequality
1 1 1 3 a+b+c
+ + ≥ +
a+b b+c c+a a + b + c 2(ab + bc + ca)
for a = 0 and for a ≤ b = c = 1.
Case 1: a = 0. The homogeneous inequality reduces to
1 1 2 b+c
+ ≥ + ,
b c b+c 2bc
which is equivalent to
(b − c)2 ≥ 0.
Case 2: a ≤ b = c = 1. The homogeneous inequality becomes
1 2 3 a+2
+ ≥ + ,
2 a + 1 a + 2 2(2a + 1)
1 a+2 3 2
− ≥ − ,
2 2(2a + 1) a + 2 a + 1
a−1 a−1
≥ ,
2(2a + 1) (a + 1)(a + 2)
a(a − 1)2 ≥ 0.
The equality holds for a = b = c = 1, and also for
p
a = 0, b=c= 3
a2 + b2 + c 2 = 3,
then
1 1 1 a+b+c 11
+ + + ≥ .
a+b b+c c+a 9 2(a + b + c)
(Vasile C., 2010)
366 Vasile Cîrtoaje
Solution. Using the same method as in the proof of P 5.12, we only need to prove
the homogeneous inequality
1 1 1 a+b+c 11
+ + + ≥
a + b b + c c + a 3(a + b + c ) 2(a + b + c)
2 2 2
1 1 1 b+c 11
+ + + ≥ ,
b c b + c 3(b + c ) 2(b + c)
2 2
b+c b+c 9
+ ≥ ,
bc 3(b + c ) 2(b + c)
2 2
1 1 9
(b + c)2
+ ≥ .
bc 3(b2 + c 2 ) 2
Using the substitution
b2 + c 2
x= , x ≥ 2,
bc
the inequality becomes
1 9
(x + 2) 1 + ≥ ,
3x 2
which is equivalent to
6x 2 − 13x + 4 ≥ 0,
x + 2(x − 2)(3x − 1) ≥ 0.
1 2 a+2 11
+ + ≥ ,
2 a + 1 3(a + 2) 2(a + 2)
2
a+2 a2 − 4a − 1
+ ≥ 0,
3(a2 + 2) 2(a + 1)(a + 2)
3a4 − 10a3 + 13a2 − 8a + 2 ≥ 0,
(a − 1)2 (3a2 − 4a + 2) ≥ 0,
a + b + c = 4,
then
1 1 1 15
+ + ≥ .
a + b b + c c + a 8 + ab + bc + ca
(Vasile C., 2010)
Solution. Using the same method as in P 5.12, we only need to prove the homo-
geneous inequality
2 2 2 15(a + b + c)
+ + ≥
a + b b + c c + a (a + b + c)2 + 2(ab + bc + ca)
for a = 0 and for a ≤ b = c = 1.
Case 1: a = 0. The homogeneous inequality reduces to
2(b + c) 2 15(b + c)
+ ≥ ,
bc b+c (b + c)2 + 2bc
2(b + c)2 15(b + c)2
+2≥ .
bc (b + c)2 + 2bc
Using the substitution
(b + c)2
x= , x ≥ 4,
bc
the inequality becomes
15x
2x + 2 ≥ ,
x +2
which is equivalent to
2x 2 − 9x + 4 ≥ 0,
(x − 4)(2x − 1) ≥ 0.
Case 2: a ≤ 1, b = c = 1. The homogeneous inequality becomes
4 15(a + 2)
1+ ≥ ,
a + 1 (a + 2)2 + 2(2a + 1)
a+5 15(a + 2)
≥ 2 ,
a + 1 a + 8a + 6
a(a − 1)2 ≥ 0.
The equality holds for a = b = c = 4/3, and also for
a = 0, b=c=2
P 5.16. If a, b, c are nonnegative real numbers, no two of which are zero, then
1 1 1 1 2
+ + ≥ +p .
a+b b+c c+a a+b+c ab + bc + ca
(Vasile C., 2010)
Solution. Using the same method as in P 5.12, we only need to prove the desired
homogeneous inequality for a = 0 and for 0 < a ≤ b = c = 1.
Case 1: a = 0. The inequality reduces to the obvious form
1 1 2
+ ≥p .
b c bc
Case 2: 0 < a ≤ 1 = b = c. The inequality becomes
1 2 1 2
+ ≥ +p ,
2 a+1 a+2 2a + 1
1 1 2 2
− ≥p − ,
2 a+2 2a + 1 a + 1
p
a 2(a + 1 − 2a + 1)
≥ p ,
2(a + 2) (a + 1) 2a + 1
a 2a2
≥ p p .
2(a + 2) (a + 1) 2a + 1 (a + 1 + 2a + 1)
Since
p p p p p
2a + 1 (a + 1 + 2a + 1) ≥ 2a + 1( 2a + 1 + 2a + 1) = 2(2a + 1),
P 5.17. If a, b, c are nonnegative real numbers, no two of which are zero, then
p p
1 1 1 3− 3 2+ 3
+ + ≥ + p .
a+b b+c c+a a + b + c 2 ab + bc + ca
P 5.18. Let a, b, c be nonnegative real numbers, no two of which are zero, so that
ab + bc + ca = 3.
If p
9+5 3
0≤k≤ ≈ 2.943,
6
then
2 2 2 9(1 + k)
+ + ≥ .
a+b b+c c+a a + b + c + 3k
(Vasile Cirtoaje and Lorian Saceanu, 2014)
Solution. From
(a + b + c)2 ≥ 3(ab + bc + ca),
we get
a + b + c ≥ 3.
Let p
9+5 3
m= , m ≥ k.
6
We claim that
1+m 1+k
≥ .
a + b + c + 3m a + b + c + 3k
Indeed, this inequality is equivalent to the obvious inequality
(m − k)(a + b + c − 3) ≥ 0.
2 2 2 9(1 + m)
+ + ≥ ,
a+b b+c c+a a + b + c + 3m
which can be rewritten in the homogeneous form
2 2 2 9(1 + m)
+ + ≥ .
a+b b+c c+a
p
a + b + c + m 3(ab + bc + ca)
2 2 2 9(1 + m)
+ + ≥ p .
b c b+c b + c + m 3bc
Substituting
b+c
x= p , x ≥ 2,
bc
EV Method for Nonnegative Variables 371
4 9(1 + m)
1+ ≥ .
a + 1 a + 2 + m 3(2a + 1)
p
P 5.19. If a, b, c are nonnegative real numbers, no two of which are zero, then
1 1 1 20
+ + ≥ p .
a+b b+c c+a a + b + c + 6 ab + bc + ca
Solution. The proof is similar to the one of P 5.12. Finally, we only need to prove
the inequality for a = 0 and for a ≤ b = c = 1.
Case 1: a = 0. The inequality reduces to
1 1 1 20
+ + ≥ p .
b c b+c b + c + 6 bc
Substituting
b+c
x= p , x ≥ 2,
bc
the inequality becomes
1 20
x+ ≥ ,
x x +6
x 3 + 6x 2 − 19x + 6 ≥ 0,
(x − 2)(x 2 + 8x − 3) ≥ 0.
1 2 20
+ ≥ p .
2 a + 1 a + 2 + 6 2a + 1
then
51 a b c
≤ + + ≤ 2.
28 b+c c+a a+b
(Vasile C., 2008)
a + b + c = 1, 0 < a ≤ b ≤ c < 1.
Since g 00 (x) > 0, g is strictly convex on [0, 1). According to Corollary 1 and Note
3, if
11
a+b+c =1 , a2 + b2 + c 2 = , 0 ≤ a ≤ b ≤ c < 1,
25
then the sum
S3 = f (a) + f (b) + f (c)
is maximal for a = b ≤ c, and is minimal for either a = 0 or 0 < a ≤ b = c. Note
that the case a = 0 is not possible because it involves 7(b2 + c 2 ) = 11bc, which is
false.
(1) To prove the right original inequality for a = b ≤ c, let us denote
c
t= , t ≥ 1.
a
The hypothesis 7(a2 + b2 + c 2 ) = 11(ab + bc + ca) involves t = 3, hence
a b c 2a c 2 t
+ + = + = + = 2.
b+c c+a a+b a + c 2a 1 + t 2
374 Vasile Cîrtoaje
c
The right inequality is an equality for a = b = (or any cyclic permutation).
3
(2) To prove the left original inequality for 0 < a ≤ b = c, let us denote
a
t= , 0 < t ≤ 1.
b
1
The hypothesis 7(a2 + b2 + c 2 ) = 11(ab + bc + ca) involves t = , hence
7
a b c a 2b t 2 51
+ + = + = + = .
b + c c + a a + b 2b a + b 2 t + 1 28
The left inequality is an equality for 7a = b = c (or any cyclic permutation).
a1 ≤ a2 ≤ · · · ≤ an .
a22 + · · · + an2 1
≥ .
(a2 + · · · + an )2 n−1
a1 + a2 + · · · + an = n + 1,
which implies
a12 + a22 + · · · + an2 = n + 3.
EV Method for Nonnegative Variables 375
involves
2n − 1 1 1 1 3n2
≤ + + ··· + ≤ .
2 a1 a2 an 2(n + 2)
We apply Corollary 5 for k = 2 and m = −1:
• If a1 , a2 , . . . , an are positive real numbers so that 0 < a1 ≤ a2 ≤ · · · ≤ an and
then
1 1 1
Sn = + + ··· +
a1 a2 an
is minimal for
0 < a1 = a2 = · · · = an−1 ≤ an ,
and is maximal for
a1 ≤ a2 = a3 = · · · = an .
(1) To prove the left original inequality, we only need to consider the case
a1 = a2 = · · · = an−1 ≤ an .
The hypothesis
a12 + a22 + · · · + an2 a + a + · · · + a 2
1 2 n
=
n+3 n+1
implies
(n − 1)a12 + an2
2
(n − 1)a1 + an
= ,
n+3 n+1
(2a1 − an )[2a1 − (n + 2)an ] = 0,
an
a1 = ,
2
hence
1 1 1 n−1 1
(a1 + a2 + · · · + an ) + + ··· + = [(n − 1)a1 + an ] +
a1 a2 an a1 an
a1 a n
= (n − 1) + 1 + (n − 1)
2
+
a n a1
(n + 1)(2n − 1)
= .
2
The equality holds for
an
a1 = a2 = · · · = an−1 =
2
376 Vasile Cîrtoaje
(2) To prove the right original inequality, we only need to consider the case
a1 ≤ a2 = a3 = · · · = an .
2an
a1 = ,
n+2
hence
1 1 1 n−1 1
(a1 + a2 + · · · + an ) + + ··· + = [(n − 1)a1 + an ] +
a1 a2 an a1 an
a1 a n
= (n − 1) + 1 + (n − 1)
2
+
a n a1
3n2 (n + 1)
= .
2(n + 2)
176
abc + bcd + cda + dab ≤ 1 + abcd.
81
bcd ≤ 1,
1 1 1 1 176
abcd + + + ≤1+ abcd
a b c d 81
and apply Corollary 5 for k = 0 and m = −1:
• If
then
1 1 1 1
S4 = + + +
a b c d
is maximal for
a ≤ b = c = d.
Thus, we only need to prove the homogeneous inequality
a = 0, b=c=d =1
3
a2 b2 c 2 + b2 c 2 d 2 + c 2 d 2 a2 + d 2 a2 b2 + abcd ≤ 1.
4
b2 c 2 d 2 ≤ 1,
378 Vasile Cîrtoaje
1 1 1 1 3
2 2 2 2
a b c d + + + + abcd ≤ 1,
a2 b2 c 2 d 2 4
and apply Corollary 5 for k = 0 and m = −2:
• If
then
1 1 1 1
S4 = 2
+ 2+ 2+ 2
a b c d
is maximal for a ≤ b = c = d.
Thus, we only need to prove the homogeneous inequality
6
a+b+c+d 1
≥ a2 b2 c 2 + b2 c 2 d 2 + c 2 d 2 a2 + d 2 a2 b2 + abcd(a + b + c + d)2
3 12
for a ≤ b = c = d = 1; that is, to show that 0 < a ≤ 1 implies
a 6 1
1+ ≥ 1 + 3a2 + a(a + 3)2 .
3 12
Since a 3 a2 a3 a2
1+ =1+a+ + >1+a+ ,
3 3 27 3
it suffices to show that
2
a2 1
1+a+ ≥ 1 + 3a2 + a(a + 3)2 ,
3 12
which is equivalent to the obvious inequality
4
a2 b2 c 2 + b2 c 2 d 2 + c 2 d 2 a2 + d 2 a2 b2 + (abcd)3/2 ≤ 1.
3
Solution. The proof is similar to the one of the preceding P 5.23. We need to prove
that a 6 4
1+ ≥ 1 + 3a2 + a3/2
3 3
for 0 ≤ a ≤ 1. Since
2a3/2 ≤ a2 + a,
it suffices to show that a 6 2 11 2
1+ ≥1+ a+ a .
3 3 3
Since a 3 a2 a3 a2
1+ =1+a+ + ≥1+a+
3 3 27 3
and
2
a2 5 2 1
1+a+ = 1 + 2a + a2 + a3 + a4
3 3 3 9
5 2 2 3
≥ 1 + 2a + a + a ,
3 3
it suffices to show that
5 2 2 11 2
1 + 2a + a2 + a3 ≥ 1 + a + a ,
3 3 3 3
which is equivalent to the obvious inequality
a(1 − a)(2 − a) ≥ 0.
a2 b2 c 2 + b2 c 2 d 2 + c 2 d 2 a2 + d 2 a2 b2 + 2(abcd)3/2 ≤ 6.
Solution. The proof is similar to the one of P 5.23. We need to prove that
a+3 6
6 ≥ 1 + 3a2 + 2a3/2
4
for 0 ≤ a ≤ 1. Since
2a3/2 ≤ a2 + a,
it suffices to show that 6
a+3
6 ≥ 1 + a + 4a2 .
4
Using the substitution
1−a 1
x= , 0≤ x ≤ ,
4 4
the inequality becomes
which is equivalent to
3
a = 0, b=c=
2
(or any cyclic permutation).
Remark. In the same manner, we can prove the following statement:
• If a, b, c, d are nonnegative real numbers so that a + b + c + d = 4, then
a2 b2 + b2 c 2 + c 2 a2 + a3 b3 + b3 c 3 + c 3 a3 ≥ 6abc.
1 1 1 1 1 1
abc 2 + 2 + 2 + a b c 2 2 2
+ + ≥ 6.
a b c a3 b3 c 3
According to Corollary 5 (case k = 0 and m < 0), if
1 1 1 1 1 1
then the sums 2
+ 2 + 2 and 3 + 3 + 3 are maximal for 0 < a ≤ b = c.
a b c a b c
Thus, we only need to prove that
for
a + 2b = 3, 1 ≤ b < 3/2.
The inequality is equivalent to
a = b = 0, c=3
which is equivalent to q p
5 3 + 2 bc ≥ 3 + 4bc.
Substituting
p b+c 3
x= bc, 0 ≤ x ≤ = ,
2 2
EV Method for Nonnegative Variables 383
66
f (x) = + 50 − 24x − 16x 3 , 0 < x ≤ 3/2.
x
Since f is decreasing, we have
for
3
a + 2b = 3, 1≤ b< .
2
Write the inequality as
p p
5 3 − 2b + 10 b ≥ 3 + 24b − 12b2 .
Substituting v
p t3
x= b, 1 ≤ x < ,
2
the inequality becomes
p
5 3 − 2x 2 ≥ 3 − 10x + 24x 2 − 12x 4 ,
p
12(x 2 − 1)2 ≥ 5 3 − 2x − 3 − 2x 2 ,
30(x − 1)2
12(x 2 − 1)2 ≥ p ,
3 − 2x + 3 − 2x 2
which is true if
5
2(x + 1)2 ≥ p .
3 − 2x + 3 − 2x 2
It suffices to show that
5
2(x + 1)2 ≥ ,
3 − 2x
which is equivalent to
1 + 8x − 2x 2 − 4x 3 ≥ 0,
7 4 − 3x
x(5 − 4x) +x + ≥ 0.
4 4
384 Vasile Cîrtoaje
Since v
t3 5 4
x< < < ,
2 4 3
the conclusion follows.
The equality holds for a = b = c = 1.
(a + b + c)2 = 6 + a2 + b2 + c 2 ,
p p
We have b + c ≥ 2 bc = 2 3, hence
Æ p Æ p p p
b+c+ 13 + 2(b + c) ≥ 2 3 + 13 + 4 3 = 4 3 + 1 > 13 − 3 3.
3 − b2 p
a= . 0< b< 3.
2b
Thus, the inequality can be written as
v
t 3 − b2 p p
1+ + 2 1 + 2b ≥ 3 3.
b
Substituting
v
t 1 + 2b
v p
1 1+2 3 5
t
t= , p <t< < ,
3 3 3 4
the inequality turns into
v
t 3 + 4t 2 − 3t 4
≥ 3 − 2t.
2(3t 2 − 1)
7 − 8t − 14t 2 + 24t 3 − 9t 4 ≥ 0,
which is equivalent to
(1 − t)2 (7 + 6t − 9t 2 ) ≥ 0.
This is true since
2
15 7
7 + 6t − 9t = 8 − (3t − 1) > 8 −
2 2
−1 = > 0.
4 16
0 ≤ k ≤ 15,
then
1 1 1 k 9+k
+ + + ≥ .
(a + b)2 (b + c)2 (c + a)2 (a + b + c)2 4(ab + bc + ca)
(Vasile C., 2007)
386 Vasile Cîrtoaje
a + b + c = 1, 0 ≤ a ≤ b ≤ c.
1 1 1 9+k
+ + +k≥ .
(1 − a)2 (1 − b)2 (1 − c)2 2(1 − a2 − b2 − c 2 )
1
f (u) = , 0 ≤ u < 1.
(1 − u)2
2 24
g(x) = f 0 (x) = , g 00 (x) = .
(1 − x)3 (1 − x)5
Since g 00 (x) > 0, g is strictly convex on [0, 1). According to Corollary 1 and Note
3, if
a + b + c = 1, a2 + b2 + c 2 = const ant, 0 ≤ a ≤ b ≤ c,
then the sum
S3 = f (a) + f (b) + f (c)
is minimal for either a = 0 or 0 < a ≤ b = c.
Case 1: a = 0. For
b c
x= + , x ≥ 2,
c b
the original inequality becomes
1 1 1+k 9+k
+ 2+ ≥ ,
b 2 c (b + c)2 4bc
1+k 9+k
x+ ≥ ,
x +2 4
(x − 2)(4x + 7 − k) ≥ 0.
This is true since
4x + 7 − k ≥ 15 − k ≥ 0.
Case 2: 0 < a ≤ b = c. The original inequality becomes
2 1 k 9+k
+ 2+ ≥ ,
(a + b)2 4b (a + 2b)2 4b(2a + b)
P 5.31. If a, b, c are nonnegative real numbers, no two of which are zero, then
1 1 1 24 8
+ + + ≥ .
(a + b)2 (b + c)2 (c + a)2 (a + b + c)2 ab + bc + ca
Solution. As shown in the proof of the preceding P 5.30, it suffices to prove the
inequality for a = 0, and for 0 < a ≤ b = c.
Case 1: a = 0. For
b c
x= + , x ≥ 2,
c b
the original inequality becomes
1 1 25 8
+ + ≥ ,
b2 c 2 (b + c)2 bc
25
x+ ≥ 8,
x +2
(x − 3)2 ≥ 0.
Case 2: 0 < a ≤ b = c. Due to homogeneity, we only need to prove the homoge-
neous inequality for 0 < a ≤ b = c = 1; that is,
2 1 24 8
+ + ≥ .
(a + 1)2 4 (a + 2)2 2a + 1
which is equivalent to
1 4(1 − a)2
≥ ,
(1 + a)2 (2a + 1)(a + 2)2
a(2a2 + 9a + 12) ≥ 4a2 (a2 − 2).
This is true since
a(2a2 + 9a + 12) ≥ 0 ≥ 4a2 (a2 − 2).
388 Vasile Cîrtoaje
P 5.32. If a, b, c are nonnegative real numbers, no two of which are zero, so that
then
1 1 1 9k 3
+ + + ≥ + k.
(a + b)2 (b + c)2 (c + a)2 (a + b + c)2 4
(Vasile C., 2007)
As shown in the proof of P 5.30, it suffices to prove this inequality for a = 0, and
for 0 < a ≤ b = c.
Case 1: a = 0. Let
b c
x=+ , x ≥ 2.
c b
The homogeneous inequality becomes
ka6 + (10k + 6)a5 − (14k − 12)a4 − (10k + 18)a3 + (17k − 24)a2 + (24 − 4k)a ≥ 0,
a(a − 1)2 [ka3 + 6(2k + 1)a2 + 3(3k + 8)a + 4(6 − k)] ≥ 0.
Clearly, the last inequality is true for 0 ≤ k ≤ 6.
The equality holds for a = b = c, and also for
a = 0, b=c
P 5.33. If a, b, c are nonnegative real numbers, no two of which are zero, then
2 2 2 8 1
(a) + 2 + 2 ≥ 2 + ;
a2 +b 2 b +c 2 c +a 2 a +b +c
2 2 ab + bc + ca
2 2 2 7 6
(b) + + ≥ + ;
a2 + b2 b2 + c 2 c 2 + a2 a2 + b2 + c 2 (a + b + c)2
2 2 2 45
(c) + 2 + 2 ≥ .
a2 +b 2 b +c 2 c +a 2 4(a + b + c ) + ab + bc + ca
2 2 2
a2 + b2 + c 2 = 1, 0 ≤ a ≤ b ≤ c.
2x 24x(1 + x 2 )
g(x) = f 0 (x) = , g 00 (x) = .
(1 − x 2 )2 (1 − x 2 )4
390 Vasile Cîrtoaje
Since g 00 (x) > 0 for x ∈ (0, 1), g is strictly convex on [0, 1). According to Corollary
1 and Note 3, if
a + b + c = const ant, a2 + b2 + c 2 = 1, 0 ≤ a ≤ b ≤ c,
2 2 6 1
+ 2≥ 2 + ,
b 2 c b +c 2 bc
6
2x ≥ + 1,
x
(x − 2)(2x + 3) ≥ 0.
Case 2: 0 < a ≤ b = c. Due to homogeneity, it suffices to prove the original
inequality for b = c = 1. Thus, we need to show that
4 8 1
1+ ≥ + ,
a2 + 1 a2 + 2 2a + 1
which is equivalent to
2a 4a2
≥ 2 ,
2a + 1 (a + 1)(a2 + 2)
a(a4 − a2 − 2a + 2) ≥ 0,
a(a − 1)2 (a2 + 2a + 2) ≥ 0.
The equality holds for a = b = c, and also for a = 0, b = c (or any cyclic permu-
tation).
(b) The proof is similar to the one of the inequality in (a). For a = 0 and
b c
x= + , x ≥ 2,
c b
the original inequality becomes
2 2 5 6
+ 2≥ 2 + ,
b 2 c b +c 2 (b + c)2
5 6
2x ≥ + ,
x x +2
EV Method for Nonnegative Variables 391
(x − 2)(2x 2 + 8x + 5) ≥ 0.
For b = c = 1, the original inequality is
4 7 6
1+ ≥ 2 + ,
a2 + 1 a + 2 (a + 2)2
The equality holds for a = b = c, and also for a = 0, b = c (or any cyclic permu-
tation).
(c) The proof is also similar to the one of the inequality in (a). For a = 0 and
b c
x= + , x ≥ 2,
c b
the original inequality becomes
1 1 2 45
2 2+ 2 + 2 ≥ ,
b c b +c 2 4(b + c 2 ) + bc
2
2 45
2x + ≥ ,
x 4x + 1
(x − 2)(8x 2 + 18x − 1) ≥ 0.
For b = c = 1, the original inequality can be written as
4 45
1+ ≥ ,
a2 + 1 4a2 + 2a + 9
a(2a3 + a2 − 8a + 5) ≥ 0,
a(a − 1)2 (2a + 5) ≥ 0.
The equality holds for a = b = c, and also for a = 0, b = c (or any cyclic permu-
tation).
P 5.34. If a, b, c are nonnegative real numbers, no two of which are zero, then
1 1 1 3 4
+ + + ≥ .
a2 + b2 b2 + c 2 c 2 + a2 a2 + b2 + c 2 ab + bc + ca
Solution. As shown in the proof of the preceding P 5.33, it suffices to prove the
inequality for a = 0, and for 0 < a ≤ b = c.
Case 1: a = 0. For
b c
x= + , x ≥ 2,
c b
the original inequality becomes
1 1 4 4
+ 2+ 2 ≥ ,
b 2 c b +c 2 bc
4
x+ ≥ 4,
x
(x − 2)2 ≥ 0.
Case 2: 0 < a ≤ b = c. Due to homogeneity, it suffices to prove the original
inequality for 0 < a ≤ b = c = 1. Thus, we need to show that
1 2 3 4
+ 2 + 2 ≥ .
2 a + 1 a + 2 2a + 1
It suffices to show that
2 3 4 1
+ ≥ − ,
a + 1 a + 2 2a + 1 2
which is equivalent to
5a + 7 7 − 2a
≥ ,
a2 + 3a + 2 4a + 2
a(2a2 + 19a + 21) ≥ 0,
P 5.35. If a, b, c are nonnegative real numbers, no two of which are zero, then
3 3 3 5 4
(a) + 2 + 2 ≥ + 2 ;
a2 + ab + b 2 b + bc + c 2 c + ca + a 2 ab + bc + ca a + b2 + c 2
3 3 3 1 24
(b) + 2 + 2 ≥ + ;
a2 + ab + b 2 b + bc + c 2 c + ca + a 2 ab + bc + ca (a + b + c)2
1 1 1 21
(c) + 2 + 2 ≥ .
a2 + ab + b b + bc + c c + ca + a
2 2 2 2(a + b + c ) + 5(ab + bc + ca)
2 2 2
a + b + c = 1, 0 ≤ a ≤ b ≤ c.
Let
1 + a2 + b2 + c 2
p= .
2
Since
1 1 1
= = ,
2(b2 + bc + c ) (a + b + c) + a + b + c − 2a(a + b + c) 2(p − a)
2 2 2 2 2
a + b + c = 1, a2 + b2 + c 2 = 2p − 1 = const ant, 0 ≤ a ≤ b ≤ c,
394 Vasile Cîrtoaje
1 1 3 5 4
3 2+ 2 + 2 ≥ + 2 ,
b c b + bc + c 2 bc b + c 2
which is equivalent to
3 4
3x + ≥5+ ,
x +1 x
(x − 2)(3x 2 + 4x + 2) ≥ 0.
Case 2: 0 < a ≤ b = c. Due to homogeneity, it suffices to prove the original
inequality for b = c = 1. Thus, we need to show that
6 5 4
+1≥ + 2 ,
a2 +a+1 2a + 1 a + 2
which is equivalent to
a(a4 − a3 + 3a2 − 7a + 4) ≥ 0,
The equality holds for a = b = c, and also for a = 0, b = c (or any cyclic permuta-
tion).
(b) The proof is similar to the one of the inequality in (a). For a = 0, the
original inequality becomes
1 1 3 1 24
3 2+ 2 + 2 ≥ + ,
b c b + bc + c 2 bc (b + c)2
which is equivalent to
3 24 b c
3x + ≥1+ , x= + ,
x +1 x +2 c b
6 1 24
+1≥ + 2 ,
a2 +a+1 2a + 1 a + 2
EV Method for Nonnegative Variables 395
which is equivalent to
The equality holds for a = b = c, and also for a = 0, b = c (or any cyclic permuta-
tion).
(c) The proof is similar to the one of the inequality in (a). For a = 0, the
original inequality becomes
1 1 1 21
+ 2+ 2 ≥ ,
b 2 c b + bc + c 2 2(b + c 2 ) + 5bc
2
which is equivalent to
1 21 b c
x+ ≥ , x= + ,
x + 1 2x + 5 c b
(x − 2)(2x 2 + 11x + 8) ≥ 0.
For b = c = 1, the original inequality becomes
2 1 21
+ ≥ 2 ,
a2 + a + 1 3 2a + 10a + 9
which is equivalent to
a(a3 + 6a2 − 15a + 8) ≥ 0,
a(a − 1)2 (a + 8) ≥ 0.
The equality holds for a = b = c, and also for a = 0, b = c (or any cyclic permuta-
tion).
Solution. Denoting
Assume that
which involve
We have
x + y + z = (a + b + c)2 = const ant,
x 2 + y 2 + z 2 = (a2 + b2 + c 2 )2 + 2(ab + bc + ca)2 = const ant.
According to the EV-Theorem (Corollary 1), since f 000 (u) ≥ 0 for u ∈ (0, ∞), the
sum f (x) + f ( y) + f (z) is maximal for x = y ≤ z, that is
1 1 1 1 2
+ 2 + 2 ≥ 2 + ,
+ 2bc b + 2ca c + 2ab
a2 a +b +c
2 2 ab + bc + ca
p p p p p
a2 + 2bc + b2 + 2ca + c 2 + 2ab ≤ a2 + b2 + c 2 + 2 ab + bc + ca,
1 1 1 1 2
p +p +p ≥p +p ,
a2 + 2bc b2 + 2ca c2 + 2ab a2 + b2 + c2 ab + bc + ca
1 1 1 85
+ + ≤ .
(a + b)2 (b + c)2 (c + a)2 36(ab + bc + ca)
a = y + z, b = z + x, c = x + y,
x + y + z = 2, 0 ≤ x ≤ y ≤ z.
1 1 5(b2 + c 2 ) + 8bc
+ =
(a + b)2 (c + a)2 (2b2 + 2c 2 + 5bc)2
and
ab + bc + ca = a(b + c) + bc = (b + c)2 + bc = b2 + c 2 + 3bc,
we need to show that
5(b2 + c 2 ) + 8bc 1 85
+ ≤ .
(2b + 2c + 5bc)
2 2 2 (b + c)2 36(b + c 2 + 3bc)
2
b c
t= + , t ≥ 2,
c b
398 Vasile Cîrtoaje
5t + 8 1 85
+ ≤ ,
(2t + 5)2 t + 2 36(t + 3)
5t + 8 49t + 62
≤ .
(2t + 5)2 36(t + 2)(t + 3)
It suffices to show that
5t + 8 48t + 64
≤ ,
(2t + 5)2 36(t + 2)(t + 3)
which is equivalent to
5t + 8 12t + 16
≤ ,
(2t + 5)2 9(t + 2)(t + 3)
3t 3 + 7t 2 − 10t − 32 ≥ 0,
(t − 2)(3t 2 + 13t + 16) ≥ 0.
Case 2: 0 < x ≤ y = z. This involves b = c. Since the original inequality is
homogeneous, we may consider b = c = 1 and 0 ≤ a ≤ b + c = 2. Thus, we only
need to show that
1 2 85
+ ≤ ,
4 (a + 1) 2 36(2a + 1)
which is equivalent to
(a − 2)(9a2 − 2a + 1) ≤ 0.
1 1 1 3(a2 + b2 + c 2 )
+ + ≤ .
(a + b)2 (b + c)2 (c + a)2 4(ab + bc + ca)
1 1 1 27(a2 + b2 + c 2 )
+ + ≤ .
(a + b)2 (b + c)2 (c + a)2 4(a + b + c)2 (ab + bc + ca)
As shown in the proof of the preceding P 5.37, it suffices to prove this inequality
for a = b + c and for b = c = 1.
EV Method for Nonnegative Variables 399
Case 1: a = b + c. Since
1 1 5(b2 + c 2 ) + 8bc
+ =
(a + b)2 (c + a)2 (2b2 + 2c 2 + 5bc)2
and
27(a2 + b2 + c 2 ) 27(b2 + c 2 + bc)
= ,
4(a + b + c)2 (ab + bc + ca) 8(b + c)2 (b2 + c 2 + 3bc)
we need to show that
5(b2 + c 2 ) + 8bc 1 27(b2 + c 2 + bc)
+ ≤ .
(2b2 + 2c 2 + 5bc)2 (b + c)2 8(b + c)2 (b2 + c 2 + 3bc)
For bc = 0, the inequality is true. For bc 6= 0, substituting
b c
t= + , t ≥ 2,
c b
the inequality becomes
5t + 8 1 27(t + 1)
+ ≤ ,
(2t + 5)2 t + 2 8(t + 2)(t + 3)
9t 2 + 38t + 41 27(t + 1)
≤ .
(2t + 5)2 8(t + 3)
It suffices to show that
9t 2 + 45t + 27 27(t + 1)
≤ ,
(2t + 5)2 8(t + 3)
which is equivalent to
t 2 + 5t + 3 3(t + 1)
≤ ,
(2t + 5)2 8(t + 3)
4t 3 + t(8t − 9) + 3 ≥ 0.
2
P 5.39. Let a, b, c ≥ so that a + b + c = 3. Then,
5
1 1 1 3
+ + ≤ .
3 + 2(a + b ) 3 + 2(b + c ) 3 + 2(c + a ) 7
2 2 2 2 2 2
1 49 − 72a + 30a2
≤ ,
3 + 4a2 7(21 − 24a + 10a2 )
a(a − 1)2 (5a − 2) ≥ 0.
2 11
a=b= , c=
5 5
(or any cyclic permutation).
n2 − 1
with equality for a1 = a2 = · · · = an = 1. If k = 2 , then the equality holds
n −n−1
also for
1 n−1
a1 = · · · = an−1 = 2 , an = n − 2
n −n−1 n −n−1
(or any cyclic permutation).
2 2 2 99
+ + ≤ .
2 + a2 + b2 2 + b2 + c 2 2 + c 2 + a2 63 + a2 + b2 + c 2
Solution. The proof is similar to the one of P 5.39. Thus, we only need to prove
the inequality for 0 ≤ a = b ≤ c; that is, to show that 2a + c = 3 involves
1 4 99
+ ≤ .
1 + a2 2 + a2 + c 2 63 + 2a2 + c 2
Write this inequality as follows
1 4 33
+ 2 ≤ ,
a2 + 1 5a − 12a + 11 2(a − 2a + 12)
2
1 19
a=b= , c=
7 7
(or any cyclic permutation).
Remark. In the same manner, we can prove the following generalization:
8
• Let a, b, c be nonnegative real numbers so that a + b + c = 3. If ≤ k ≤ 3, then
5
k+2 k+2 k+2 9(3k2 + 11k + 10)
+ + ≤ ,
k + a2 + b2 k + b2 + c 2 k + c 2 + a2 9(k2 + 2k + 6) + (5k − 8)(a2 + b2 + c 2 )
3−k 2k + 15
a=b= , c=
7 7
(or any cyclic permutation).
1 1 1 18
+ + ≤ .
3+a + b
2 2 3+ b +c
2 2 3+c +a
2 2 27 + a + b2 + c 2
2
Solution. The proof is similar to the one of P 5.39. Thus, we only need to prove
the inequality for 0 ≤ a = b ≤ c. Therefore, we only need to show that 2a + c = 3
involves
1 2 18
+ ≤ .
3 + 2a 2 3+a +c
2 2 27 + 2a2 + c 2
Write this inequality as follows
1 2 3
+ 2 ≤ 2 ,
2a2 + 3 5a − 12a + 12 a − 2a + 6
a2 (a − 1)2 ≥ 0.
a = b = 0, c=3
a1 = · · · = an−1 = 0, an = n
5 5 5 27
+ + ≥ .
3+a + b
2 2 3+ b +c
2 2 3+c +a
2 2 6 + a + b2 + c 2
2
m = 3 + a2 + b2 + c 2 ,
a + b + c = 3, a2 + b2 + c 2 = const ant, 0 ≤ a ≤ b ≤ c,
5 5 5 27
+ + ≥ ,
(b + c)2 + 3b2 (b + c)2 + 3c 2 (b + c)2 + 3(b2 + c 2 ) 2(b + c)2 + 3(b2 + c 2 )
5[5(b2 + c 2 ) + 4bc] 5 27
+ ≥ .
4(b + c ) + 10bc(b + c ) + 13b c
2 2 2 2 2 2 2 4(b + c ) + 2bc
2 2 5(b + c 2 ) + 4bc
2
b c
+ = t, t ≥ 2,
c b
we may write the inequality as
5(5t + 4) 5 27
+ ≥ ,
4t 2 + 10t + 13 4t + 2 5t + 4
5(5t + 4) 83t + 34
≥ .
4t 2+ 10t + 13 2(2t + 1)(5t + 4)
Since
83t + 34 ≤ 90t + 20,
it suffices to show that
5t + 4 9t + 2
≥ ,
4t 2 + 10t + 13 (2t + 1)(5t + 4)
which is equivalent to
14t 3 + 7t 2 − 65t − 10 ≥ 0,
(t − 2)(14t 2 + 35t + 5) ≥ 0.
Case 2: 0 < a ≤ b = c. We only need to prove the homogeneous inequality for
b = c = 1; that is,
10 5 27
+ ≥ ,
(a + 2)2 + 3(a + 1) (a + 2) + 6 2(a + 2) + 3(a2 + 2)
2 2 2
10 5 27
+ 2 ≥ 2 ,
4a2 + 4a + 7 a + 4a + 10 5a + 8a + 14
a(a3 − 3a + 2) ≥ 0,
a(a − 1)2 (a + 2) ≥ 0.
EV Method for Nonnegative Variables 405
where
n2 (n − 1)2 k + n3 (n2 − n − 1) n(n − 1)2 k2 + n2 (n2 − n − 1)k + n3
p= , q= ,
(n − 1)3 k + n(n3 − 2n2 − n + 1) (n − 1)3 k + n(n3 − 2n2 − n + 1)
with equality for a1 = a2 = · · · = an = 1, and also for
n
a1 = 0, a2 = · · · = a n =
n−1
(or any cyclic permutation).
For k = 0 and k = n, we get the inequalities
X 1 n2 (n2 − n − 1)
≥ ,
a22 + · · · + an2 n2 + (n3 − 2n2 − n + 1)(a12 + a22 + · · · + an2 )
X 2n − 1 n2 (2n − 3)
≥ .
n + a22 + · · · + an2 n(n − 1) + (n − 2)(a12 + a22 + · · · + an2 )
406 Vasile Cîrtoaje
Solution. The proof is similar to the one of P 5.39. Thus, we only need to prove
the inequality for 0 ≤ a = b = c ≤ d, that is to show that 3a + d = 4 involves
1 9 296
+ ≤ .
1 + 2a2 3 + 4a2 + 2d 2 218 + 3a2 + d 2
Write this inequality as follows
1 9 148
+ ≤ ,
1 + 2a 2 35 − 48a + 22a 2 3(39 − 4a + 2a2 )
1 53
a=b=c= , d=
14 14
(or any cyclic permutation).
4 4 4 21
+ + ≥ .
2 + a2 + b2 2 + b2 + c 2 2 + c 2 + a2 4 + a2 + b2 + c 2
Solution. The proof is similar to the one of P 5.42. Thus, we only need to prove
the inequality for a = 0 and for 0 < a ≤ b = c.
Case 1: a = 0. We need to show that bc = 3 involves
1 1 1 21
+ + ≥ .
2+ b 2 2+c 2 2+ b +c
2 2 4(4 + b2 + c 2 )
Denote
x = b2 + c 2 , x ≥ 2bc = 6.
Since
1 1 4 + b2 + c 2 x +4
+ = = ,
2 + b2 2 + c 2 b2 c 2 + 2(b2 + c 2 ) + 4 2x + 13
EV Method for Nonnegative Variables 407
which is equivalent to
8 1 21
+ ≥ ,
3a2 + 4a + 5 a + 2 3a2 + 8a + 10
a2 + 4a + 7 7
≥ 2 ,
(3a + 4a + 5)(a + 2) 3a + 8a + 10
2
a(3a3 − a2 − 7a + 5) ≥ 0,
a(a − 1)2 (3a + 5) ≥ 0.
The equality holds for a = b = c = 1, and also for
p
a = 0, b=c= 3
1 1 1 45
+ 2 + 2 ≥ .
a2 +b 2 b +c 2 c +a 2 2(ab + bc + ca) + 8(a2 + b2 + c 2 )
1 1 1 1
+ + ≤ .
10 − (a + b)2 10 − (b + c)2 10 − (c + a)2 2
Solution. Let
s = a + b + c, s ≤ 3.
We need to show that
1 1 1 1
+ + ≤
10 − (s − a)2 10 − (s − b)2 10 − (s − c)2 2
−1
f (u) = , 0 ≤ u ≤ s ≤ 3.
10 − (s − u)2
We have
2(s − x)
g(x) = f 0 (x) = ,
[10 − (s − x)2 ]2
24(s − x)[10 + (s − x)2 ]
g 00 (x) = .
[10 − (s − x)2 ]4
Since g 00 (x) > 0 for x ∈ [0, s), g is strictly convex on [0, s]. According to the
Corollary 1, if
a + b + c = s, a2 + b2 + c 2 = 3, 0 ≤ a ≤ b ≤ c,
then
S3 = f (a) + f (b) + f (c)
EV Method for Nonnegative Variables 409
2 2a2 + 1
≤ ,
7a2 − 6a + 17 (5a2 + 4)(a2 + 2)
4a4 − 12a3 + 13a2 − 6a + 1 ≥ 0,
(a − 1)2 (2a − 1)2 ≥ 0.
The equality holds for a = b = c = 1, and also for
2
2a = b = c = p
3
(or any cyclic permutation).
P 5.46. If a, b, c are nonnegative real numbers, no two of which are zero, so that
a4 + b4 + c 4 = 3, then
1 1 1 3
+ 5 + 5 ≥ .
a5 +b 5 b +c 5 c +a 5 2
(Vasile C., 2010)
410 Vasile Cîrtoaje
3
f (x) + f ( y) + f (z) ≥ ,
2
where
1
f (u) = , 0 ≤ u < p4/5 .
p − u5/4
We will apply the EV-Theorem for k = 5/4. We have
5u1/4
f 0 (u) = ,
4(p − u5/4 )2
1 5x
g(x) = f 0 x k−1 = f 0 (x 4 ) = ,
4(p − x 5 )2
75x 4 (2p + 3x 5 )
g 00 (x) = .
2(p − x 5 )4
Since g 00 (x) ≥ 0, g is strictly convex. According to the EV-Theorem and Note 3, if
then
S3 = f (x) + f ( y) + f (z)
is minimal for either x = 0 or 0 < x ≤ y = z. Thus, we only need to prove the
homogeneous inequality
5/4
1 1 1 3 3
+ + ≥
a5 + b5 b5 + c 5 c 5 + a5 2 a4 + b4 + c 4
9/4
1 3 1
2A 5/2
+ 5/2 ≥ · 5/2 ,
2A 2 B
where 2/5 1/2
t 5/2 + t −5/2 t 2 + t −2
b
A= , B= , t= .
2 2 c
By power mean inequality, we have A ≥ B ≥ 1. Since
1 1 1
2A + 5/2 − 2B + 5/2 = A − B
5/2 5/2 5/2 5/2
2 − 5/2 5/2 ≥ 0,
2A 2B 2A B
54 > 23 · 33
and
32 · 23 > 36 .
Case 2: 0 < a ≤ 1 = b = c. The homogeneous inequality becomes
5/4
a5 + 5 3
≥3 4 ,
a5 + 1 a +2
5 9 ln 3
g(a) = ln(a5 + 5) − ln(a5 + 1) + ln(a4 + 2) − ,
4 4
with
There exists d ∈ (0, 1) so that g 0 (d) = 0, g 0 (a) > 0 for a ∈ [0, d) and g 0 (a) < 0 for
a ∈ (d, 1). Therefore, g is increasing on [0, d] and is decreasing on [d, 1]. Since
g(1) = 0, we only need to show that g(0) ≥ 0. Indeed,
1 54 · 25
g(0) = ln > 0.
4 39
for
a + (n − 1)b = n.
By squaring, the inequality becomes
Æ
2n(n − 1) (a2 + 1)(b2 + 1) ≥ (n − 2)a2 − (n − 2)(n − 1)2 b2 + n3 ,
which is equivalent to
Æ
(b2 + 1)[(n − 1)2 b2 − 2n(n − 1)b + n2 + 1] ≥ n − (n − 2)b.
This is true if
which is equivalent o
1
a1 = a2 = · · · = an−1 = , an = n − 1
n−1
(or any cyclic permutation).
Solution. The proof is similar to the one of the preceding P 5.47. Thus, it suf-
fices to prove the inequality for a1 = a2 = · · · = an−1 . Write the inequality in the
homogeneous form
Xq q
n(3n − 4)a12 + S 2 ≥ n(3n − 4)(a12 + a22 + · · · + an2 ) + (4n2 − 7n + 4)S 2 ,
414 Vasile Cîrtoaje
c 3
a=b= =
3 5
(or any cyclic permutation).
EV Method for Nonnegative Variables 415
We have
−x
g(x) = f 0 (x) = p ,
x2 + 4
12x
g 00 (x) = 2 .
(x + 4)5/2
Since g 00 (x) > 0 for x > 0, g(x) is strictly convex for x ≥ 0. By Corollary 1, if
a + b + c = 3, a2 + b2 + c 2 = const ant , a ≤ b ≤ c,
Denoting
p = x y, 0 ≤ p ≤ 1,
we have
Solution. The proof is similar to the one of P 5.49. Thus, it suffices to prove the
homogeneous inequality
XÆ Æ
96a2 + (a + b + c)2 ≤ 96(a2 + b2 + c 2 ) + 73(a + b + c)2
By squaring, we get
p p Æ
(b + c) 98b2 + 2c 2 + 2b2 + 98c 2 + 2 (49b2 + c 2 )(b2 + 49c 2 ) ≤
≤ 34(b2 + c 2 ) + 72bc.
Since
p p Æ Æ
98b2 + 2c 2 + 2b2 + 98c 2 ≤ 2(98b2 + 2c 2 + 2b2 + 98c 2 ) = 10 2(b2 + c 2 )
and Æ
10(b + c) 2(b2 + c 2 ) ≤ 20(b + c)2 ,
it suffices to show that
Æ
(49b2 + c 2 )(b2 + 49c 2 ) ≤ 7(b2 + c 2 ) + 36bc.
bc(b − c)2 ≥ 0.
By squaring, we get
Æ
(97a2 + 4a + 4)(a2 + 4a + 100) ≤ 17a2 + 68a + 20.
418 Vasile Cîrtoaje
The equality holds for a = b = c = 1, and also for a = 0 and b = c = 3/2 (or any
cyclic permutation).
Remark. By squaring, we deduce the inequality
Æ Æ Æ
(32a2 + 3)(32b2 + 3) + (32b2 + 3)(32c 2 + 3) + (32c 2 + 3)(32a2 + 3) ≤ 105,
(a1 a2 − 1)2 ≥ 0.
Solution. Since
We have
−6x
g(x) = f 0 (x) = ,
1 + 3x 2
108x(1 − x 2 )
g (x) =
00
.
(1 + 3x 2 )3
Since g 00 (x) > 0 for x ∈ (0, 1), g is strictly convex on [0, 1]. According to Corollary
1 and Note 2, if
then
S3 = f (a) + f (b) + f (c)
is maximal for a = b ≤ c. or for c = 1. Thus, we only need to prove the original
inequality for these cases.
Case 1: a = b ≤ c. We need to show that
(1 + 3a2 )2 (1 + 3c 2 ) ≥ (1 + a2 + 2ac)3 .
For c = 0, the inequality is an equality. For fixed c, 0 < c ≤ 1, we need to show that
h(a) ≥ 0, where
From
12a 6(a + c) 6(1 − a2 )(a − c)
h0 (a) = − = ≤ 0,
1 + 3a2 1 + a2 + 2ac (1 + 3a2 )(1 + a2 + 2ac)
it follows that h is decreasing on [0, c], hence h(a) ≥ h(c) = 0.
Case 2: c = 1. We need to show that
(1 − a)3 ≥ 0.
Let p = a + b + c. Since
a2 + b2 + c 2 = p2 − 2p, a2 b2 + b2 c 2 + c 2 a2 = p2 − 2pabc,
then the product abc is maximal for a = b, and is minimal for a = 0 or b = c. Thus,
we only need to prove that 3p ≥ 4 + 5abc for a = b, and p + 5abc ≥ 4 for a = 0
and for b = c.
EV Method for Nonnegative Variables 421
a(2 − a) 1
c= , < a ≤ 2,
2a − 1 2
hence
(a − 1)2 (5a2 + 4)
3p − 4 − 5abc = (3 − 5a2 )c + 6a − 4 = ≥ 0.
2a − 1
For a = 0, from a + b + c = ab + bc + ca we get
b
c= , b > 1,
b−1
hence
(b − 2)2
p + 5abc − 4 = b + c − 4 = ≥ 0.
b−1
For b = c, from a + b + c = ab + bc + ca we get
b(2 − b) 1
a= , < b ≤ 2,
2b − 1 2
hence
(2 − b)(5b3 − 3b + 2)
p + 5abc − 4 = a(5b2 + 1) + 2b − 4 =
2b − 1
(2 − b)[4b3 + (b − 1)2 (b + 2)]
= ≥ 0.
2b − 1
The equality holds for a = b = c = 1, and also for a = 0 and b = c = 2 (or any
cyclic permutation).
3a 1
3a + d = 4a3 d, d= , a> p
3
,
4a3 − 1 4
involves
3 1
+ ≥ 1,
3a + 1 3d + 1
3 4a3 − 1
+ 3 ≥ 1,
3a + 1 4a + 9a − 1
4a3 − 9a2 + 6a − 1 ≥ 0,
(a − 1)2 (4a − 1) ≥ 0.
The equality holds for a = b = c = d = 1.
a1 + a2 + · · · + an = na1 a2 · · · an ,
then
1 1 1
+ + ··· + ≥ 1.
1 + (n − 1)a1 1 + (n − 1)a2 1 + (n − 1)an
1 1 1
a1 + a2 + · · · + an = + + ··· + ,
a1 a2 an
then
1 1 1
+ + ··· + ≥ 1.
1 + (n − 1)a1 1 + (n − 1)a2 1 + (n − 1)an
(Vasile C., 1996)
a1 ≤ a2 ≤ · · · ≤ an ,
1
f (u) = , u > 0.
1 + (n − 1)u
EV Method for Nonnegative Variables 423
We have
−(n − 1)
f 0 (u) = ,
[1 + (n − 1)u]2
1 −(n − 1)x
g(x) = f p
0
= p ,
x [ x + n − 1]2
3(n − 1)2
g 00 (x) = p p .
2 x( x + n − 1)4
Since g 00 (x) > 0 for x > 0, g is strictly convex on [0, ∞). By Corollary 2, if
0 < a1 ≤ a2 ≤ · · · ≤ an and
1 1 1
a1 + a2 + · · · + an = const ant, + + ··· + = const ant,
a1 a2 an
then the sum
Sn = f (a1 ) + f (a2 ) + · · · + f (an )
is minimal for a2 = · · · = an . Therefore, we only need to show that
1 n−1
+ ≥1
1 + (n − 1)a 1 + (n − 1)b
for
1 n−1
a + (n − 1)b = + , 0 < a ≤ b.
a b
Write the hypothesis as
1 1
− a = (n − 1) b − ,
a b
which involves a ≤ 1 ≤ b and
1 1
−a≥ b− , ab ≤ 1.
a b
Write the desired inequality as
n−1 1
≥1− ,
1 + (n − 1)b 1 + (n − 1)a
which is equivalent to
n−1 (n − 1)a
≥ ,
1 + (n − 1)b 1 + (n − 1)a
1 − a ≥ (n − 1)a(b − 1).
For the nontrivial case b 6= 1, we have
b(1 − a2 ) (1 − a)(1 − ab)
1 − a − (n − 1)a(b − 1) = 1 − a − a(b − 1) = ≥ 0.
a(b2 − 1) b+1
If n ≥ 3, then the equality holds for a1 = a2 = · · · = an = 1.
424 Vasile Cîrtoaje
a + b + c + d + e = const ant, a4 + b4 + c 4 + d 4 + e4 = 5, 0 ≤ a ≤ b ≤ c ≤ d ≤ e,
then p
(n + n − 1)(a12 + a22 + · · · + an2 − n) ≥ (a1 + a2 + · · · + an )2 − n2 ,
with equality for a1 = a2 = · · · = an = 1, and also for
an 1
a1 = · · · = an−1 = p =p
4
n−1 n−1
(or any cyclic permutation).
EV Method for Nonnegative Variables 425
then
Sn = a14 + a24 + · · · + an4
is maximal for a1 = · · · = an−1 ≤ an . Thus, we only need to prove the homogeneous
inequality
1
a1 = · · · = an−1 = , an = n − 1
n−1
(or any cyclic permutation).
n2 − n + 1 2 1
(a13 + a23 + · · · + an3 )2 ≥ 3
(a1 + a2 + · · · + an ) + 1 −
2 2 3
(a16 + a26 + · · · + an6 )
n n
an 1
a1 = · · · = an−1 = =p
n−1 n−1
(or any cyclic permutation).
1 1 1 50
4 + + + ≥ 27.
a b c a+b+c
then
1 1 1
+ + S3 =
a b c
is minimal for 0 < a = b ≤ c. Thus, we only need to prove that
2 1 50
4 + + ≥ 27
a c 2a + c
EV Method for Nonnegative Variables 427
for
a2 c = 1, a ≤ 1.
The inequality is equivalent to
1 1 1 (2n + n − 1)2
2 n
+ + ··· + + ≥ 2n(2n + 1),
a1 a2 an a1 + a2 + · · · + a n
We have
Since
(n − 1)2 a n − 2n ≤ (n − 1)2 − 2n < 0,
428 Vasile Cîrtoaje
1 1
it follows that f (a) < 0 for a ∈ 0,
0
, and f (a) > 0 for a ∈
0
, 1 . Therefore,
2 2
1 1
f is decreasing on 0, and increasing on , 1 , hence
2 2
1
f (a) ≥ f = 0.
2
We have
f 0 (a) = k(a k−1 − a−k−1 ) − (2k − 1)(1 − a−2 ),
f 00 (a) = k[(k − 1)a k−2 + (k + 1)a−k−2 ] − 2(2k − 1)a−3 .
By the weighted AM-GM inequality, we get
(k−1)(k−2)+(k+1)(−k−2)
(k − 1)a k−2 + (k + 1)a−k−2 ≥ 2ka 2k = 2ka−3 ,
hence
f 00 (a) ≥ 2k2 a−3 − 2(2k − 1)a−3 = 2(k − 1)2 a−3 > 0,
f 0 is strictly increasing. Since f 0 (1) = 0, it follows that f 0 (a) < 0 for a < 1 and
f 0 (a) > 0 for a > 1, f is decreasing on (0, 1] and increasing on [1, ∞), hence
f (a) ≥ f (1) = 0.
Consider further that n ≥ 3. Replacing a1 , a2 , . . . , an by 1/a1 , 1/a2 , . . . , 1/an , we
need to show that a1 a2 · · · an = 1 involves
1 1 1
+ + ··· + + (2k − n)n ≥ (2k − n + 1)(a1 + a2 + · · · + an ).
a1k a2k ank
a1 + a2 + · · · + an = const ant, a1 a2 · · · an = 1,
then
1 1 1
Sn = + + ··· +
a1k a2k ank
is minimal for 0 < a1 = · · · = an−1 ≤ an . Thus, we only need to prove the original
inequality for a1 = · · · = an−1 ≥ 1; that is, to show that t ≥ 1 involves f (t) ≥ 0,
where
1 n−1
f (t) = (n − 1)t + k(n−1) + (2k − n)n − (2k − n + 1)
k
+t n−1
.
t t
430 Vasile Cîrtoaje
We have
(n − 1)g(t)
f 0 (t) = , g(t) = k(t kn − 1) − (2k − n + 1)t kn−k−1 (t n − 1),
t kn−k+1
g 0 (t) = t kn−k−2 h(t), h(t) = k2 nt k+1 − (2k − n + 1)[(k + 1)(n − 1)t n − kn + k + 1],
h0 (t) = (k + 1)nt n−1 [k2 t k−n+1 − (2k − n + 1)(n − 1)].
If k = n − 1, then h(t) = n(n − 1)(n − 2) > 0. If k > n − 1, then
nk
a2k + · · · + ank ≥
(n − 1)k−1
EV Method for Nonnegative Variables 431
Therefore, the function h(x) is strictly decreasing for x ∈ [0, 1]. Since f (0) =
f (1) = 0, there exists x 1 ∈ (0, 1) so that f (x) is increasing on [0, x 1 ] and decreasing
on [x 1 , 1]. As a consequence, f (x) ≥ 0 for x ∈ [0, 1].
The equality holds for a1 = a2 = · · · = an = 1, and also for
n
a1 = 0, a2 = · · · = a n =
n−1
(or any cyclic permutation).
Remark. For the particular case k = n, the inequality has been posted in 2004 on
Art of Problem Solving website by Gabriel Dospinescu and Calin Popa.
1 1 1
P 5.63. If a, b, c are positive real numbers so that + + = 3, then
a b c
4(a2 + b2 + c 2 ) + 9 ≥ 21abc.
then
1 1 1
S3 =2
+ 2+ 2
a b c
is minimal for 0 < a = b ≤ c. Thus, we only need to prove that
2 1 21
4 2 + 2 +9≥ 2
a c a c
for 2a + b = 3. The inequality is equivalent to
1 1 1
P 5.64. If a1 , a2 , . . . , an are positive real numbers so that + + ··· + = n,
a1 a2 an
then
a1 + a2 + · · · + an − n ≤ en−1 (a1 a2 · · · an − 1),
where n−1
1
en−1 = 1 + .
n−1
1 1 1
a1 a2 · · · an + + ··· + − n + en−1 ≤ en−1 .
a1 a2 an
a1 + a2 + · · · + an = n, a1 a2 · · · an = const ant,
then
1 1 1
Sn = + + ··· +
a1 a2 an
is maximal for 0 < a1 ≤ a2 = · · · = an . Using the notation a1 = x and a2 = y, we
only need to show that f (x) ≤ 0 for
x + (n − 1) y = n, 0 < x ≤ 1,
where
1 n−1
f (x) = x y n−1
+ − n + en−1 − en−1
x y
= y n−1 + (n − 1)x y n−2 − (n − en−1 )x y n−1 − en−1 .
Since
−1
y0 = ,
n−1
we get
f 0 (x)
= ( y − x)h(x),
y n−3
where
n− x
h(x) = n − 2 − (n − en−1 ) y = n − 2 − (n − en−1 )
n−1
is a linear increasing function. Since
2
n
h(0) = en−1 − 3 + <0
n−1 n
434 Vasile Cîrtoaje
and
h(1) = en−1 − 2 > 0,
there exists x 1 ∈ (0, 1) so that h(x 1 ) = 0, h(x) < 0 for x ∈ [0, x 1 ), and h(x) > 0 for
x ∈ (x 1 , 1]. Consequently, f is strictly decreasing on [0, x 1 ] and strictly increasing
on [x 1 , 1]. From
f (0) = f (1) = 0,
it follows that f (x) ≤ 0 for x ∈ [0, 1].
The equality holds for a1 = a2 = · · · = an = 1. If n = 2, then the equality holds
for a1 + a2 = 2a1 a2 .
then the sum a1n−1 +a2n−1 +· · ·+ann−1 is maximal and the product a1 a2 · · · an is minimal
for either a1 = 0 or 0 < a1 ≤ a2 = · · · = an . Consequently, we only need to consider
these cases.
Case 1: a1 = 0. The inequality reduces to
(n − 2)a1n + a1 ≥ (n − 1)a1n−1 .
a1 = 0, a2 = · · · = a n
then the sum a1n + a2n + · · · + ann is maximal and the product a1 a2 · · · an is minimal for
either a1 = 0 or 0 < a1 ≤ a2 = · · · = an . Consequently, we only need to consider
these cases.
Case 1: a1 = 0. The inequality reduces to
a1 = 0, a2 = · · · = a n
(1 − a1 )2 (1 − a2 )2 ≥ 0.
1 2 1
x n−2
+ 2x n−3
+ · · · + (n − 1) y + n−2 + n−3 + · · · + (n − 1) ≥ n(n − 1),
x x y
which is equivalent to
1 1 1
x n−2
y + n−2 − 2 + 2 x n−3
y + n−3 − 2 + · · · + (n − 1) y + − 2 ≥ 0,
x y x y y
1 1
−Ç < 1.
a1 + a2 + · · · + an − n
p
1 1 1
a1 + a2 + ··· + an −n
Solution. Let
1 1 1
A = a1 + a2 + · · · + an − n, B= + + ··· + − n.
a1 a2 an
By the AM-GM inequality, it follows that A > 0 and B > 0. According to the pre-
ceding P 5.68, the following inequality holds
1 1 1
(a1 + · · · + an+1 − n − 1) + ··· + − n − 1 + a1 · · · an+1 + ≥ 2,
a1 an+1 a1 · · · an+1
438 Vasile Cîrtoaje
which is equivalent to
1 1
(A − 1 + an+1 ) B − 1 + + an+1 + ≥ 2,
an+1 an+1
A
+ Ban+1 + AB − A − B ≥ 0.
an+1
Choosing v
tA
an+1 = ,
B
we get p
2 AB + AB − A − B ≥ 0,
p p 2
AB ≥ A− B ,
1 1
1≥ p −p .
A B
n−1 n (n − 2)a1 a2 · · · an
2
1 1 1
a1 +a2 +· · ·+an +
n−1 n−1
≥ (n−1)a1 a2 · · · an + + ··· +
a1 + a2 + · · · + an a1 a2 an
for a2 = · · · = an = 1; that is, to show that f (x) ≥ 0 for x ∈ [0, 1], where
n2 (n − 2)
f (x) = x n−2 + − (n − 1)2 ,
x +n−1
f 0 (x) n2
=x n−3
− .
n−2 (x + n − 1)2
Since f 0 is increasing, we have f 0 (x) ≤ f 0 (1) = 0 for x ∈ [0, 1], f is decreasing on
[0, 1], hence f (x) ≥ f (1) = 0.
The equality holds for a1 = a2 = · · · = an = 1. If n = 2, then the equality holds
for a1 a2 = 1.
EV Method for Nonnegative Variables 439
abc − 1 2
(a + b + c − 3)2 ≥ (a + b2 + c 2 − 3).
abc + 1
b2 + c 2 + bc + 3 ≥ 3(b + c),
which is equivalent to
(b − c)2 + 3(b + c − 2)2 ≥ 0.
then
S3 = a2 + b2 + c 2
is minimal and maximal when two of a, b, c are equal. Thus, we only need to prove
the desired inequality for a = b; that is,
a2 c − 1
(2a + c − 3)2 ≥ (2a2 + c 2 − 3),
a2 c + 1
which is equivalent to
p1
(a1 a2 · · · an ) n−1 (a12 + a22 + · · · + an2 ) ≤ n.
P 5.74. If a1 , a2 , . . . , an are positive real numbers so that a13 + a23 + · · · + an3 = n, then
a1 + a2 + · · · + an ≥ n n+1 a1 a2 · · · an .
p
Solution. For n = 2, we need to show that a13 + a23 = 2 involves (a1 + a2 )3 ≥ 8a1 a2 .
Let
x = a1 + a2 .
From
2 = a13 + a23 = x 3 − 3a1 a2 x,
we get
x3 − 2
a1 a2 = .
3x
Thus,
f (x) ≥ f (1) = 0.
ak + bk + c k a + b r + c r k/r
r
≥ .
3 3
Thus, it suffices to show that
a r + b r + c r ≥ 3.
Since
2(ab + bc + ca) = (a + b + c)2 − (a2 + b2 + c 2 ),
according to Corollary 5 (case k = 2, m = r), if a ≤ b ≤ c and
then
S3 = a r + b r + c r
is minimal for either a = 0 or 0 < a ≤ b = c.
Case 1: a = 0. We need to show that bc = 3 implies b r + c r ≥ 3. Indeed, by the
AM-GM inequality, we have
Æ
b r + c r ≥ 2 (bc) r = 2 · 3 r/2 = 3.
ab + bc + ca r/2
a +b +c ≥3
r r r
3
444 Vasile Cîrtoaje
2a + 1
r/2
a +2≥3
r
,
3
which is equivalent to f (a) ≥ 0, where
a r + 2 r 2a + 1
f (a) = ln − ln .
3 2 3
The derivative
r a r−1 r r g(a)
f 0 (a) = − = 1−r r ,
a + 2 2a + 1 a (a + 2)(2a + 1)
r
where
g(a) = a − 2a1−r + 1.
From
2(1 − r)
g 0 (a) = 1 − ,
ar
it follows that g 0 (a) < 0 for a ∈ (0, a1 ), and g 0 (a) > 0 for a ∈ (a1 , 1], where
a1 = (2 − 2r)1/r ≈ 0.416.
Then, g is strictly decreasing on [0, a1 ] and strictly increasing on [a1 , 1]. Since
g(0) = 1 and g(1) = 0, there exists a2 ∈ (0, 1) so that g(a2 ) = 0, g(a) > 0 for
a ∈ [0, a2 ), and g(a) < 0 for a ∈ (a2 , 1]. Consequently, f is increasing on [0, a2 ]
and decreasing on [a2 , 1]. Since f (0) = f (1) = 0, we have f (a) ≥ 0 for 0 < a ≤ 1.
ln 4
The equality holds for a = b = c = 1. If k = 2 − , then the equality holds also
ln 3
for p
a = 0, b=c= 3
(or any cyclic permutation).
Remark. For k = 3/4, we get the following nice results (see P 3.33 in Volume 1):
• Let a, b, c be positive real numbers.
(a) If a4 b4 + b4 c 4 + c 4 a4 = 3, then
a3 + b3 + c 3 ≥ 3.
(b) If a3 + b3 + c 3 = 3, then
a4 b4 + b4 c 4 + c 4 a4 ≤ 3.
EV Method for Nonnegative Variables 445
ln 9 − ln 8
k≥ ≈ 0.29,
ln 3 − ln 2
then
a k + b k + c k ≥ ab + bc + ca.
a+b+c
k
1
a +b +c ≥3
k k k
= 3 = (a + b + c)2 ≥ ab + bc + ca.
3 3
Let
ln 9 − ln 8
r= .
ln 3 − ln 2
Assume further that
r ≤ k < 1,
and write the inequality as
2(a k + b k + c k ) + a2 + b2 + c 2 ≥ 9.
By Corollary 5, if a ≤ b ≤ c and
a + b + c = 3, a2 + b2 + c 2 = const ant,
1 n−1
+ ≤ 1,
n + 1 − a2 a3 · · · an n + 1
which is equivalent to
n+1
a2 a3 · · · an ≤ .
2
Since a + a + · · · + a n−1
2 3 n
a2 a3 · · · an ≤ = en−1 ,
n−1
it suffices to show that
n+1
en−1 ≤ .
2
For n = 4, we have
n+1 7
− en−1 = > 0.
2 54
For n ≥ 5, we get
n+1
≥ 3 > en−1 .
2
Case 2: 0 < a1 ≤ a2 ≤ · · · ≤ an . Denote
a1 a2 · · · an = (n + 1)r, r > 0.
(n + 1)r
a1 ≥ a, a= > r.
en−1
1 1 1 1
+ + ··· + ≤ ,
a1 − r a2 − r an − r r
1
f (a1 ) + f (a2 ) + · · · + f (an ) + ≥ 0,
r
where
−1
f (u) =
, u ≥ a.
u−r
We will apply Corollary 3 to the function f . We have
1
f 0 (u) = ,
(u − r)2
1 x2 4r x + 2
g(x) = f 0
= , g 00 (x) = > 0.
x (1 − r x)2 (1 − r x)4
According to Corollary 3, if a ≤ a1 ≤ a2 ≤ · · · ≤ an and
a + b + c ≥ 2, ab + bc + ca ≥ 1,
then p p p
3
a+ b+
3 3
c ≥ 2.
If c < 1, then
p p
3 p
a+ b+ c ≥ a + b + c ≥ 2.
3 3
1 1 p
f (a) = 4 ln 3a + 3 − ln 3a + 6 − ln 36 3,
2
0 < a ≤ 1.
a a
Since
12(a4 − 1) 6(a8 − 1) 6(a4 − 1)2 (3a4 − 1)
f (a) =
0
− = ,
a(3a4 + 1) a(3a8 + 1) a(3a4 + 1)(3a8 + 1)
p
4 p 4
f is decreasing on [0, 1/ 3] and increasing on [1/ 3, 1]; therefore,
1
f (a) ≥ f p 4
= 0.
3
The equality holds for
1 p
4
a=b=c= p
4
, d= 27
3
(or any cyclic permutation).
16 Æ
(a1 + a2 + · · · + an )4 ≥ (n − 1)3n−2 (a12 + a22 + · · · + an2 ),
n
n
with equality for
1 Æ
a1 = a2 = · · · = an−1 = p an = (n − 1)n−1
n
n
,
n−1
(or any cyclic permutation).
where
1p
f (u) = − 33u2 + 16, u ≥ 0.
33
We have
−x
g(x) = f 0 (x) = p ,
33x 2 + 16
33 · 48x
g 00 (x) = .
(33x 2 + 16)5/2
Since g 00 (x) > 0 for x > 0, g is strictly convex on [0, ∞). According to Corollary
1, if 0 ≤ a ≤ b ≤ c and
We see that p
9(b + c) − 4 ≥ 18 bc − 4 = 14 > 0.
452 Vasile Cîrtoaje
where
t = b + c ≥ 2.
Since
24t 2 − 36t + 25 ≥ 6t 2 + 25,
it suffices to show that
528t 2 + 289 ≤ (6t 2 + 25)2 ,
which is equivalent to
(t 2 − 4)(3t 2 − 7) ≥ 0.
72a(2a3 − a2 − 4a + 3) ≥ 0,
72a(a − 1)2 (2a + 3) ≥ 0.
1
The equality holds for a = b = c = p , and also for
3
a = 0, b=c=1
3
a2 b2 + b2 c 2 + c 2 a2 ≤ p
3
.
abc
Since
Therefore, we only need to prove the homogeneous inequality for 0 < a ≤ 1 and
b = c = 1. Taking logarithms, we have to show that f (a) ≥ 0, where
a+2 2a2 + 1
f (a) = 15 ln − ln a − 3 ln .
3 3
1 1 1 1
is negative for a ∈ 0, ∪ , 1 and positive for a ∈ , , f is decreasing
4 2 4 2
1 1 1 1
on 0, ∪ , 1 and increasing on , . Therefore, it suffices to show that
4 2 4 2
1
f ≥ 0 and f (1) ≥ 0. Indeed, we have f (1) = 0 and
4
1 312
f = ln 19 > 0.
4 2
then
a16 + a26 + · · · + an6 ≤ n.
b+c 2Æ
b +c +2
3 3
3(b2 + c 2 ) ≥ (b + c)(b2 + c 2 ).
3
1, if 0 ≤ a ≤ b ≤ c and
a + b + c = 3, ab + bc + ca = const ant,
then the sum S3 = f (a) + f (b) + f (c) is minimal for either a = 0 or 0 < a ≤ b = c.
Therefore, we only need to prove the homogeneous inequality
v
p p p t 16 2(ab + bc + ca)
a+b+ b+c+ c+a≤ (a + b + c) +
3 a+b+c
for a = 0 and b = c = 1.
v
p p t 16
2
x +2+ x≤ . x+
3 x
By squaring twice, the inequality becomes as follows:
Æ 5 1
x(x + 2) ≤ x −1+ ,
3 x
16x 4 − 48x 3 + 39x 2 − 18x + 9 ≥ 0,
(x − 2)[16x 2 (x − 1) + 7x − 4] + 1 ≥ 0.
where p
f (x) = − s − x. 0 ≤ x < s.
From
1 3
g(x) = f 0 (x) = (s − x)−1/2 , g 00 (x) = (s − x)−5/2 > 0,
2 8
it follows that g is strictly convex. Thus, by Corollary 1 and Note 2, the sum f (a) +
f (b) + f (c) is minimal for either a ≤ b = c or a = 0.
Case 1: a ≤ b = c. We need to show that 2ac + c 2 = 4 yields
p p p
2 a + c + 2c ≤ 3 + 5,
458 Vasile Cîrtoaje
that is v
t 2(c 2 + 1) p p
+ 2c ≤ 3 + 5.
c
From 2ac + c = 4,it follows that
2
2
p ≤ c ≤ 2.
3
p
Since 2c ≤ 2, it is enough to show that
v
t 2(c 2 + 1) p
≤ 1 + 5,
c
that is p
c 2 − (3 + 5)c + 4 ≤ 0.
Indeed, p
c 2 − (3 + 5)c + 4 ≤ c 2 − 5c + 4 = (c − 1)(c − 4) < 0.
S = ab + bc + ca
a (a
+ b)
c bc
a c
=q p +q p −p p
(a+b)(a+c) a+ c+a
a + b+c
S
a + a+b
p
a(a + b)
c a
≥ p p −p p ≥ 0.
a a+c+ a a+ c+a
Also, the right inequality is true for S = 4 and a, b, c ∈ [0, 4] since a > 1 and
4 p
f a, , 0 − 3 − 5 =
a
v
t 4 p 2 p
= a+− 5+ p + a−3
a a
(a − 1) 1 − 4a
p 2
= q p + ( a − 1) 1 − pa ≤ 0.
a + 4a + 5
1 3 1
5 2a + 2 ≥ 27 2 + 2a + 4 ,
2
a a
12a5 + 4a4 − 4a3 − 12a2 + 10a + 5 > 2a5 + 4a4 − 4a3 − 12a2 + 10a
= 2a(a − 1)2 (a2 + 4a + 5) ≥ 0.
a3 + b3 + c 3 = const ant, a3 b3 c 3 = 1,
1 2 1
2a + 6 ≥ 3 2a + 8 .
3 4
a a
f (t) = t 18 − 3t 14 + 4t 9 − 6t 2 + 4.
We have
f 0 (t) = 6t g(t), g(t) = 3t 16 − 7t 12 + 6t 7 − 2,
g 0 (t) = 6t 6 h(t), h(t) = 8t 9 − 14t 5 + 7,
h0 (t) = 2t 4 (36t 2 − 35).
Since h0 (t) > 0 for t ≥ 1, h is increasing, h(t) ≥ h(1) = 1 for t ≥ 1, g is increasing,
g(t) ≥ g(1) = 0 for t ≥ 1, f is increasing, hence f (t) ≥ f (1) = 0 for t ≥ 1.
The equality holds for a = b = c = 1.
a + b + c + d = 4, a2 + b2 + c 2 + d 2 = const ant,
then
S4 = a3 + b3 + c 3 + d 3
is minimal for either 0 < a ≤ b = c = d or a = 0.
Case 1: 0 < a ≤ b = c = d. We need to show that a + 3d = 4 involves
(1 − d)2 (1 + d)a ≥ 0.
Case 2: a = 0. Let
s = b2 + c 2 + d 2 .
We need to show that b + c + d = 4 involves
s2
(s − 4)(s + 18) ≤ 10 −4 ,
4
which is equivalent to the obvious inequality
(s − 4)(3s − 16) ≥ 0.
a + b + c + d = 4,
then
(a4 + b4 + c 4 + d 4 )2 ≥ (a2 + b2 + c 2 + d 2 )(a5 + b5 + c 5 + d 5 ).
(Vasile C., 2020)
n(a14 + a24 + · · · + an4 )2 ≥ (a1 + a2 + · · · + an )(a12 + a22 + · · · + an2 )(a15 + a25 + · · · + an5 ).
1/4 1/4
Replacing a1 , a2 , . . . , an with x 1 , x 2 , . . . , x n1/4 , the inequality becomes
n(x 1 + x 2 + · · · + x n )2 ≥
1/4 1/4 1/2 1/2 5/4 5/4
≥ x 1 + x 2 + · · · + x n1/4 x 1 + x 2 + · · · + x n1/2 x 1 + x 2 + · · · + x n5/4 .
By Corollary 5 (case k = 5/4), if
5/4 5/4
x 1 + x 2 + · · · + x n = const ant, x 1 + x 2 + · · · + x n5/4 = const ant,
0 ≤ x 1 = x 2 = · · · = x n−1 ≤ x n .
We have
f (a)
= a8 − a7 − a6 − (n − 1)a5 + 2na4 − a3 − (n − 1)a2 − (n − 1)a + n − 1
n−1
= a3 A − (n − 1)B,
where
A = a5 − a4 − a3 + 2a − 1, B = a5 − 2a4 + a2 + a − 1.
Since
A = (a − 1)2 (a3 + a2 − 1), B = (a − 1)2 (a3 − a − 1),
EV Method for Nonnegative Variables 463
we have
f (a) = (n − 1)(a − 1)2 g(a),
where
g(a) = a6 + a5 − na3 + (n − 1)a + n − 1.
The inequality is true if g(a) ≥ 0. For n = 4, we have
g(a) = a6 + a5 − 4a3 + 3a + 3 > 2a5 − 4a3 + 2a = 2a(a2 − 1)2 ≥ 0.
The equality occurs for a = b = c = d = 1.
Remark 1. Since g(a) ≥ 0 for n ≤ 16, the homogeneous inequality is true for all
n ≤ 16.
Remark 2. Since
(a1 + a2 + · · · + an )(a15 + a25 + · · · + an5 ) ≤ |(a1 + a2 + · · · + an )(a15 + a25 + · · · + an5 )|
≤ (|a1 | + |a2 | + · · · + |an |)(|a1 |5 + |a2 |5 + · · · + |an |5 ),
the homogeneous inequality is true for n ≤ 16 and real a1 , a2 , . . . , an .
which is equivalent to
then v v p
t3 s a 1+ 7
t b s
c
3 ≤ + + ≤ p .
5 b+c c+a a+b 2
(Vasile C., 2006)
we get
a2 + b2 + c 2 = 51.
Also, from 2(b2 + c 2 ) ≥ (b + c)2 and
b + c = 9 − a, b2 + c 2 = 51 − a2 ,
EV Method for Nonnegative Variables 465
a + b + c = 9, a2 + b2 + c 2 = 51,
then the sum S3 = f (a) + f (b) + f (c) is maximal for a = b ≤ c, and is minimal for
either a = 0 or 0 < a ≤ b = c.
(a) To prove the right inequality, it suffices to consider the case a = b ≤ c.
From
a + b + c = 9, a2 + b2 + c 2 = 51,
we get a = b = 1 and c = 7, therefore
v p
1+ 7
s s
a t b c
+ + = p .
b+c c+a a+b 2
The original right inequality is an equality for a = b = c/7 (or any cyclic permuta-
tion).
(b) To prove the left inequality, it suffices to consider the cases a = 0 and
0 < a ≤ b = c. For a = 0, from
a + b + c = 9, a2 + b2 + c 2 = 51,
we get
b c 17
+ = ,
c b 5
therefore
v v v v
s
a t b s
c tb sc tb c t3
+ + = + = + +2=3 .
b+c c+a a+b c b c b 5
The case 0 < a ≤ b = c is not possible, because from
a + b + c = 9, a2 + b2 + c 2 = 51,
466 Vasile Cîrtoaje
b c 17
a = 0, + =
c b 5
(or any cyclic permutation).
then
19 a b c 141
≤ + + ≤ .
12 b+c c+a a+b 88
(Vasile C., 2006)
Solution. The proof is similar to the one of the preceding P 5.91. Assume that
a + b + c = 15, which involves a2 + b2 + c 2 = 81 and a ∈ [3, 7], then write the
inequality in the form
19 141
≤ f (a) + f (b) + f (c) ≤ ,
12 88
where
u
f (u) = , 3 ≤ u ≤ 7.
15 − u
We have
1 90
g(x) = f 0 (x) = (15 − x)2 , g 00 (x) = .
5 (15 − x)4
Since g is strictly convex on [3, 7], according to Corollary 1, if 0 ≤ a ≤ b ≤ c and
a + b + c = 15, a2 + b2 + c 2 = 81,
then the sum S3 = f (a) + f (b) + f (c) is maximal for a = b ≤ c, and is minimal for
either a = 0 or 0 < a ≤ b = c.
(a) To prove the right inequality, it suffices to consider the case a = b ≤ c,
which involves
a = b = 4, c = 7,
and
a b c 141
+ + = .
b+c c+a a+b 88
The original right inequality is an equality for a = b = 4c/7 (or any cyclic permu-
tation).
EV Method for Nonnegative Variables 467
(b) To prove the left inequality, it suffices to consider the cases a = 0 and
0 < a ≤ b = c. The first case is not possible, while the second case involves
a = 3, b = c = 6,
and
a b c 19
+ + = .
b + c c + a a + b 12
The original left inequality is an equality for 2a = b = c (or any cyclic permutation).
a + b + c = 3, ab + bc + ca = const ant,
(a − 1)2 ≥ 0.
1
The equality occurs for a = b = c = 1, and also for a = b = and c = 2 (or any
2
cyclic permutation).
EV Method for Nonnegative Variables 469
x 3 + y 3 + z 3 = a3 + b3 + c 3 .
Then,
(a + b + c)(x + y + z) p
3
≥ 3.
ab + bc + ca + x y + yz + z x
(Vasile Cîrtoaje, 2019)
1 1
p
3
(a + b + c)(x + y + z) ≥ ab + bc + ca + (x + y + z)2 .
3 3
c 2 + 2c
g(u) = u2 − (2c + 1)u + p3
,
3
with p
3
u= 2c 3 + 1, c ∈ [0, 1].
Consider two cases: c ∈ [0, 4/5] and c ∈ [4/5, 1].
p3
Case 1: c ∈ [0, 4/5]. Since 3 > 4/3, we have
c(2c + 6 − 5c 2 ) ≥ 0.
p
3
Case 2: c ∈ [4/5, 1]. Since 3 > 7/5, we have g(u) < h(u), where
5(c 2 + 2c)
h(u) = u2 − (2c + 1)u + .
7
It suffices to prove that h(u) ≤ 0. From
h0 (u) = 2u − 2c − 1
and
64 24 −9
(2u)3 − (2c + 1)3 = 7 + 8c 3 − 12c 2 − 6c ≤ 7 − 4c 2 − 6c ≤ 7 − − = < 0,
25 5 25
it follows that h0 (u) < 0, hence h(u) is a decreasing function. Since
c3
u>1+ ,
3
EV Method for Nonnegative Variables 471
it follows that
c3 5c c 2 c 5 4 2c 3
h(u) < h 1 + =c + + − − .
3 7 3 9 7 3
Since
5c c 2 c 5 5c c c 3 22c c 3
+ + ≤ + + = + ,
7 3 9 7 3 9 21 9
it suffices to show that
22c c 3 4 2c 3
+ − − ≤ 0,
21 9 7 3
that is
22c 4 5c 3
− − ≤ 0.
21 7 9
Indeed, we have
v
4 5c 3 2 2 5c 3 t3 20c 3 22c
+ = + + ≥3 > .
7 9 7 7 9 49 · 9 21
1 1 1 1
a+b+c+d = + + + ,
a b c d
then
ab + ac + ad + bc + bd + cd + 3abcd ≥ 9.
(a + b + c + d)2 + 6abcd ≥ 18 + a2 + b2 + c 2 + d 2
1 1 1 1
a + b + c + d = const ant , + + + = const ant, a ≤ b ≤ c ≤ d,
a b c d
then the product abcd is minimal and the sum a2 + b2 + c 2 + d 2 is maximal for
a = b = c ≤ d.
472 Vasile Cîrtoaje
a1 + a2 + · · · + an = n,
then v
1 X
p u
a1 + a2 + · · · + an ≤ 2n − 1 + 2 1 −
p p t ai a j .
n i< j
then
(n − 1)(n − 2) Xp
(a1 + a2 + · · · + an ) + ai a j ≥ n(n − 1).
2 i< j
Since X
2 ai a j = (a1 + a2 + · · · + an )2 − a12 − a22 − · · · − an2
i< j
and Xp p
ai a j = ( a1 + a2 + · · · + an )2 − a1 − a2 − · · · − an ,
p p
2
i< j
(n − 1)(n − 2) 4
x 2 + (n − 1) y 2 = (n − 1)x 2 y 2 + y
2
implies
(n − 2) 2 (n − 2) 2
[x + (n − 1) y 2 ] + x y + y ≥ n,
2 2
which can be written in the homogeneous form
involves
(n − 1)(n − 2) X p
(a2 + a3 + · · · + an ) + ai a j ≥ n(n − 1). (2)
2 2≤i< j
From
(a2 + a3 + · · · + an )2 ≤ (n − 1)(a22 + a23 + · · · + an2 )
X
= (n − 1)(a2 + a3 + · · · + an ) − 2(n − 1)
2
ai a j ,
2≤i< j
we get
X
(n − 2)(a2 + a3 + · · · + an )2 ≥ 2(n − 1) ai a j = 2(n − 1)(a2 + a3 + · · · + an ),
2≤i< j
hence
2(n − 1)
a2 + a3 + · · · + an ≥ . (3)
n−2
On the other hand, by the induction hypothesis, (1) involves
(n − 2)(n − 3) X p
(a2 + a3 + · · · + an ) + ai a j ≥ (n − 1)(n − 2).
2 2≤i< j
(n − 1)(n − 2) (n − 2)(n − 3)
(a2 + a3 + · · · + an ) + (n − 1)(n − 2) − (a2 + a3 + · · · + an )
2 2
≥ n(n − 1),
which is equivalent to (3).
2
The inequality is an equality for a1 = a2 = · · · = an = , and also for a1 = 0
n−1
2
and a2 = a3 = · · · = an = (or any cyclic permutation).
n−2
476 Vasile Cîrtoaje
P 5.99. Let
Then,
1 1 1
F (a1 , a2 , . . . , an ) ≥ F , ,..., .
a1 a2 an
(Vasile C., 2020)
1 2
1 1 1 1 1
n(a1 +a2 +· · ·+an )−(a1 +a2 +· · ·+an ) ≥ n 2 + 2 + · · · + 2 −
2 2 2 2
+ + ··· + .
a1 a2 an a1 a2 an
x 2 [ y 2 + (n − 2)z 2 ] ≥ n − 1,
then
1 1 n−2 1 1 n−2 2
n[x + y +(n−2)z ]−[x+ y+(n−2)z] ≥ n
2 2 2
+ + 2 − 2
+ + ,
x2 y2 z x y z
which is equivalent to
1 1 1
(x − y) 1 − 2 2 + (n − 2)( y − z) 1 − 2 2 + (n − 2)(z − x) 1 − 2 2 ≥ 0.
2 2 2
x y y z z x
EV Method for Nonnegative Variables 477
From
n − 1 ≤ x 2 [ y 2 + (n − 2)z 2 ] ≤ (n − 1)x 2 z 2 ,
it follows that
xz ≥ 1, yz ≥ 1.
Thus, suffices to show that
1 1
(x − y)2 1 − + (n − 2)(z − x)2
1 − ≥ 0,
x2 y2 z2 x 2
that is
x 2 2 1 x 2 1
(n − 2) 1 − z − 2 ≥ 1− 2
−y .
z x y x2
Since
x x
1− ≥ 1 − ≥ 0,
z y
it suffices to show that
1 1
(n − 2) z − 2 ≥ 2 − y 2 ,
2
x x
that is equivalent to the hypothesis
n−1
y 2 + (n − 2)z 2 ≥ .
x2
1
The equality occurs for a1 = a2 = · · · = an ≥ 1 and for = a2 = · · · = an ≥ 1.
a1
Remark. Since a1 (a2 + a3 + · · · + an ) ≥ n − 1 yields a12 (a22 + a32 + · · · + an2 ) ≥ n − 1,
the inequality is also true for
a1 (a2 + a3 + · · · + an ) ≥ n − 1.
In addition, it is true in the particular case
a1 , a2 , . . . , an ≥ 1.
P 5.100. Let
F (a1 , a2 , . . . , an ) = a1 + a2 + · · · + an − n n a1 a2 · · · an ,
p
v
[ y + (n − 2)z][(n − 2) y + z] t x n−1
≥ y + (n − 2)z + x − n[ y + (n − 2)z]
n
.
yz yz n−2
For fixed y and z, write this inequality as f (x) ≥ 0, x ∈ (0, y]. We will show that
f (x) ≥ f ( y) ≥ 0.
v v
u
t y n−2 n−2
z n yz
n−1
t
≥ (n − 1) + +n−3 · (n − 1) .
z y x n−1
Thus, it suffices to show that
v v
y + (n − 2)z
u
t y z n−2 n−2
n yz
n−1
t
+ +n−3 · (n − 1) n−1
≥ p ,
z y x n
x yz n−2
which is equivalent to
n−2
y z
(n − 1) + +n−3 yz n−2 ≥ [ y + (n − 2)z]n−1 .
z y
Due to homogeneity, we may set z = 1, when the inequality becomes
(n − 1)Ay ≥ y + n − 2,
where n−2
y + 1/ y + n − 3
A= , 0 < y ≤ 1.
y +n−2
By Bernoulli’s inequality, we have
1/ y − 1 n−2 (n − 2)(1/ y − 1) y2 + n − 2
A= 1+ ≥1+ = ,
y +n−2 y +n−2 y( y + n − 2)
hence
(n − 1)( y 2 + n − 2)
(n − 1)Ay − ( y + n − 2) ≥ − ( y + n − 2)
y +n−2
(n − 2)( y − 1)2
= ≥0.
y +n−2
The inequality f ( y) ≥ 0 has the form
+ (n
p y[ y − 2)z] 2 n − 2 n
2 y + (n − 2)z − n +
n
y 2 z n−2 ≥ −p .
n−1 y z n
y 2 z n−2
Due to homogeneity, we may set z = 1 (hence 0 < y ≤ 1), when the inequality
becomes
+
p y( y n − 2) 2 n
2 y + n − 2 − n y2 ≥ +n−2− p
n
.
n−1 y n
y2
Denoting
p
t= n
y, 0 < t ≤ 1,
we need to show that g(t) ≥ 0, where
= −(n − 2)t 2n + nt 2n−2 − (n − 2)(n − 4)t n + n(n − 2)t n−2 − n(n − 1)t 2 + (n − 2)(n − 3) .
For n = 3, we have
g(t) = t(1 − t)3 (3 + 3t + t 2 ) ≥ 0,
and for n = 4, we have
g(t) = 2(1 − t 2 )3 (1 + t 2 ) ≥ 0.
For n ≥ 5, we have
g 0 (t) = nt g1 (t),
g1 (t) = −2(n − 2)t 2n−2 + 2(n − 1)t 2n−4 − (n − 2)(n − 4)t n−2 + (n − 2)2 t n−4 − 2(n − 1),
g10 (t) = (n − 2)t n−5 (1 − t 2 )[4(n − 1)t n + n − 2] ≥ 0 ,
hence g1 (t) is increasing, g1 (t) ≤ g1 (1) = 0, g 0 (t) ≤ 0, g(t) is decreasing, g(t) ≥
g(1) = 0. Thus, the proof is completed. The equality holds for a1 = a2 = · · · =
an ≥ 1.
Remark 1. Since a1n−1 a2 a3 · · · an ≥ 1 yields a1 (a2 + a3 + · · · + an ) ≥ n − 1, the
inequality
1 1 1
F (a1 , a2 , . . . , an ) ≥ F , ,...,
a1 a2 an
is also valid if a1 , a2 , . . . , an are positive real numbers such that
a1 ≤ a2 ≤ · · · ≤ an , a1n−1 a2 a3 · · · an ≥ 1.
a1 , a2 , . . . , an ≥ 1.
P 5.101. Let
v
t a2 + a2 + · · · + a2
1 2 n a1 + a2 + · · · + an
F (a1 , a2 , . . . , an ) = − ,
n n
where a1 , a2 , . . . , an are positive real numbers such that a1 ≤ a2 ≤ · · · ≤ an and
a1n−1 (a2 + a3 + · · · + an ) ≥ n − 1.
EV Method for Nonnegative Variables 481
Then,
1 1 1
F (a1 , a2 , . . . , an ) ≥ F , ,..., .
a1 a2 an
(Vasile C., 2020)
Solution. For n = 2, we need to show that a1 a2 ≥ 1 involves
q
(a1 a2 − 1)( 2(a12 + a22 ) − a1 − a2 ) ≥ 0,
x n−1 [ y + (n − 2)z] ≥ n − 1,
1
y2 + n−2
z2 + n−1
w2
≥Ç q ,
1 1 1
x2 + y2 + n−2
z2 + x2 + n−1
w2
which is true if
1 1 1
·p +p
n−1 n yz[ y + (n − 2)z]
p
x 2 + y 2 + (n − 2)z 2 + x 2 + (n − 1)w2
y 2 + 2(n − 1) yz + (n − 2)z 2 1
≥ ·Ç .
y 2 z 2 [ y + (n − 2)z]2
q
1 1 1
x2 + + + +
n−2 n−1
y2 z2 x2 w2
y 2 + 2(n − 1) yz + (n − 2)z 2 1
≥ ·Ç .
y 2 z 2 [ y + (n − 2)z]2
Ç
2 1
y2 + n−2
z2 + y2 + n−1
w2
y 2 + 2(n − 1) yz + (n − 2)z 2 1
≥ ·Ç .
y 2 z 2 [ y + (n − 2)z]2
Ç
2 1
y2 + n−2
z2 + y2 + n−1
z2
Since
y 2 + 2(n − 1) yz + (n − 2)z 2 = [ y 2 + (n − 2)z 2 ] + 2(n − 1) yz,
we rewrite the inequality as
A + B ≥ C + D,
where
1 1
A= ·p ,
n−1 2 y 2 + (n − 2)z 2 + y 2 + (n − 1)z 2
p
1
B=p ,
n yz[ y + (n − 2)z]
y 2 + (n − 2)z 2 1
C= ·Ç ,
y 2 z 2 [ y + (n − 2)z]2
Ç
2 1
y2 + n−2
z2 + y2 + n−1
z2
EV Method for Nonnegative Variables 483
2(n − 1) yz 1
D= ·Ç .
y 2 z 2 [ y + (n − 2)z]2
Ç
2 1
y2 + n−2
z2 + y2 + n−1
z2
Since
p p 2z + n − 2 z + n − 1 3z + 2n − 3
2z 2 + n − 2 + z2 + n − 1 ≥ p + p = p ,
n n n
it is sufficient to show that
which is equivalent to
y n−1 [ y + (n − 2)z] ≥ n − 1.
that is
1
≥
2 y 2 + (n − 2)z 2 + y 2 + (n − 1)z 2
p p
(n − 1)[ y 2 + (n − 2)z 2 ] C0
≥ ·Ç ,
y 2 z 2 [ y + (n − 2)z]
Ç
2 1
y2 + n−2
z2 + y2 + n−1
z2
and
p [1 + (n − 2)z 2 ][2(n − 2)z + n2 − 3n + 4] Æ
z2 + n − 1 ≥ (n − 2)z 2 + 2 .
nz[1 + (n − 2)z]2
Since
2z 2 + n − 2 z2 + n − 1 (n − 3)(z 2 − 1)2
− = ≥0,
(n − 1)z 2 + 1 (n − 2)z 2 + 2 [n − 1)z 2 + 1][(n − 2)z 2 + 2]
it suffices to prove the second inequality. After squaring and making many cal-
culations, this inequality can be written as (z − 1)P(z) ≥ 0, where P(z) ≥ 0 for
z ≥ 1.
To complete the proof, we need to show that E(x, w, w) ≥ 0 for x n−1 w ≥ 1.
Write the required inequality as follows:
v
1 n−1 1 n−1
Æ t
n[x + (n − 1)w ] − [x + (n − 1)w] ≥ n 2 +
2 2 − + ,
x w2 x w
This is true if
(n − 1)x + w 1 x + (n − 1)w
Æ Æ
(n − 1)x 2 + w2 + p ≥ · x + (n − 1)w +
2 2 p .
n xw n
(n − 1)x + 1 + − 1
p
Æ n−2 x n
(n − 1)x 2 + 1 + p ≥x n x2 + n − 1 + p .
n n
and
(n − 1)x + 1 n−2 x +n−1
p ≥x n · p .
n n
Replacing x with x 2 in the second inequality gives the first inequality. Thus,it suf-
fices to prove the second inequality, which can be rewritten as f (x) ≥ 0, where
n−2
f (x) = ln[(n − 1)x + 1] − ln(x + n − 1) − ln x .
n
From
1 1 1
F (a1 , a2 , . . . , an ) ≥ F , ,...,
a1 a2 an
a1 , a2 , . . . , an ≥ 1.
a1 + a2 + · · · + an = n, an = max{a1 , a2 , . . . , an },
then
1 1 1
n + + ··· + ≥ 4(a12 + a22 + · · · + an2 ) + n(n − 5).
a1 a2 an−1
(Vasile C., 2021)
486 Vasile Cîrtoaje
1 1 1
then the sum + + ··· + is minimal for a1 = a2 = · · · = an−2 ≤ an−1 .
a1 a2 an−1
Therefore, it suffices to consider the case a1 = a2 = · · · = an−2 , that is to show that
F (a, b) ≥ 0, where
n−2 1
F (a, b) = n + − 4(n − 2)a2 − 4b2 − 4c 2 − n(n − 5), c = n − (n − 2)a − b,
a b
with a, b positive real numbers such that a ≤ b ≤ c. From c ≥ b, we get
(n − 2)a + 2b ≤ n.
we get
n2 n(4 − n)
4ab − n ≤ −n= ≤ 0.
2(n − 2) n−2
In addition,
n(n − 1)
F (t, t) = − 4(n − 1)t 2 − 4[n − (n − 1)t]2 − n(n − 5)
t
EV Method for Nonnegative Variables 487
Sn = f (x 1 ) + f (x 2 ) + · · · + f (x n )
EV-Lemma. Let a, b, c be fixed real numbers, not all equal, and let x, y, z be real
numbers satisfying
x ≤ y ≤ z, x + y + z = a + b + c, x k + y k + z k = ak + bk + c k ,
where k is an even positive integer. Then, there exist two real numbers m and M so
that m < M and
(1) y ∈ [m, M ];
489
490 Vasile Cîrtoaje
a+b+c k
a +b +c ≥3
k k k
,
3
with equality if and only if a = b = c.
According to the relations
x + z = a + b + c − y, x k + z k = ak + bk + c k − y k ,
we get
y k−1 − z k−1 y k−1 − x k−1
x0 = , z 0
= . (*)
z k−1 − x k−1 x k−1 − z k−1
The two-sided inequality
x( y) ≤ y ≤ z( y)
is equivalent to the inequalities f1 ( y) ≤ 0 and f2 ( y) ≥ 0, where
f1 ( y) = x( y) − y, f2 ( y) = z( y) − y.
x ≤ y ≤ z, x + y + z = a + b + c, x k + y k + z k = ak + bk + c k ,
EV Method for Real Variables 491
S = f (x) + f ( y) + f (z)
Proof. If a = b = c, then
a+b+c k
a= b=c ⇒ a +b +c =3 k k k
3
x + y + z k
⇒ x k + y k + zk = 3 ⇒ x = y = z.
3
Consider further that a, b, c are not all equal. As it is shown in the proof of EV-
Lemma, we have x < z. According to the relations
x + z = a + b + c − y, x k + z k = ak + bk + c k − y k ,
F 0 ( y) = x 0 f 0 (x) + f 0 ( y) + z 0 f 0 (z)
y k−1 − z k−1 y k−1 − x k−1
= k−1 g(x ) + g( y ) + k−1
k−1 k−1
g(z k−1 ),
z −x k−1 x −z k−1
F 0 ( y) g(x k−1 )
=
( y k−1 − x k−1 )( y k−1 − z k−1 ) (x k−1 − y k−1 )(x k−1 − z k−1 )
g( y k−1 ) g(z k−1 )
+ k−1 + .
(y − z k−1 )( y k−1 − x k−1 ) (z k−1 − x k−1 )(z k−1 − y k−1 )
Since g is strictly convex, the right hand side is positive. Moreover, since
This result contradicts the assumption that Sn attains its maximum value at (b1 , b2 , . . . , bn )
with b1 < bn−1 . Similarly, we can prove that Sn is minimum for x 2 = x 3 = · · · = x n .
Taking k = 2 in EV-Theorem, we obtain the following corollary.
Corollary 1. Let a1 , a2 , . . . , an (n ≥ 3) be fixed real numbers, and let x 1 , x 2 , . . . , x n
be real variables so that
x1 ≤ x2 ≤ · · · ≤ x n,
x 1 + x 2 + · · · + x n = a1 + a2 + · · · + an ,
x 12 + x 22 + · · · + x n2 = a12 + a22 + · · · + an2 .
If f is a differentiable function on R so that the derivative f 0 is strictly convex on R,
then the sum
Sn = f (x 1 ) + f (x 2 ) + · · · + f (x n )
is minimum for x 2 = x 3 = · · · = x n , and is maximum for x 1 = x 2 = · · · = x n−1 .
m(m − 1) k−1
p
g 0 (x) = x m−k
k−1
is strictly increasing on R.
x 1 + x 2 + · · · + x n = a1 + a2 + · · · + a n ,
Proof. The proof is based on EV-Lemma. Without loss of generality, assume that
x ≤ y ≤ z. For the nontrivial case when a, b, c are not all equal (which involves
x < z), consider the function of y
F ( y) = x 4 ( y) + y 4 + z 4 ( y).
x + y + b4 = −x + b4 6= 0,
x 1 + x 2 + · · · + x n = a1 + a2 + · · · + an ,
Sn = x 1m + x 2m + · · · + x nm
x + y + z = a + b + c, x 2 + y 2 + z 2 = a2 + b2 + c 2 .
Sm = x m + y m + z m
Proof. Consider the nontrivial case where a, b, c are not all equal. Let
p = a + b + c, q = ab + bc + ca, r = x yz.
which is equivalent to
Note. The EV-Theorem for real variables and Corollary 1 are also valid under the
conditions in Note 2 and Note 3 from the preceding chapter 5, where m, M ∈ R.
496 Vasile Cîrtoaje
EV Method for Real Variables 497
6.2 Applications
76
(a + b + c + d − 4) a + b + c + d +
2 2 2 2 2 2 2 2
≥ 8(a3 + b3 + c 3 + d 3 − 4).
3
E = a2 + b2 + c 2 + d 2 − 4, F = a3 + b3 + c 3 + d 3 − 4.
Prove that v
tE
E + 3 ≥ F.
3
2 m 2 m
(n − 1) 1 + − n− ≤ a1m + a2m + · · · + anm ≤ nm − n + 1.
n n
2 m 2 m
n−1≤ a1m + a2m + ··· + anm ≤ (n − 1) 1 − + 2− .
n n
then
a15 + a25 + · · · + an5 ≥ n − 1.
EV Method for Real Variables 499
a3 + b3 + c 3 + 3 ≥ 2(a + b + c).
then
a14 + a24 + · · · + an4 ≤ n(n − 1)(n2 − 3n + 3).
then
a14 + a24 + · · · + an4 ≤ 16n4 + n − 1.
then
a14 + a24 + · · · + an4 ≥ n(n2 − 1)(n2 + 3).
then
a14 + a24 + · · · + an4 ≥ n4 + (n − 1)(n + 1)4 .
then
a14 + a24 + · · · + an4 ≥ (n + 1)4 + (n − 1)n4 .
500 Vasile Cîrtoaje
then
a14 + a24 + · · · + an4 ≥ 2n4 + (n − 2)(n + 1)4 .
then
a16 + a26 + · · · + an6 ≤ (n − 1)6 + n − 1.
then
a16 + a26 + · · · + an6 ≤ n6 + n − 1.
then
a18 + a28 + · · · + an8 ≤ (n − 1)8 + n − 1.
then
a18 + a28 + · · · + an8 ≤ n8 + n − 1.
EV Method for Real Variables 501
−5
6.27. Let a, b, c, d, e 6= be real numbers so that a + b + c + d + e = 5. Then,
4
a(a − 1) b(b − 1) c(c − 1) d(d − 1) e(e − 1)
+ + + + ≥ 0.
(4a + 5)2 (4b + 5)2 (4c + 5)2 (4d + 5)2 (4e + 5)2
6.3 Solutions
a + b + c + d = 4, a2 + b2 + c 2 + d 2 = const ant,
then
S4 = a3 + b3 + c 3 + d 3
is maximum for a = b = c ≤ d.
Thus, we only need to show that 3a + d = 4 involves
8 2 64
3a + d +
2 2
≥ 4 3a + d +
3 3
.
3 9
This inequality is equivalent to
a1 + a2 + · · · + an = n,
then
2 n4 (n2 + 16n − 16)
n3
a12 + a22 + ··· + an2 + ≥ n a13 + a23 + · · · + an3 + ,
8n − 8 64(n − 1)2
with equality for a1 = a2 = · · · = an = 1, and also for
n n
a1 = a2 = · · · = an−1 = , an =
2n − 2 2
(or any cyclic permutation).
504 Vasile Cîrtoaje
3a + d = 4
involves
76
(3a + d − 4) 3a + d +
2 2 2
2
≥ 8(3a3 + d 3 − 4).
3
This inequality is equivalent to
a1 + a2 + · · · + an = n,
then
2 n(n2 + n − 1)
a12 + ··· + an2 − n a1 + · · · + an +
2
≥ 2n a13 + · · · + an3 − n ,
n−1
with equality for a1 = a2 = · · · = an = 1, and also for
1
a1 = a2 = · · · = an−1 = , an = n − 1
n−1
(or any cyclic permutation).
2a + c = 3
involves
(2a2 + c 2 − 3)(2a2 + c 2 + 93) ≥ 24(2a3 + c 3 − 3).
This inequality is equivalent to
(a2 − 1)2 ≥ 0.
a = b = −1, c=5
a = b = 1 − k, c = 1 + 2k
3a + d = 4
involves
(3a2 + d 2 − 4)(3a2 + d 2 + 116) ≥ 24(3a3 + d 3 − 4).
This inequality is equivalent to
(a2 − 1)2 ≥ 0.
506 Vasile Cîrtoaje
a = b = c = −1, d =7
a1 + a2 + · · · + an = n.
a1 = · · · = an−1 = 7, an = 7 − 6n
E = a2 + b2 + c 2 + d 2 − 4, F = a3 + b3 + c 3 + d 3 − 4.
Prove that v
tE
E + 3 ≥ F.
3
Solution. As shown in the proof of P 6.1, we only need to prove the desired in-
equality for 3a + d = 4 and
E = 3a2 + d 2 − 4, F = 3a3 + d 3 − 4.
Since
E = 12(1 − a)2 , F = 12(5 − 2a)(1 − a)2 ,
we get
v
tE
E + 3 − F = 12(1 − a)2 (2|1 − a| + 3) − 12(5 − 2a)(1 − a)2
3
= 24(1 − a)2 [|1 − a| − (1 − a)] ≥ 0.
Then, v
t E
E (n − 2) + 3 ≥ F,
n(n − 1)
with equality for
n − an
a1 = · · · = an−1 = ≤1
n−1
(or any cyclic permutation).
a1 ≤ a2 ≤ · · · ≤ an .
a1 + a2 = 0, a12 + a22 = 2
implies
a1m + a2m ≤ 0.
We have
a1 = −1, a2 = 1,
therefore a1m + a2m = 0. Assume now that n ≥ 3. According to Corollary 2, the sum
involve
(n − 1)a m + b m ≤ (n − 1)m − n + 1.
From the equations above, we get
a = −1, b = n − 1;
therefore,
a1 = · · · = an−1 = −1, an = n − 1
a1 = −n + 1, a2 = a3 = · · · = an = 1
2 m 2 m
(n − 1) 1 + − n− ≤ a1m + a2m + · · · + anm ≤ nm − n + 1.
n n
a1 ≤ a2 ≤ · · · ≤ an .
a1 + a2 = 1, a12 + a22 = 5,
implies
2m − 1 ≤ a1m + a2m ≤ 2m − 1.
We have
a1 = −1, a2 = 2,
for which a1m + a2m = 2m − 1. Assume now that n ≥ 3.
(a) Consider the right inequality. According to Corollary 2, the sum
involve
(n − 1)a m + b m ≤ nm − n + 1.
From the equations above, we get
a = −1, b = n;
therefore,
(n − 1)a m + b m = (n − 1)(−1)m + nm = nm − n + 1.
The equality holds for
a1 = a2 = · · · = an−1 = −1, an = n
involve
2 m 2 m
a + (n − 1)b ≥ (n − 1) 1 +
m m
− n− .
n n
From the equations above, we get
2 2
a = −n + , b =1+ ;
n n
therefore,
2 m 2 m
a + (n − 1)b = −n +
m m
+ (n − 1) 1 +
n n
m m
2 2
= (n − 1) 1 + − n− .
n n
The equality holds for
2 2
a1 = −n + , a2 = a3 = · · · = an = 1 +
n n
(or any cyclic permutation).
a1 ≤ a2 ≤ · · · ≤ an .
a1 + a2 = 1, a12 + a22 = 1,
EV Method for Real Variables 511
implies
1 ≤ a1m + a2m ≤ 1.
We have
a1 = 0, a2 = 1,
when a1m + a2m = 1. Assume now that n ≥ 3.
(a) Consider the left inequality. According to Corollary 2, the sum
involve
a m + (n − 1)b m ≤ n − 1 − (n − 2)m .
From the equations above, we get
a = 2 − n, b = 1;
therefore,
a1 = 2 − n, a2 = a3 = · · · = an = 1
involve m
2 2 m
(n − 1)a + b ≤ n − 2 +
m m
− (n − 1) 1 − .
n n
From the equations above, we get
2 2
a = −1 + , b = n−2+ ;
n n
512 Vasile Cîrtoaje
therefore,
2 m 2 m
(n − 1)a + b = (n − 1) −1 +
m m
+ n−2+
n n
m
2 2 m
= n−2+ − (n − 1) 1 − .
n n
The equality holds for
2 2
a1 = · · · = an−1 = −1 + , an = n − 2 +
n n
(or any cyclic permutation).
2 m 2 m
n−1≤ a1m + a2m + ··· + anm ≤ (n − 1) 1 − + 2− .
n n
a1 ≤ a2 ≤ · · · ≤ an .
a1 + a2 = 1, a12 + a22 = 1,
implies
1 ≤ a1m + a2m ≤ 1.
The above equations involve
a1 = 0, a2 = 1,
involve
a m + (n − 1)b m ≥ n − 1.
From the equations above, we get
a = 0, b = 1;
therefore,
a m + (n − 1)b m = n − 1.
The equality holds for
a1 = 0, a2 = · · · = a n = 1
(or any cyclic permutation).
(b) Consider the right inequality. According to Corollary 2, the sum
involve
2 m 2 m
(n − 1)a m + b m ≤ (n − 1) 1 − + 2− .
n n
From the equations above, we get
2 2
a =1− , b =2− ,
n n
when
2 m 2 m
(n − 1)a + b = (n − 1) 1 −
m m
+ 2− .
n n
a1 = · · · = an−1 = −1, an = n + k − 1
a1 ≤ a2 ≤ · · · ≤ an .
a1 + a2 = 3, a12 + a22 = 5,
implies
2m + 1 ≤ a1m + a2m ≤ 2m + 1.
We get
a1 = 1, a2 = 2,
when a1m + a2m = 2 + 1. Assume now that n ≥ 3.
m
involve m
2 2 m
a + (n − 1)b ≥
m m
+ (n − 1) 1 + .
n n
From the equations
a + (n − 1)b = n + 1, a2 + (n − 1)b2 = n + 3,
we get
2 2
a= , b =1+ ;
n n
therefore, m
2 2 m
a + (n − 1)b =
m m
+ (n − 1) 1 + .
n n
The equality holds for
2 2
a1 = , a2 = · · · = a n = 1 +
n n
(or any cyclic permutation).
(b) Consider the right inequality. According to Corollary 2, the sum
involve
(n − 1)a m + b m ≤ 2m + n − 1.
From the equations
(n − 1)a + b = n + 1, (n − 1)a2 + b2 = n + 3,
we get
a = 1, b = 2;
therefore,
(n − 1)a m + b m = n − 1 + 2m .
The equality holds for
a1 = · · · = an−1 = 1, an = 2
2k 2k
a1 = − 1 + n − k, a2 = · · · = an = −1
n n
(or any cyclic permutation). The right inequality is an equality for
a1 = · · · = an−1 = 1, an = k − n + 1
then
a15 + a25 + · · · + an5 ≥ n − 1.
a1 + a2 = 1, a14 + a24 = 1,
implies
a15 + a25 ≥ 1.
We have
a1 = 0, a2 = 1,
or
a1 = 1, a2 = 0.
For each of these cases, the inequality is an equality. Assume now that n ≥ 3 and
a1 ≤ a2 ≤ · · · ≤ an .
EV Method for Real Variables 517
involve
a5 + (n − 1)b5 ≥ n − 1.
The equations
a + (n − 1)b = n − 1, a4 + (n − 1)b4 = n − 1,
are equivalent to
that is,
b = 1, a = 0,
and
a3 = 1 + b + b2 + b3 , a = (n − 1)(1 − b).
For the second case, the condition a ≤ b involves
b3 ≥ 1 + b + b2 + b3 ,
a5 + (n − 1)b5 ≥ n − 1
for a = 0 and b = 1, that is clearly true. Thus, the proof is completed. The equality
holds for
a1 = 0, a2 = · · · = a n = 1
(or any cyclic permutation).
a2 + b2 + c 2 = 3,
then
a3 + b3 + c 3 + 3 ≥ 2(a + b + c).
a + b + c = const ant, a2 + b2 + c 2 = 3,
the sum
S3 = a3 + b3 + c 3
is minimum for a ≤ b = c. Thus, we only need to show that
a2 + 2b2 = 3, a ≤ b,
involves
a3 + 2b3 + 3 ≥ 2(a + 2b).
We will show this by two methods. From a2 + 2b2 = 3 and a ≤ b, it follows that
v v
p t3 t3
− 3 ≤ a ≤ 1, − <b≤ .
2 2
a3 − 2a + 3 ≥ b(a2 + 1).
For a ≥ 0, we have
a3 − 2a + 3 ≥ −2a + 3 > 0,
and for a ≤ 0, we have
which is equivalent to
(a − 1)2 f (a) ≥ 0,
where
f (a) = a4 + 2a3 + 2a + 5.
We need to prove that f (a) ≥ 0. For a ≥ −1, we have
then
a14 + a24 + · · · + an4 ≤ n(n − 1)(n2 − 3n + 3).
a1 + a2 = 0, a12 + a22 = 2,
implies
a14 + a24 ≤ 2.
We have
a1 = −1, a2 = 1,
or
a1 = 1, a2 = −1.
For each of these cases, the desired inequality is an equality. Assume now that
n ≥ 3. According to Theorem 1, the sum
is maximum for
a1 = · · · = a j , a j+1 = · · · = an ,
where j ∈ {1, 2, . . . , n − 1}. Thus, we only need to show that
involve
ja14 + (n − j)an4 ≤ n(n − 1)(n2 − 3n + 3).
From the equations above, we get
(n − j)(n − 1) j(n − 1)
a12 = , an2 = ;
j n− j
therefore,
(n − j)3 + j 3 n2
ja14 + (n − j)an4 = (n − 1) =
2
− 3 n(n − 1)2 .
j(n − j) j(n − j)
Since
j(n − j) − (n − 1) = ( j − 1)(n − j − 1) ≥ 0,
EV Method for Real Variables 521
we get
n2
ja14 + (n − j)an4 ≤ − 3 n(n − 1)2 = n(n − 1)(n2 − 3n + 3).
n−1
a1 = −n + 1, a2 = · · · = an = 1
and for
a1 = n − 1, a2 = · · · = an = −1
(or any cyclic permutation).
then
a14 + a24 + · · · + an4 ≤ 16n4 + n − 1.
then
a14 + a24 + · · · + a2n+1
4
≤ 2n(2n + 1)(4n2 − 2n + 1),
with equality for
a1 = −2n, a2 = · · · = a2n+1 = 1
and for
a1 = 2n, a2 = · · · = a2n+1 = −1
(or any cyclic permutation).
Putting
an+1 = · · · = a2n+1 = −1,
it follows that
involve
a14 + a24 + · · · + an4 + n + 1 ≤ 2n(2n + 1)(4n2 − 2n + 1).
This is equivalent to the desired statement. The equality holds for
a1 = 2n, a2 = · · · = an = −1
then
a14 + a24 + · · · + an4 ≥ n(n2 − 1)(n2 + 3).
is minimum for
a1 = · · · = a j , a j+1 = · · · = an ,
where j ∈ {1, 2, . . . , n − 1}. Thus, we only need to show that
involve
ja14 + (n − j)an4 ≤ n(n2 − 1)(n2 + 3).
From the equations above, we get
(n − j)(n2 − 1) j(n2 − 1)
a12 = , an2 = ;
j n− j
therefore,
(n − j)3 + j 3 2 n2
ja14 + (n − j)an4 = (n − 1) =
2
− 3 n(n2 − 1)2 .
j(n − j) j(n − j)
Since
n2 − 1 (n − 2 j)2 − 1
− j(n − j) = ≥ 0,
4 4
we get
4n2
ja14 + (n − j)an4 ≥ 2
− 3 n(n2 − 1)2 = n(n2 − 1)(n2 + 3).
n −1
EV Method for Real Variables 523
n−1
The equality holds when of a1 , a2 , . . . , an are equal to −n − 1 and the other
2
n+1 n−1
are equal to n − 1, and also when of a1 , a2 , . . . , an are equal to n + 1
2 2
n+1
and the other are equal to −n + 1.
2
then
a14 + a24 + · · · + an4 ≥ n4 + (n − 1)(n + 1)4 .
then
a14 + a24 + · · · + a2n+1
4
≥ n(n + 1)(2n + 1)(n2 + n + 1),
with equality when n of a1 , a2 , . . . , a2n+1 are equal to −n − 1 and the other n + 1 are
equal to n, and also when n of a1 , a2 , . . . , a2n+1 are equal to n + 1 and the other n + 1
are equal to −n.
Putting
an+1 = · · · = a2n = −n, a2n+1 = n + 1,
it follows that
a1 + a2 + · · · + an + n(−n) + (n + 1) = 0
and
a12 + a22 + · · · + an2 + n(−n)2 + (n + 1)2 = n(n + 1)(2n + 1)
involve
a1 = · · · = an−1 = n + 1, an = −n
then
a14 + a24 + · · · + an4 ≥ (n + 1)4 + (n − 1)n4 .
(Vasile Cîrtoaje, 2010)
Solution. As shown in the proof of the preceding P 6.16, the following statement
holds:
• If a1 , a2 , . . . , a2n+1 are real numbers so that
then
a14 + a24 + · · · + a2n+1
4
≥ n(n + 1)(2n + 1)(n2 + n + 1),
with equality when n of a1 , a2 , . . . , a2n+1 are equal to −n − 1 and the other n + 1 are
equal to n, and also when n of a1 , a2 , . . . , a2n+1 are equal to n + 1 and the other n + 1
are equal to −n.
Putting
an+1 = · · · = a2n−1 = −n − 1, a2n = a2n+1 = n,
it follows that
a1 + a2 + · · · + an + (n − 1)(−n − 1) + 2n = 0
and
a12 + a22 + · · · + an2 + (n − 1)(−n − 1)2 + 2n2 = n(n + 1)(2n + 1)
involve
a14 + a24 + · · · + an4 + (n − 1)(−n − 1)4 + 2n4 ≤ n(n + 1)(2n + 1)(n2 + n + 1),
a1 = −n − 1, a2 = · · · = a n = n
then
a14 + a24 + · · · + an4 ≥ 2n4 + (n − 2)(n + 1)4 .
(Vasile Cîrtoaje, 2010)
EV Method for Real Variables 525
then
a14 + a24 + · · · + a2n+1
4
≥ n(n + 1)(2n + 1)(n2 + n + 1),
with equality when n of a1 , a2 , . . . , a2n+1 are equal to −n − 1 and the other n + 1 are
equal to n, and also when n of a1 , a2 , . . . , a2n+1 are equal to n + 1 and the other n + 1
are equal to −n.
Putting
an+1 = · · · = a2n−1 = −n, a2n = a2n+1 = n + 1,
it follows that
a1 + a2 + · · · + an + (n − 1)(−n) + 2(n + 1) = 0
and
a12 + a22 + · · · + an2 + (n − 1)(−n)2 + 2(n + 1)2 = n(n + 1)(2n + 1)
involve
a14 + a24 + · · · + an4 + (n − 1)(−n)4 + 2(n + 1)4 ≤ n(n + 1)(2n + 1)(n2 + n + 1),
a1 = a2 = −n, a3 = · · · = an = n + 1
Since
(1 − ac)(5 − ac) ≥ 0.
It is true because
1
ac ≤ (a + c)2 = 1.
4
Case 2: b = c = d. We need to show that a + 3b = 4 involves
Since
5
a = −1, b=c=d=
3
(or any cyclic permutation).
then
a16 + a26 + · · · + an6 ≤ (n − 1)6 + n − 1.
a1 + a2 = 0, a12 + a22 = 2,
implies
a16 + a26 ≤ 2.
We have
a1 = −1, a2 = 1,
or
a1 = 1, a2 = −1.
For each of these cases, the desired inequality is an equality. According to Theorem
2, the sum
Sn = a16 + a26 + · · · + an6
is maximum for
a1 = · · · = a j , a j+1 = · · · = an ,
where j ∈ {1, 2, . . . , n − 1}. Thus, we only need to show that
involve
ja16 + (n − j)an6 ≤ (n − 1)6 + n − 1.
From the equations above, we get
(n − j)(n − 1) j(n − 1)
a12 = , an2 = .
j n− j
(n − j)5 + j 5 (n − 1)5 + 1
≤ ,
j 2 (n − j)2 (n − 1)2
n
a = n − 1, b= − 1.
j
Since a ≥ b and
n−1
n
ab − 1 = (n − 1) −1 −1= n − 1 ≥ 0,
j j
we have
1 a+b
f (a) − f (b) = (a − b) a + b − 1 + −
ab a2 b2
1 1
= (a − b) 1 − (a + b) 1 + − 1 ≥ 0.
ab ab
The equality holds for
a1 = −n + 1, a2 = · · · = an = 1,
and for
a1 = n − 1, a2 = · · · = an = −1
(or any cyclic permutation).
then
a16 + a26 + · · · + an6 ≤ n6 + n − 1.
(Vasile Cîrtoaje, 2010)
Solution. The inequality follows from the preceding P 6.20 by replacing n with
n + 1, and then making an+1 = −1. The equality holds for
a1 = n, a2 = · · · = an = −1
then
a18 + a28 + · · · + an8 ≤ (n − 1)8 + n − 1.
(Vasile Cîrtoaje, 2010)
EV Method for Real Variables 529
a1 + a2 = 0, a12 + a22 = 2,
implies
a18 + a28 ≤ 2.
We have
a1 = −1, a2 = 1,
or
a1 = 1, a2 = −1.
For each of these cases, the desired inequality is an equality. According to Theorem
2, the sum
Sn = a18 + a28 + · · · + an8
is maximum for
a1 = · · · = a j , a j+1 = · · · = an ,
where j ∈ {1, 2, . . . , n − 1}. Thus, we only need to show that
involve
ja18 + (n − j)an8 ≤ (n − 1)8 + n − 1.
From the equations above, we get
(n − j)(n − 1) j(n − 1)
a12 = , an2 = .
j n− j
(n − j)7 + j 7 (n − 1)7 + 1
≤ ,
j 3 (n − j)3 (n − 1)4
(n − j)3 (n − j)2 n − j j j2 j3
− + + − + ≤
j3 j2 j n − j (n − j)2 (n − j)3
1 1 1
≤ (n − 1)3 − (n − 1)2 + (n − 1) + − + ,
n − 1 (n − 1)2 (n − 1)3
f (a) ≥ f (b),
where
n
a = n − 1, b= − 1,
j
1 1 1
f (x) = x 3 − x 2 + x + − 2 + 3, x > 0.
x x x
530 Vasile Cîrtoaje
Since
1
f (x) = (t − 1)(t 2 − 2), t=x+ ≥ 2,
x
it suffices to show that
1 1
a+ ≥ b+ .
a b
We have a ≥ b,
n−1
n
ab − 1 = (n − 1) −1 −1= n − 1 ≥ 0,
j j
therefore
1 1 1
a + − b − = (a − b) 1 − ≥ 0.
a b ab
The equality holds for
a1 = −n + 1, a2 = · · · = an = 1
and for
a1 = n − 1, a2 = · · · = an = −1
then
a18 + a28 + · · · + an8 ≤ n8 + n − 1.
Solution. The inequality follows from the preceding P 6.22 by replacing n with
n + 1, and making an+1 = −1. The equality holds for
a1 = n, a2 = · · · = an = −1
we only need to consider the case when C is minimum. Thus, according to Corollary
2, it suffices to prove the required inequality for a1 < a2 = a3 = · · · = an . Setting
a1 := a, a2 = a3 = · · · = an := b, a < b,
After dividing the numerator and denominator of the left fraction by (a − b)2 , the
inequality reduces to
v
−4n2 ab t 2n2
≤ 1 + 1 + ,
(n + 1)a2 + 2(n − 1)ab + (2n2 − 3n + 1)b n−1
−2ab 1
≤p ,
(n + 1)a2 + 2(n − 1)ab + (2n − 3n + 1)b
2
(n2 − 1)(2n − 1) − n + 1
v 2
t 2n2 − 3n + 1
a+ b ≥ 0.
n+1
The equality holds for
v
t n+1
− a1 = a2 = · · · = an
(n − 1)(2n − 1)
532 Vasile Cîrtoaje
a + b + c + d = 2,
then
3
a4 + b4 + c 4 + d 4 ≤ 40 + (a2 + b2 + c 2 + d 2 )2 .
4
(Vasile Cîrtoaje, 2010)
Solution. Write the inequality in the homogeneous form
a = 5, b = c = d = −1
Solution. We proceed as in the proof of the preceding P 6.25. Taking into account
Theorem 1, it suffices to consider the cases b = c = d = e, and a = b and c = d = e.
Case 1: b = c = d = e. Due to homogeneity, we may consider b = c = d = e = 0
and b = c = d = e = 1. The first case is trivial. In the second case, the inequality
becomes p
31 + 18 3 3
a +4≤
4
(a + 4)4 + (a2 + 4)2 ,
8 4
p 2 p
a+2+2 3 f (a) + 2 3 g(a) ≥ 0,
where
f (a) = 29a2 + 164a + 272, g(a) = 9a2 + 50a + 76.
It suffices to show that f (a) ≥ 0 and g(a) ≥ 0. Indeed, we have
82 2 1
f (a) > 25a + 164a + 269 = 5a +
2
+ > 0,
5 25
25 2 5
g(a) > 9a + 50a + 70 = 3a +
2
+ > 0.
3 9
−5
P 6.27. Let a, b, c, d, e 6= be real numbers so that a + b + c + d + e = 5. Then,
4
a(a − 1) b(b − 1) c(c − 1) d(d − 1) e(e − 1)
+ + + + ≥ 0.
(4a + 5)2 (4b + 5)2 (4c + 5)2 (4d + 5)2 (4e + 5)2
4a + 5 4b + 5 4e + 5
a1 = , a2 = , . . . , a5 = ,
9 9 9
we need to prove that a1 + a2 + a3 + a4 + a5 = 5 involves
5
2 5
X X
7 ai2 − 25 ≥ 20 ai4 .
i=1 i=1
By Theorem 1, for a1 + a2 + a3 + a4 + a5 = 5 and a12 + a22 + a32 + a42 + a52 = const ant,
the sum a14 + a24 + a34 + a44 + a54 is maximum when the set (a1 , a2 , a3 , a4 , a5 ) has at
most two distinct values. Therefore, we need to consider the following two cases.
Case 1: a1 = x and a2 = a3 = a4 = a5 = y. The homogeneous inequality reduces
to
(3x 2 + 6 y 2 − 4x y)2 ≥ 5(x 4 + 4 y 4 ),
EV Method for Real Variables 535
(x − y)2 (x − 2 y)2 ≥ 0.
5 5
a= , b=c=d=e=
2 8
(or any cyclic permutation).
Then,
x 1 (x 1 − 1) x 2 (x 2 − 1) x n (x n − 1)
+ + ··· + ≥ 0,
(x 1 + k) 2 (x 2 + k)2 (x n + k)2
n
with equality for x 1 = x 2 = · · · = x n = 1. If k = p , then the equality holds
2 n−1
also for
n n
x1 = , x2 = · · · = x n =
2 2(n − 1)
(or any cyclic permutation).
a + b + c = 9, ab + bc + ca = 15,
then
19 1 1 1 7
≤ 2 + 2 + 2 ≤ .
175 b + bc + c 2 c + ca + a 2 a + ab + b 2 19
(Vasile C., 2011)
536 Vasile Cîrtoaje
Solution. From
(b + c)2 ≥ 4bc
and
b + c = 9 − a, bc = 15 − a(b + c) = 15 − a(9 − a) = a2 − 9a + 15,
we get a ≤ 7. Since
b2 + bc + c 2 = (a + b + c)(b + c) − (ab + bc + ca) = 9(9 − a) − 15 = 3(22 − 3a),
we may write the inequality in the form
57 21
≤ f (a) + f (b) + f (c) ≤ .
175 19
where
1
f (u) = , u ≤ 7.
22 − 3u
We have
3
g(x) = f 0 (x) = ,
(22 − 3x)2
162
g 00 (x) = .
(22 − 3x)4
Since g 00 (x) > 0 for x ≤ 7, g is strictly convex on (−∞, 7]. According to Corollary
1, if a ≤ b ≤ c and
a + b + c = 9, a2 + b2 + c 2 = 51,
then the sum S3 = f (a) + f (b) + f (c) is maximum for a = b ≤ c, and is minimum
for a ≤ b = c.
(a) To prove the right inequality, it suffices to consider the case a = b ≤ c.
From
a + b + c = 9, ab + bc + ca = 15,
we get a = b = 1 and c = 7, therefore
1 1 1 7
+ 2 + 2 = .
b2 + bc + c 2 c + ca + a 2 a + ab + b 2 19
The original right inequality is an equality for a = b = 1 and c = 7 (or any cyclic
permutation).
(b) To prove the left inequality, it suffices to consider the case a ≤ b = c, which
involves a = −1 and b = c = 5, hence
1 1 1 19
+ + = .
b2 + bc + c 2 c 2 + ca + a2 a2 + ab + b2 175
The original left inequality is an equality for a = −1 and b = c = 5 (or any cyclic
permutation).
EV Method for Real Variables 537
then
419 a2 b2 c2 311
≤ 2 + + ≤ .
175 b + bc + c 2 c 2 + ca + a2 a2 + ab + b2 19
(Vasile C., 2011)
a + b + c = 9, a2 + b2 + c 2 = 51.
Next, the proof is similar to the one of the preceding P 6.28. Write the inequality
in the form
1257 933
≤ f (a) + f (b) + f (c) ≤ .
175 19
where
u2
f (u) = , u ≤ 7.
22 − 3u
We have
−3x 2 + 44x 8712
g(x) = f 0 (x) = , g 00 (x) = .
(22 − 3x) 2 (22 − 3x)4
Since g is strictly convex on (−∞, 7], according to Corollary 1, the sum S3 =
f (a) + f (b) + f (c) is maximum for a = b ≤ c, and is minimum for a ≤ b = c.
(a) To prove the right inequality, it suffices to consider the case a = b ≤ c,
which involves
a = b = 1, c = 7,
and
a2 b2 c2 311
+ + = .
b + bc + c
2 2 c + ca + a
2 2 a + ab + b
2 2 19
The original right inequality is an equality for a = b = c/7 (or any cyclic permuta-
tion).
(b) To prove the left inequality, it suffices to consider the case a ≤ b = c, which
involves a = −1 and b = c = 5, hence
a2 b2 c2 419
+ + = .
b + bc + c
2 2 c + ca + a
2 2 a + ab + b
2 2 175
The original left inequality is an equality for −5a = b = c (or any cyclic permuta-
tion).
538 Vasile Cîrtoaje
According to Corollary 2, for a1 +a2 +· · ·+an = const ant > 0 and a12 +a22 +· · ·+an2 =
const ant, the sum
S = a13 + a23 + · · · + an3
is maximal when n − 1 of a1 , a2 , . . . , an are equal. Therefore, it suffices to consider
the case a2 = a3 = · · · = an . Due to homogeneity, for the nontrivial case a2 = a3 =
· · · = an 6= 0, we may consider that a2 = a3 = · · · = an = 1. Thus we only need to
prove that
which is equivalent to
where
Glosar
a1 + a2 + · · · + an ≥ n n a1 a2 · · · an ,
p
p1 + p2 + · · · + pn = 1.
1 1 1
(a1 + a2 + · · · + an ) + + ··· + ≥ n2 ,
a1 a2 an
539
540 Vasile Cîrtoaje
5. BERNOULLI’S INEQUALITY
−1 ≤ a1 , a2 , . . . , an ≤ 0,
then
(1 + a1 )(1 + a2 ) · · · (1 + an ) ≥ 1 + a1 + a2 + · · · + an .
6. SCHUR’S INEQUALITY
For any nonnegative real numbers a, b, c and any positive number k, the inequality
holds
a k (a − b)(a − c) + b k (b − c)(b − a) + c k (c − a)(c − b) ≥ 0,
with equality for a = b = c, and for a = 0 and b = c (or any cyclic permutation).
For k = 1, we get the third degree Schur’s inequality, which can be rewritten as
follows
a3 + b3 + c 3 + 3abc ≥ ab(a + b) + bc(b + c) + ca(c + a),
(a + b + c)3 + 9abc ≥ 4(a + b + c)(ab + bc + ca),
9abc
a2 + b2 + c 2 + ≥ 2(ab + bc + ca),
a+b+c
Glosar 541
with equality for a = b = c, and also for a/m = b = c (or any cyclic permutation).
This inequality is equivalent to
X X X X
a + m(m + 2)
4
a b + (1 − m )abc
2 2 2
a ≥ (m + 1) ab(a2 + b2 ),
X
(b − c)2 (b + c − a − ma)2 ≥ 0.
7. CAUCHY-SCHWARZ INEQUALITY
If a1 , a2 , . . . , an and b1 , b2 , . . . , bn are real numbers, then
8. HÖLDER’S INEQUALITY
If x i j (i = 1, 2, · · · , m; j = 1, 2, · · · n) are nonnegative real numbers, then
n v !m
m
Y X n uY
X m
m
≥ .
t
xi j xi j
i=1 j=1 j=1 i=1
542 Vasile Cîrtoaje
9. CHEBYSHEV’S INEQUALITY
Let a1 ≥ a2 ≥ · · · ≥ an be real numbers.
a) If b1 ≥ b2 ≥ · · · bn , then
n
n
n
X X X
n ai bi ≥ ai bi ;
i=1 i=1 i=1
b) If b1 ≤ b2 ≤ · · · ≤ bn , then
n
n
n
X X X
n ai bi ≤ ai bi .
i=1 i=1 i=1
and
and
b1 + · · · + bi ≥ c1 + · · · + ci , i = 1, 2, · · · , n.
If a1 ≥ a2 ≥ · · · ≥ an ≥ 0, then
a1 b1 + a2 b2 + · · · + an bn ≥ a1 c1 + a2 c2 + · · · + an cn .
Notice that all these inequalities follow immediately from the identity
n n
i i
X X X X
ai (bi − ci ) = (ai − ai+1 ) bj − cj , an+1 = 0.
i=1 i=1 j=1 j=1
Glosar 543
p = a + b + c, q = ab + bc + ca, r = abc,
p p
s = p2 − 3q = a2 + b2 + c 2 − ab − bc − ca.
From the identity
it follows that
A = (a1 , a2 , . . . , an ), ai ∈ I,
B = (b1 , b2 , . . . , bn ), bi ∈ I,
then
f (a1 ) + f (a2 ) + · · · + f (an ) ≥ f (b1 ) + f (b2 ) + · · · + f (bn ).
We say that a sequence A = (a1 , a2 , . . . , an ) with a1 ≥ a2 ≥ · · · ≥ an majorizes a
sequence B = (b1 , b2 , . . . , bn ) with b1 ≥ b2 ≥ · · · ≥ bn , and write it as
A B,
if
a1 ≥ b1 ,
a1 + a2 ≥ b1 + b2 ,
·····················
a1 + a2 + · · · + an−1 ≥ b1 + b2 + · · · + bn−1 ,
a1 + a2 + · · · + an = b1 + b2 + · · · + bn .
544 Vasile Cîrtoaje
f (αx + β y) ≤ α f (x) + β f ( y)
Right Half Convex Function Theorem (Vasile Cîrtoaje, 2004). Let f be a real
function defined on an interval I and convex on I≥s , where s ∈ int(I). The inequality
a + a + ··· + a
1 2 n
f (a1 ) + f (a2 ) + · · · + f (an ) ≥ n f
n
holds for all a1 , a2 , . . . , an ∈ I satisfying
a1 + a2 + · · · + an = ns
if and only if
f (x) + (n − 1) f ( y) ≥ n f (s)
for all x, y ∈ I such that x ≤ s ≤ y and x + (n − 1) y = ns.
Left Half Convex Function Theorem (Vasile Cîrtoaje, 2004). Let f be a real function
defined on an interval I and convex on I≤s , where s ∈ int(I). The inequality
a + a + ··· + a
1 2 n
f (a1 ) + f (a2 ) + · · · + f (an ) ≥ n f
n
holds for all a1 , a2 , . . . , an ∈ I satisfying
a1 + a2 + · · · + an = ns
if and only if
f (x) + (n − 1) f ( y) ≥ n f (s)
Glosar 545
If a1 , a2 , . . . , an ∈ I such that
a1 + a2 + · · · + an = S = const ant,
then
(a) E is minimum for a1 = a2 = · · · = an−1 ≤ an ;
(b) E is maximum for either a1 = a or a < a1 ≤ a2 = · · · = an .
Right Half Convex Function Theorem for Ordered Variables (Vasile Cîrtoaje,
2008). Let f be a real function defined on an interval I and convex on I≥s , where
s ∈ int(I). The inequality
a + a + ··· + a
1 2 n
f (a1 ) + f (a2 ) + · · · + f (an ) ≥ n f
n
holds for all a1 , a2 , . . . , an ∈ I satisfying
a1 + a2 + · · · + an = ns
and
a1 ≤ a2 ≤ · · · ≤ am ≤ s, m ∈ {1, 2, . . . , n − 1},
if and only if
f (x) + (n − m) f ( y) ≥ (1 + n − m) f (s)
for all x, y ∈ I such that
x ≤ s ≤ y, x + (n − m) y = (1 + n − m)s.
Left Half Convex Function Theorem for Ordered Variables (Vasile Cîrtoaje, 2008).
Let f be a real function defined on an interval I and convex on I≤s , where s ∈ int(I).
The inequality
a + a + ··· + a
1 2 n
f (a1 ) + f (a2 ) + · · · + f (an ) ≥ n f
n
holds for all a1 , a2 , . . . , an ∈ I satisfying
a1 + a2 + · · · + an = ns
and
a1 ≥ a2 ≥ · · · ≥ am ≥ s, m ∈ {1, 2, . . . , n − 1},
546 Vasile Cîrtoaje
if and only if
f (x) + (n − m) f ( y) ≥ (1 + n − m) f (s)
for all x, y ∈ I such tht
x ≥ s ≥ y, x + (n − m) y = (1 + n − m)s.
Right Partially Convex Function Theorem (Vasile Cîrtoaje, 2012). Let f be a real
function defined on an interval I and convex on [s, s0 ], where s, s0 ∈ I, s < s0 . In
addition, f is decreasing on I≤s0 and f (u) ≥ f (s0 ) for u ∈ I. The inequality
a + a + ··· + a
1 2 n
f (a1 ) + f (a2 ) + · · · + f (an ) ≥ n f
n
holds for all a1 , a2 , . . . , an ∈ I satisfying
a1 + a2 + · · · + an = ns
if and only if
f (x) + (n − 1) f ( y) ≥ n f (s)
for all x, y ∈ I such that x ≤ s ≤ y and x + (n − 1) y = ns.
Left Partially Convex Function Theorem (Vasile Cîrtoaje, 2012). Let f be a real
function defined on an interval I and convex on [s0 , s], where s0 , s ∈ I, s0 < s. In
addition, f is increasing on I≥s0 and f (u) ≥ f (s0 ) for u ∈ I. The inequality
a + a + ··· + a
1 2 n
f (a1 ) + f (a2 ) + · · · + f (an ) ≥ n f
n
holds for all a1 , a2 , . . . , an ∈ I satisfying
a1 + a2 + · · · + an = ns
if and only if
f (x) + (n − 1) f ( y) ≥ n f (s)
for all x, y ∈ I such that x ≥ s ≥ y and x + (n − 1) y = ns.
Right Partially Convex Function Theorem for Ordered Variables (Vasile Cirtoaje,
2014). Let f be a real function defined on an interval I and convex on [s, s0 ], where
s, s0 ∈ I, s < s0 . In addition, f is decreasing on I≤s0 and f (u) ≥ f (s0 ) for u ∈ I. The
inequality a + a + ··· + a
1 2 n
f (a1 ) + f (a2 ) + · · · + f (an ) ≥ n f
n
holds for all a1 , a2 , . . . , an ∈ I satisfying
a1 + a2 + · · · + an = ns
Glosar 547
and
a1 ≤ a2 ≤ · · · ≤ am ≤ s, m ∈ {1, 2, . . . , n − 1},
if and only if
f (x) + (n − m) f ( y) ≥ (1 + n − m) f (s)
for all x, y ∈ I such that x ≤ s ≤ y and x + (n − m) y = (1 + n − m)s.
Left Partially Convex Function Theorem for Ordered Variables (Vasile Cirtoaje,
2014). Let f be a real function defined on an interval I and convex on [s0 , s], where
s0 , s ∈ I, s0 < s. In addition, f is increasing on I≥s0 and f (u) ≥ f (s0 ) for u ∈ I. The
inequality a + a + ··· + a
1 2 n
f (a1 ) + f (a2 ) + · · · + f (an ) ≥ n f
n
holds for all a1 , a2 , . . . , an ∈ I satisfying
a1 + a2 + · · · + an = ns
and
a1 ≥ a2 ≥ · · · ≥ am ≥ s, m ∈ {1, 2, . . . , n − 1},
if and only if
f (x) + (n − m) f ( y) ≥ (1 + n − m) f (s)
for all x, y ∈ I such that x ≥ s ≥ y and x + (n − m) y = (1 + n − m)s.
Equal Variables Theorem for Nonnegative Variables (Vasile Cirtoaje, 2005). Let
a1 , a2 , . . . , an (n ≥ 3) be fixed nonnegative real numbers, and let
0 ≤ x1 ≤ x2 ≤ · · · ≤ x n
such that
x 1 x 2 · · · x n = a1 a2 · · · an .
Sn = f (x 1 ) + f (x 2 ) + · · · + f (x n )
is maximum for
x 1 = x 2 = · · · = x n−1 ≤ x n ,
548 Vasile Cîrtoaje
Equal Variables Theorem for Real Variables (Vasile Cirtoaje, 2010). Let a1 , a2 , . . . , an
(n ≥ 3) be fixed real numbers, and let
0 ≤ x1 ≤ x2 ≤ · · · ≤ x n
such that
Sn = f (x 1 ) + f (x 2 ) + · · · + f (x n )
Best Upper Bound of Jensen’s Difference Theorem (Vasile Cirtoaje, 1990). Let
p1 , p2 , . . . , pn (n ≥ 3) be fixed positive real numbers, and let f be a convex function
on I = [a, b]. If a1 , a2 , . . . , an ∈ I, then Jensen’s difference
Bibliography
[1] Andreescu T., Cîrtoaje V., Dospinescu G., Lascu M., Old and New Inequalities,
GIL Publishing House, 2004.
[2] Bin X., Boreico I., Can V.Q.B., Bulj A., Lascu M., Opympiad Inequalities, GIL
Publishing House, 2015.
[3] Bin X., Boreico I., Can V.Q.B., Cîrtoaje V., Lascu M., An Introduction to Inequali-
ties, GIL Publishing House, 2015.
[4] Can V.Q.B., Pohoaţă C., Old and New Inequalities, GIL Publishing House, 2008.
[5] Can V.Q.B., Anh T.Q., Su Dung Phuong Phap Cauchy-Schwarz De Chung Minh
Bat Dang Thuc, Nha Xuat Ban Dai Hoc Su Pham, 2010.
[6] Cîrtoaje V., Asupra unor inegalitati cu restrictii, Revista matematica din Timisoara,
Nr. 1, 1990.
[7] Cîrtoaje V., Two Generalizations of Popoviciu’s Inequality, Crux Mathematicorum,
Issue 5, 2005.
[8] Cîrtoaje V., A Generalization of Jensen’s Inequality, Gazeta Matematica-A, Nr. 2,
2005.
[9] Cîrtoaje V., Algebraic Inequalities-Old and New Methods, GIL Publishing House,
2006.
[10] Cîrtoaje V., The Equal Variable Method, Journal of Inequalities In Pure and
Applied Mathematics, Volume 8, Issue 1, 2007.
[11] Cîrtoaje V., On Jensen Type Inequalities with Ordered Variables, Journal of In-
equalities In Pure and Applied Mathematics, Volume 9, Issue 1, 2008.
[12] Cîrtoaje V., The Proof of Three Open Inequalities, Crux Mathematicorum, Vol-
ume 34, Issue 4, 2008.
[13] Cîrtoaje V., Can V.Q.B., Anh T.Q., Inequalities with Beautiful Solutions, GIL Pub-
lishing House, 2009.
[14] Cîrtoaje V., The Best Lower Bound Depended on Two Fixed Variables for Jensen’s
Inequality with Ordered Variables, Journal of Inequalities and Applications, Volume
2010.
549
550 Vasile Cîrtoaje
[15] Cîrtoaje V., The Best Upper Bound for Jensen’s Inequality, Australian Journal of
Mathematical Analysis and Aplications, Volume 7, Issue 2, Art. 22, 2011.
[16] Cîrtoaje V., Baiesu A., An Extension of Jensen’s discrete inequality to half convex
functions, Journal of Inequalities and Applications, Volume 2011.
[17] Cîrtoaje V., The Best Lower Bound for Jensen’s Inequality with three fixed ordered
variables Journal, Banach Journal of Mathematical Analysis, Volume 7, Issue 1,
2013.
[18] Cîrtoaje V., An Extension of Jensen’s discrete inequality to partially convex func-
tions, Journal of Inequalities and Applications, Volume 2013:54.
[19] Cîrtoaje V., On the Equal Variables Method Applied to Real Variables, Creative
Mathematics and Informatics, no. 2, 2015.
[20] Cîrtoaje V., Three extensions of HCF and PCF theorems, Advances in Inequalities
and Applications, no. 2, 2016.
[21] Cîrtoaje V., Extensions of Jensen’s discrete inequality with ordered variables to
half and partially convex functions, Journal of Inequalities and Special Functions,
Volume 8, Issue 3, 2017.
[22] Cîrtoaje V., Mathematical Inequalities - Volume 1, Symmetric Polynomial In-
equalities, Lambert Academic Publishing, 2018.
[23] Cîrtoaje V., Mathematical Inequalities - Volume 2, Symmetric Rational and Non-
rational Inequalities, Lambert Academic Publishing, 2018.
[24] Cîrtoaje V., Mathematical Inequalities - Volume 3, Cyclic and Noncyclic Inequal-
ities, Lambert Academic Publishing, 2018.
[25] Cvetkovski, Inequalities: Theorems, Techniques and Selected Problems, Springer-
Verlag Berlin Heidelberg, 2012.
[26] Hung P.K., Secrets in Inequalities, Volume 1: Basic Inequalities, GIL Publishing
House, 2007.
[27] Hung P.K., Secrets in Inequalities, Volume 2: Advanced Inequalities, GIL Pub-
lishing House, 2008.
[28] Littlewood G.H., Polya J.E., Inequalities, Cambridge University Press, 1967.
[29] Mitrinovic̆ D.S., Pecaric̆ J.E., Fink A.M., Classical and New Inequalities in Anal-
ysis, Kluwer, 1993.